260-Compelete STUDY MATERIAL December

You might also like

Download as pdf or txt
Download as pdf or txt
You are on page 1of 260

STUDY MATERIAL

CLASS – XII
Mathematics
Session – 2022-23

Mathematics Page 1 of 260 XII


KENDRIYA VIDYALAYA SANGATHAN
Regional Office Agra
STUDY MATERIAL 2022-23

ADVISORS
Sh. C. S. Azad
Deputy commissioner KVS RO Agra
Dr. M L Mishra
Assistant Commissioner
KVS Regional Office Agra

COORDINATOR
Sh. S. K. Sharma
Principal (K.V. Lalitpur)
Mathematics Page 2 of 260 XII
CONTENT TEAM
S.N. Name of teacher Designation Name of KV Chapter / Topics
1. Mr. Manish PGT (MATHS) Buland Shahar Relations and Functions
Kumar Sharma (shift-I)
2. Ms. Archna Savita PGT (MATHS) Meerut PL Inverse trigonometric Functions
3. Ms. Suman PGT (MATHS) Aligarh Matrices
Verma
4. Mr. Indrajeet PGT (MATHS) Moradabad Determinants
Singh
5. Mr. Bhavya Singh PGT (MATHS) Meerut DL Continuity and Differentiability
6. Ms. Priyanka PGT (MATHS) Etawah Application of Derivatives
Kaushik
7. Mr. Sumit Kumar PGT (MATHS) Mathura Cantt. Integrals
Sharma
8. Mr. Chhavikant PGT (MATHS) Buland Shahar Application of Integrals
(shift-I)
9. Mr. Pradeep PGT (MATHS) Noida (shift-2) Differential Equations
Kumar Gupta
10. Mr. Vivek Johri PGT (MATHS) Meerut SL Vectors
11. Mr. Ravi Kumar PGT (MATHS) Hindon -2 Three Dimensional Geometry
Garg
12. Mr. Vipin Kumar PGT (MATHS) Baad Mathura Linear programming
13. Ms. Kumudini PGT (MATHS) Agra No. 3 Probability
Kant Pathak
14. Ms. Deepa PGT (MATHS) Lalitpur Compiling and cover pages
Meena preparation

Mathematics Page 3 of 260 XII


MATHEMATICS - XII
(Code No. 041)
Session – 2022-23
No. Units Marks
I Relations and Functions 08
II Algebra 10
III Calculus 35
IV Vectors and Three - Dimensional Geometry 14
V Linear Programming 05
VI Probability 08
Total 80
Internal Assessment 20
Total 100

Unit-I: Relations and Functions


1. Relations and Functions
Types of relations: reflexive, symmetric, transitive and equivalence
relations. One to one and onto functions.
2. Inverse Trigonometric Functions
Definition, range, domain, principal value branch. Graphs of inverse
trigonometric functions.

Unit-II: Algebra
1. Matrices
Concept, notation, order, equality, types of matrices, zero and identity
matrix, transpose of a matrix, symmetric and skew symmetric matrices.
Operation on matrices: Addition and multiplication and multiplication with a
scalar. Simple properties of addition, multiplication and scalar
Mathematics Page 4 of 260 XII
multiplication. On commutativity of multiplication of matrices and existence
of non-zero matrices whose product is the zero matrix (restrict to square
matrices of order 2). Invertible matrices and proof of the uniqueness of
inverse, if it exists; (Here all matrices will have real entries).

2. Determinants
Determinant of a square matrix (up to 3 x 3 matrices), minors, co-factors
and applications of determinants in finding the area of a triangle. Adjoint
and inverse of a square matrix. Consistency, inconsistency and number
of solutions of system of linear equations by examples, solving system of
linear equations in two or three variables (having unique solution) using
inverse of a matrix.
Unit-III: Calculus
1. Continuity and Differentiability
Continuity and differentiability, chain rule, derivative of inverse
trigonometric functions, 𝑙𝑖𝑘𝑒 sin−1 𝑥 , cos −1 𝑥 𝑎𝑛𝑑 tan−1 𝑥, derivative of
implicit functions. Concept of exponential and logarithmic functions .
Derivatives of logarithmic and exponential functions . Logarithmic
differentiation, derivative of functions expressed in parametric forms.
Second order derivatives.

2. Applications of Derivatives
Applications of derivatives: rate of change of bodies,
increasing/decreasing functions, maxima and minima (first derivative test
motivated geometrically and second derivative test given as a provable
tool). Simple problems (that illustrate basic principles and understanding of
the subject as well as real life situations) .
3. Integrals
Integration as inverse process of differentiation. Integration of a variety
of functions by substitution, by partial fractions and by parts, Evaluation of
simple integrals of the following types and problems based on them.
𝑑𝑥 𝑑𝑥 𝑑𝑥 𝑑𝑥 𝑑𝑥
∫ 𝑥2 ±𝑎2 , ∫ √ 2 2 , ∫ √𝑎2 −𝑥2 , ∫ 𝑎𝑥2 +𝑏𝑥+𝑐 , ∫ √𝑎𝑥2 +𝑏𝑥+𝑐 ,
𝑥 ±𝑎
𝑝𝑥+𝑞 𝑝𝑥+𝑞
∫ 𝑎𝑥2 +𝑏𝑥+𝑐 𝑑𝑥 , ∫ √𝑎𝑥 2+𝑏𝑥+𝑐 𝑑𝑥 , ∫ √𝑥 2 ± 𝑎2 𝑑𝑥 ,
Mathematics Page 5 of 260 XII
∫ √𝑎2 − 𝑥 2 𝑑𝑥 , ∫ √𝑎𝑥 2 + 𝑏𝑥 + 𝑐 𝑑𝑥

Fundamental Theorem of Calculus (without proof). Basic properties of


definite integrals and evaluation of definite integrals.

4. Applications of the Integrals


Applications in finding the area under simple curves, especially lines,
circles/parabolas/ellipses (in standard form only)

5. Differential Equations
Definition, order and degree, general and particular solutions of a
differential equation. Solution of differential equations by method of
separation of variables, solutions of homogeneous differential equations of
first order and first degree.
Solutions of linear differential equation of the type:
𝑑𝑦
+ 𝑝𝑦 = 𝑞, where p and q are functions of 𝑥 or constants.
𝑑𝑥
𝑑𝑥
+ 𝑝𝑥 = 𝑞, where p and q are functions of 𝑦 or constants.
𝑑𝑦

Unit-IV: Vectors and Three-Dimensional Geometry


1. Vectors
Vectors and scalars, magnitude and direction of a vector. Direction
cosines and direction ratios of a vector. Types of vectors (equal, unit, zero,
parallel and collinear vectors), position vector of a point, negative of a
vector, components of a vector, addition of vectors, multiplication of a
vector by a scalar, position vector of a point dividing a line segment in a
given ratio. Definition, Geometrical Interpretation, properties and
application of scalar (dot) product of vectors, vector (cross) product of
vectors.
2. Three - dimensional Geometry
Direction cosines and direction ratios of a line joining two points.
Cartesian equation and vector equation of a line, skew lines, shortest
distance between two lines. Angle between two lines.

Mathematics Page 6 of 260 XII


Unit-V: Linear Programming
1. Linear Programming
Introduction, related terminology such as constraints, objective function,
optimization, graphical method of solution for problems in two variables,
feasible and infeasible regions (bounded or unbounded), feasible and
infeasible solutions, optimal feasible solutions (up to three non-trivial
constraints).
Unit-VI: Probability
1. Probability
Conditional probability, multiplication theorem on probability,
independent events, total probability, Bayes’ theorem, Random variable
and its probability distribution, mean of random variable.

INTERNAL ASSESSMENT 20 MARKS


Periodic Tests (Best 2 out of 3 tests conducted) 10 Marks
Mathematics Activities 10 Marks

Note : For activities NCERT Lab Manual may be referred.

Mathematics Page 7 of 260 XII


KVS RO AGRA
Study Material 2022-23
CLASS – XII
SUBJECT – MATHEMATICS
Chapter - 1
Relations and Functions
A relation R in a set A is called :

(i) Reflexive: - If (a, a) ∈ R, for every a∈ A,

Eg1. Let A ={1,2,3} and consider the relation R=


{(1,1),(2,2),(3,3),(1,2),(2,3),(1,3)} . R is reflexive since each element of set A
related to itself.

(ii) Symmetric: - If (a1, a2) ∈ R implies that (a2, a1)∈ R, for all a1, a2∈ A.

Eg2. Let L be the collection of all lines in a plane and R1 be the relation on L
as
R1 = {(L1, L2) : L1  L2 } is symmetric relation since if L1  L2 then L2  L1 .
(iii) Transitive: - If (a1, a2) ∈ R and (a2, a3)∈ R implies that (a1, a3)∈ R, for all
a1, a2, a3∈ A.
Eg3. Let A ={1,2,3} and consider the relation R= {(1,1).(1,2),(2,3),(1,3)} . R is
transitive relation.
** Equivalence Relation : R is equivalence if it is reflexive, symmetric and
transitive.

Eg4. Let T be the collection of all triangles in a plane and R1 be the relation
on T as
R1 = {(T1, T2) : T1  T2 } is reflexive, symmetric and transitive relation so it is
equivalence relation.

Mathematics Page 8 of 260 XII


** Equivalence class : Let R be an equivalence relation on a set A and let a ∈
A, then the set of all those elements of a which are related to a is called
equivalence class determined by a and is denoted as [a]. [a] = {x ∈ A : (x, a) ∈
R}

Eg5. The relation R in the set Z of integers given by R = {(a, b) : 3 divides a –


b} is an equivalence relation.
[3] = {x ∈ Z : x-3 is a multiple of 3} = {..., – 6, – 3, 0, 3, 6, ...}

Some important counting results of Relation


Total number of reflexive relations on A having n elements = 2n(n-1)
Total number of symmetric relations on A having n elements = 2n(n+1)/2
Total number of equivalence relations on A : -
Number of element of A 0 1 2 3 4 5

Total equivalence relation 1 1 2 5 15 52

Function
** Function : A relation f : A →B is said to be a function if every element of A
has unique image in B.

*The Set A is domain of the function

* The Set B is co-domain of the function

* Range f = {f(x) : x ∈ A }

Note: Range ⊆ Co-domain

One - One (Injective) mapping: - If any two different elements in A is always


correlated to different elements in B, i.e. x1≠ x2⇒ f(x1) ≠ f(x2) or f(x1) = f(x2)
⇒ x1 =x2

Mathematics Page 9 of 260 XII


Eg 6. The function f: N→N given by f(x)=3x+2 is one-one

Let x1 , x2∈ N such that f(x1) = f(x2)

 3 x1+2 = 3x2+2

 x1 = x2

So f is one-one

Many one mapping: - If there exist at least two elements in A such that
their images are same..

Into mapping: - if at least one element in B which is not the image of any
element of A.
Or
If range f  Co-domain then f is into.
Onto mapping (Surjective): - If each element in B is having at least one pre-
image
Or
If range f = Co-domain of f then f is onto.
Eg 7. The function f: Z→Z given by f(x)=[x] is onto where [x] denote
greatest integer function.
We have range f = Z , Co-domain = Z
Since range f = Co-domain , so f is onto
Bijective mapping: - If it is both one to one and onto.
Eg 8. The function f: N→N given by f(x)= | x | is one-one and onto . Where
N is set of natural number.

Some important counting results of Function


Let f: A→B where n(A) = m , n(B) = n
Total number of function from A to B is = nm
Total number of one-one function from A to B is = nPm if n  m
Total number of one-one function from A to B is = 0 if n < m
Mathematics Page 10 of 260 XII
Let f: A→B where n(A) = n(B) = m
Total number of one-one function = Total number of onto function = Total
number of Bijective function = m!
SOME EXAMPLES OF CBSE EXAMINATION:

(a) Let A ={1, 2, 3}, then


(i) R = {(1 , 1), (2 , 2), (3 , 3), (1 , 2), (2 , 3) } is reflexive but neither
symmetric nor transitive. As (1 , 1), (2 , 2), (3 , 3)  R ,(1 , 2)  R but (2 , 1) 
R, and (1 , 2), (2 , 3)  R but (1 , 3)  R
(ii) R = {(1,1), (2,2), (1 , 2), (2 , 3)} is neither reflexive nor symmetric nor
transitive.
As (3 , 3)  R , (1 , 2)  R but (2 , 1)  R, and (1 , 2), (2 , 3)  R but (1 , 3)  R
(iii) R = {(1 , 1), (2 , 2), (3 , 3), (1 , 2), (2 , 1), (2 , 3), (3 , 2), (1 , 3), (3 , 1)} is
reflexive, symmetric and transitive ( Equivalence Relation) as (a, a) ∈ R, for
every a ∈ A, (a, b) ∈ R then (b , a)∈ R and (a , b) ∈ R and (b , c)∈ R then (a ,
c)∈ R.

(b) The relation R in the set Z of integers given by R = {(a , b) : 2 divides a –


b} is an equivalence relation.
Given R = {(a , b) : 2 divides a – b}
Reflexive  a – a = 0, divisible by 2,  a  A
 (a , a)  A,  a  A  R is reflexive.
Symmetric : Let (a , b)  R  2 divides a – b , say a – b = 2m
 b – a = –2m
 2 divides b – a
 (b , a)  R  R is symmetric.
Transitive: Let (a , b) , (b , c)  R
 2 divides a – b, say a – b = 2m
 2 divides b – c, say a – b = 2n
 a – b + b – c = 2m + 2n
 a – c = 2(m + n)  2 divides a – c.

Mathematics Page 11 of 260 XII


 (a , c)  R.  R is Transitive.
 R is reflexive, symmetric and transitive
 R is an equivalence relation.
(c) Let A = R – {3} and B = R – {1}. Consider the function f : A  B defined by
 x2
f (x) =   is one-one and onto.
 x 3
 x2
Sol. f: R → R is given by, f(x) =  .
 x 3
 x  2   x2  2 
Let f ( x1)  f ( x 2 )   1    
 x1  3   x 2  3 
 x1 x 2  3x1  2x 2  6  x1 x 2  3x 2  2x1  6
 x1  x 2  f is a one - one
x2 3y  2
Let y  f ( x )   xy  3y  x  2  x 
x 3 y 1
3y  2
Therefore, for any y  B, there exists A
y 1
 f is onto.
 f is one-one and onto.
(d) Show that the relation R in the set R of real numbers, defined as R = {(a,
b) : a ≤ b2} is neither reflexive nor symmetric nor transitive

Sol. (1/2 , 1/2)  R , so it is not reflexive

(1,2)  R but (2,1)  R , so it is not symmetric

(8,3)  R , (3,2)  R but (8,2)  R , so it is not transitive

(e) Let A = {1, 2, 3} and R = {(1, 1), (2, 3), (1, 2)} be a relation on A, then write
the minimum number of ordered pairs to be added in R to make R
equivalence.

Sol. Three ordered pairs (2,2), (3,3) and (1,3) should be added to make it
equivalence.
Mathematics Page 12 of 260 XII
MULTIPLE CHOICE QUESTIONS

1. Let T be the set of all triangles in the Euclidean plane,and let a


relation R on T be defined as T1 R T2 if T1 is similar to T2 for all T1 ,T2
∈ T.Then R is
(A) reflexive but not transitive (B) transitive but not symmetric
(C) equivalence (D) none of these
2. If a relation R on the set A = {a,b,c}, then number of symmetric
relation on A is
(A) 16 (B) 64
(C) 9 (D) none of these
3. If f: R→R given by f(x) = [x] where [x] denote greatest integer
function, then
(A) f is one one onto (B) f is many one onto
(C) f is one one but not onto (D) f is neither one one
nor onto
4. Let A = {1,2,3,4.5}. The number of bijective function from A to A is
(A) 100 (B) 120 (C) 36 (D) 64
2
5. Let f:R→R defined by f(x) = x . Choose the correct answer
(A) f is one one onto (B) f is many one onto
(C)f is one one but not onto (D) f is neither one one
nor onto
6. Let f:R→R defined by f(x) = 5x -2 Choose the correct answer
(A) f is one one onto (B) f is many one onto
( C) f is one one but not onto (D) f is neither one one nor onto
7. If A = {1,2,3}, B = {4,6,9} and R is a relation from A to B defined by ‘ x
is smaller than y’. The range of R is
(A) {1,4,6,9} (B) {4,6,9} (C) {1} (D) none of these
8. The relation R = { (1,1),(2,2),(3,3)} on {1,2,3} is
(A) symmetric only (B) reflexive only
(C) transitive only (D) an equivalence relation
9. Let A ={1,2,3} and consider the relation R= {(1,1),(2,2),(3,3),(1,2)}
then R is
(A) reflexive but not symmetric (B) reflexive but not transitive
(C) reflexive and transitive (D)neither symmetric nor transitive
Mathematics Page 13 of 260 XII
10. Let us define a relation R in R as aRb if a= b .Then R is
(A) an equivalence relation
(B) reflexive,transitive but not symmetric
(C) symmetric ,transitive but not reflexive
(D) neither transitive nor reflexive but symmetric

ANSWERS
1. 2. 3. 4. 5. 6. 7. 8. 9. 10.

C B D B D A B D C A

Solutions :
1. Every triangle is similar to itself so reflexive
If T1 is similar to T2 then T2 also to T1, so symmetric
Similarly transitive also
Hence equivalence
2. Number of symmetric relation on A 23(3+1)/2 =64
3. We have [1.1] =1 , [1.2] =1 , since two elements have same
image so it is not one -2 , Range of [x] =set of integer not
equal to codomain (R) , so it is not onto
4. The number tof bijective function from A to A = 5! =120
5. Since f(-1)= f(1) =1, since two elements have same image so
it is not one -2 , Again Range f is not equal to Co-domain , so
it is not onto.
6. Linear function from R to R is always one-2 onto.
7. R= {(1,4),(1,6),(1,9), (2,4),(2,6),(2,9), (3,4),(3,6),(3,9)}
Range f = {4,6,9}
8. It is the smallest equivalence relation in given set.
9. By definition it is reflexive and transitive .
10. Since a=a for all a , so it is reflexive

Mathematics Page 14 of 260 XII


If a=b then b=a also, so it is symmetric
If a=b, b= c than a=c also , so it is transitive also
Hence equivalence.

Three / Five marks Questions


1. Check whether the relation R in R defined by R = {(a, b) : a ≤ b3} is
reflexive, symmetric or transitive.

2. Let f : N  N be defined by
n 1
 , if n is odd
f (n) =  2 for all n  N
n
 , if n is even
 2
State whether the function f is bijective? Justify your answer.
3. Show that the relation R defined in the set A of all polygons as R = {(P 1, P2)
: P1 and P2 have same number of sides}, is an equivalence relation. What is
the set of all elements in A related to the right angle triangle T with sides 3,
4 and 5?

4. Let f : X  Y be a function. Define a relation R in X given by R = {(a, b): f(a)


= f(b)}. Examine if R is an equivalence relation.

5. Given a non-empty set X, consider P(X) which is the set of all subsets of X.
Define the relation R in P(X) as follows: For subsets A, B in P(X), ARB if and
only if A  B. Is R an equivalence relation on P(X)? Justify your answer

6. Consider f : R+  [– 5,  ) given by f (x) = 9x2 + 6x – 5. Show that f is one-


one and onto.
Hints/Solutions:
1. Since (-2, -2)  R, so it is not reflexive
(1, 2)  R but (2, 1)  R , so it is not symmetric
Since (26, 3), (3, 2)  R but (26 , 2)  R , so it is not transitive.

Mathematics Page 15 of 260 XII


2. Since the elements 1 and 2 have same image so it is not one-one
Here range f= N= Co-domain, so it is onto
Since it is not one-one, so it is not bijective.

3. Since P and P have same number of sides, so reflexive


If P1 and P2 have same number of sides, than If P 2 and P1 also having same
number of sides, so symmetric.
If P1 and P2 have same number of sides, than If P2 and P3 have same number
of sides , Then P1 and P3 also having same number of sides , so transitive
 R is reflexive, symmetric and transitive.
 R is an equivalence relation.

4. Since f(a) = f(a) , so reflexive


If f(a) = f(b) than f(b) = f(a) , so symmetric
If f(a) = f(b) ,f(b) = f(c) than If f(a) = f(c) , so transitive
 R is reflexive, symmetric and transitive.
 R is an equivalence relation.

5. Since A  A for all A, so reflexive


If A={1} , B = {1,2} , here A subset of B but B is not subset of A , so it is not
symmetric
If A  B, B  C than A  C so transitive
 R is not an equivalence relation
A will be the set of all triangle
6. Let x1 , x2 R*( Set of non-negative real number)

f(x1) = f(x2)  9x12 + 6x1 -5 = 9x22 + 6x2 -5

3(x1 – x2)[3(x1 + x2) + 2] = 0

(x1 – x2) =0 since 3(x1 + x2) + 2 ≠ 0, as x1 , x2 R*

x1 = x2, so f is one-one

For onto let y  [-5, ∞) has a pre image x  R*


Mathematics Page 16 of 260 XII
 f(x) = y  9x2 + 6x - 5 = y (3x + 1)2 = y + 6

3𝑥 + 1 = ± √𝑦 + 6

3𝑥 + 1 = √𝑦 + 6 since x R*

1
𝑥= ( √𝑦 + 6 − 1)
3
1
Since 𝑥 ≥ 0  3 (√𝑦 + 6 − 1) ≥ 0𝑦 ≥ −5

Range = [-5, ∞) = co-domain

So f is onto

Assertion and Reason Questions


The following questions consist of two statements Assertion (A) and Reason
(R), Answer these questions selecting the appropriate option given below.

(P) Both A and R are correct; R is the correct explanation of A.


(Q) Both A and R are correct; R is not the correct explanation of A.
(R) A is correct; R is incorrect.
(S) R is correct; A is incorrect.
(1) Assertion (A) : Let f: RR given by f(x) = 5x -3, then f is a one-one
function.
Reason (R) : A function f: A  B is said to be onto if for each b ∈ B,  a ∈ A
such that
f(a) = b
(i) P (ii) Q (iii) R (iv) S

(2) Assertion (A) : Let L be the collection of all lines in a plane and R 1 be the
relation on L as R1 = {(L1, L2) : L1 || L2 } is symmetric relation.

Reason (R) : Any relation R on a set A is symmetric relation, if (a1, a2) ∈ R


implies that , (a2, a1)∈ R, for all a1, a2∈ A.
(i) P (ii) Q (iii) R (iv) S
Mathematics Page 17 of 260 XII
(3) Assertion (A) : The relation R on the set of integers Z given by R = {(a, b) :
a=b} is an equivalence relation.
Reason (R) : Any relation R on a set A is an equivalence relation, if it is
reflexive, symmetric and transitive.
(i) P (ii) Q (iii) R (iv) S

(4) Assertion (A) : The relation R on the set of male human beings given by
R = {(a, b) : a is a brother of b} is a reflexive relation.
Reason (R) : Any relation R on a set A is a reflexive relation, if (a, a) ∈ R, for
every a∈ A
(i) P (ii) Q (iii) R (iv) S

(5) Assertion (A) : Let L be the collection of all lines in a plane and R 1 be the
relation on L as R1 = {(L1, L2) : L1  L2 } is transitive relation.
Reason (R) : Any relation R on a set A is transitive relation, if (a1, a2) ∈ R and
(a2, a3)∈ R implies that (a1, a3)∈ R, for all a1, a2, a3∈ A.
(i) P (ii) Q (iii) R (iv) S

(6) Assertion (A) : Let A and B are two sets having 3 and 5 elements
respectively, then total number of one-one function from A to B is 50.
Reason (R) : If A and B are two sets having m and n elements (n > m)
respectively, then total number of one-one function from A to B is nPm .
(i) P (ii) Q (iii) R (iv) S

(7) Assertion (A) : Let L be the collection of all lines in a plane and R 1 be the
relation on L as R1 = {(L1, L2) : L1 || L2 } is reflexive relation.
Reason (R) : Any relation R on a set A is symmetric relation, if (a1, a2) ∈ R
and (a2, a3)∈ R implies that (a1, a3)∈ R, for all a1, a2, a3∈ A.
(i) P (ii) Q (iii) R (iv) S

(8) Assertion (A) : Let f: RR given by f(x) = x , then f is a onto function.
Reason (R) : A function f is said to be onto if range(f) = co-domain
(i) P (ii) Q (iii) R (iv) S

Hints and solution


Mathematics Page 18 of 260 XII
1. Linear function of one degree from R to R is always one-2
Answer (ii) Q
2. If one line is || to second then second also || to first
Answer (i) P
3. Since it is reflexive ,symmetric and transitive , so it is equivalence .
Answer (i) P
4. Any person can be brother of himself so it is reflexive.
Answer (i) P
5. If L1  L2 and L2  L3 then L1 not  L3 . So it is not transitive
Answer (iv) S
6. Total number of one-2 function is 5P3 = 60
Answer (iv) S
7. Any line is || to itself, so R is reflexive.
This definition is for transitive relation .
Answer (iii) R
8. Here range(f) = set of non-negative real number ≠ codomain
Answer (i) S

Case study question


500 people of a city used to work in an IT company. The company used to
work in two shifts, day and night . Fewer workers were employed in the
night shift than the day shift. Since the transport is less during the night
than that of day, the company used to provide transport facility for the night
workers.

Mathematics Page 19 of 260 XII


Let A be the set of employees who worked in both shifts. A relation ‘R‘ is
defined on A as follows
R = { (P,Q ) : P and Q worked in same shift of company }

(i) Two friends A and B worked in the company but not in the same
shift. Then which of the following is true?
(a) (A,B) ∈ R (b) (A,B)  R (c) A ,B ∈ R
(d) None of these

(ii) Three friends X, Y and Z worked in company. Then which of the


following is true?
(a) If (X,Y) ∈ R , (Y,Z) ∈ R then (X,Z)  R (b) if (X,Y)  R , (Y,Z) 
R then (X,Z)  R
(c) if (X,Y) ∈ R , (Y,Z) ∈ R then (X,Z) ∈ R (d) None of these

(iii) Two friends P and Q worked in the company . Then which of the
following is true?
(a) If (P,Q) ∈ R then (Q,P) ∈ R (b) (P,Q) R but (Q,P) ∈ R
(c) (P,Q) ∈ R but (Q,P)  R (d) None of these

(iv) The above defined relation R is ……….


(a) Reflexive and Symmetric but not Transitive
(b) Symmetric, Transitive but not Reflexive
(c) Equivalence relation
(d) None of these
Hints and solution
Mathematics Page 20 of 260 XII
(i) Since they are not in same shift so (A,B)  R
Answer (b)
(ii) Since X and Y, Y and Z worked in same shift .So all are in
same shift
Answer (c)
(iii) If P and Q worked in same company than Q, P also
Answer (a)
(iv) Clearly the relation is Reflexive , Symmetric, Transitive so
it is equivalence
(v) Answer (c)

ITT-JEE QUESTIONS

1. Let f is a function defined on [0, 2], then find the domain of the function
g(x) = f(9x2-1)
2. Find the inverse of the function f : R→(-∞,1) given by f(x) = 1- 2-x
3. f : N→N where f(x) = x - (-1)x , Then check f is bijective or not
4. Find the period of the function f(x) = elog (sinx) + tan3x – cosec (3x-5)
5. The function f : [0, 3] → [1, 29] defined by f(x) = 2x3 – 15x2 + 36x + 1
(a) one-one and onto
(b) onto but not one-one
(c) one-one but not onto
(d) neither one-one nor onto
Hints and solution
1. g is meaningful if 0 ≤ 9x2-1 ≤2  1 ≤9x2 ≤3

 1 1  1 1 
i.e x   ,    , 
 3 3 3 3 

2. f(x) = 1- 2-x = y
Or 2-x = y+1

Mathematics Page 21 of 260 XII


Or –x = log 2(1-y)
Or f-1(x) = - log2(1-x)

3. Since different elements of domain have different image, so it is one-one


Also range f = Co-domain , so f is onto
4. The period of the function elog (sinx) is 2π, of tan3x is π , of cosec (3x-5) is
2
3
2
So period of f(x) = LCM { 2π, π , } = 2π
3
5. The function f : [0, 3] → [1, 29] defined by
f(x) = 2x3 – 15x2 + 36x + 1
f'(x) = 6x2 – 30x + 36
= − 6(x2 – 5x + 6) - +
-
= - 6(x – 2)(x – 3) 2 3
f(x) is increasing in [0, 2] and decreasing in [2, 3].
We know, if function is strictly increasing or decreasing in its domain, then it
is one-one. But given function is increasing as well as decreasing.
So, f(x) is many one.
f (0) = 1
f (2) = 29 and
f (3) = 28
Range is [1, 29].
The function is onto.
So answer is (b)

Mathematics Page 22 of 260 XII


Chapter -2
Inverse Trigonometric Functions

Theoretical concepts with examples:


Inverse Trigonometric Functions: Trigonometric functions are many-one
functions but we know that inverse of function exists if the function is
bijective. If we restrict the domain of trigonometric functions, then these
functions become bijective and the inverse of trigonometric functions are
defined within the restricted domain. The inverse of f is denoted by f-1.
Let y = f(x) = sin x, then its inverse is x = sin-1 y.

Domain and R

angle of Inverse Trigonometric Functions

Property Set 1:
 sin−1(x) = cosec−1(1/x), x∈ [−1,1]−{0}
 cos−1(x) = sec−1(1/x), x ∈ [−1,1]−{0}
 tan−1(x) = cot−1(1/x), if x > 0 (or) cot−1(1/x) −π, if x < 0
 cot−1(x) = tan−1(1/x), if x > 0 (or) tan−1(1/x) + π, if x < 0

Mathematics Page 23 of 260 XII


Property Set 2:
 sin−1(−x) = −sin−1(x)
 tan−1(−x) = −tan−1(x)
 cos−1(−x) = π − cos−1(x)
 cosec−1(−x) = − cosec−1(x)
 sec−1(−x) = π − cec−1(x)
 cot−1(−x) = π − cot−1(x)
Proofs:
1. Sin−1(−x) = −Sin−1(x)
Let sin−1(−x) = y, i.e.,−x = sin y
⇒ x = − sin y
Thus,
x = sin (− y)
Or,
sin−1(x) = −y = −sin−1(−x)
Therefore, sin−1(−x) = −sin−1(x)

Similarly, using the same concept following results can be obtained:

 cosec−1(−x) = −cosec−1x, |x|≥1


 tan−1(−x) = −tan−1x, xϵR
2. Cos−1(−x) = π − Cos−1(x)
Let cos−1(−x) = y i.e., −x = cos y
⇒ x = −cos y = cos(π–y)
Thus,
cos−1(x) = π–y
Or,
cos−1(x) = π–cos−1(−x)
Therefore, cos−1(−x) = π–cos−1(x)
Similarly using the same concept following results can be obtained:

 sec−1(−x) = π–sec−1x, |x|≥1


 cot−1(−x) = π–cot−1x, xϵR

Mathematics Page 24 of 260 XII


Property Set 3:

 Sin−1(1/x) = cosec−1x, x≥1 or x≤−1


 Cos−1(1/x) = sec−1x, x≥1 or x≤−1
 Tan−1(1/x) = −π + cot−1(x)
Proof: Sin−1(1/x) = cosec−1x, x≥1 or x≤−1
Let cosec−1 x = y, i.e. x = cosec y
⇒ (1/x) = sin y
Thus, sin−1(1/x) = y
Or,
sin−1(1/x) = cosec−1x
Similarly using the same concept the other results can be obtained.
Illustrations:

 sin−1(⅓) = cosec−1(3)
 cos−1(¼) = sec−1(4)
 sin−1(−¾) = cosec−1(−4/3) = sin−1(3/4)
 tan−1(−3) = cot−1(−⅓)−π

Property Set 4:

 sin−1(cos θ) = π/2 − θ, if θ∈[0,π]


 cos−1(sin θ) = π/2 − θ, if θ∈[−π/2, π/2]
 tan−1(cot θ) = π/2 − θ, θ∈[0,π]
 cot−1(tan θ) = π/2 − θ, θ∈[−π/2, π/2]
 sec−1(cosec θ) = π/2 − θ, θ∈[−π/2, 0]∪[0, π/2]
 cosec−1(sec θ) = π/2 − θ, θ∈[0,π]−{π/2}

sin-1(sinθ) = θ; ∀ θ ∈ [−π/2,π/2]
cos-1(cosθ) = θ; ∀ θ ∈ [0, π]
tan-1(tanθ) = θ; ∀ θ (−π/2,π/2)
cosec-1(cosecθ) = 0; ∀ θ ∈ [−π/2,π/2] , θ ≠ 0

Mathematics Page 25 of 260 XII


sec-1(secθ) = θ; ∀ θ ∈ [0, π], θ ≠ π/2
cot-1(cotθ) = θ; ∀ θ ∈ (0, π)
sin(sin-1 x) = x, ∀ x ∈ [-1, 1]
cos(cos-1 x) = x; ∀ x ∈ [-1, 1]
tan(tan-1x) = x, ∀ x ∈ R
cot(cot-1x) = x, ∀ x ∈ R
sec(sec-1x) = x, ∀ x ∈ (-∞, -1] ∪ [1, ∞)
cosec(cosec-1x) = x, ∀ x ∈ (-∞, -1] ∪ [1, ∞)
Note: sin-1(sinθ) = θ ; sin-1 x should not be confused with (sinx)-1 = 1sinx or
sin-1 x = sin-1(1x) land similarly for other trigonometric functions.
The value of an inverse trigonometric function, which lies in the range of
principal value branch, is called the principal value of the inverse
trigonometric function.
Note: Whenever no branch of an inverse trigonometric function is
mentioned, it means we have to consider the principal value branch of that
function.

Arcsine Function
Arcsine function is an inverse of the sine function denoted by sin -1x. It is
represented in the graph as shown below:

Domain -1 ≤ x ≤ 1

Range -π/2 ≤ y ≤ π/2

Mathematics Page 26 of 260 XII


Arccosine Function

Arccosine function is the inverse of the cosine function denoted by cos -1x. It
is represented in the graph as shown below:

Therefore, the inverse of cos function can be expressed as; y = cos-


1
x (arccosine x)
Domain & Range of arcsine function:

Domain -1≤x≤1

Range 0≤y≤π

Arctangent Function

Arctangent function is the inverse of the tangent function denoted by tan -1x.
It is represented in the graph as shown below:

Mathematics Page 27 of 260 XII


Therefore, the inverse of tangent function can be expressed as; y = tan-
1
x (arctangent x)
Domain & Range of Arctangent:

Domain -∞ < x < ∞

Range -π/2 < y < π/2

Arccotangent (Arccot) Function

Arccotangent function is the inverse of the cotangent function denoted by


cot-1x. It is represented in the graph as shown below:

Mathematics Page 28 of 260 XII


Therefore, the inverse of cotangent function can be expressed as; y = cot-
1
x (arccotangent x)
Domain & Range of Arccotangent:

Domain -∞ < x < ∞

Range 0<y<π

Arcsecant Function

What is arcsecant (arcsec)function? Arcsecant function is the inverse of the


secant function denoted by sec-1x. It is represented in the graph as shown
below:

Therefore, the inverse of secant function can be expressed as; y = sec-


1
x (arcsecant x)
Domain & Range of Arcsecant:

Domain -∞ ≤ x ≤ -1 or 1 ≤ x ≤ ∞

Range 0 ≤ y ≤ π, y ≠ π/2

Mathematics Page 29 of 260 XII


Arccosecant Function

What is arccosecant (arccsc x) function? Arccosecant function is the inverse


of the cosecant function denoted by cosec-1x. It is represented in the graph
as shown below:

Therefore, the inverse of cosecant function can be expressed as; y = cosec-


1
x (arccosecant x)
Domain & Range of Arccosecant is:

Domain -∞ ≤ x ≤ -1 or 1 ≤ x ≤ ∞

Range -π/2 ≤ y ≤ π/2, y ≠ 0

Example 1: Find the value of x, for sin(x) = 2.


Solution: Given: sin x = 2
x =sin-1(2), which is not possible.
Hence, there is no value of x for which sin x = 2; since the domain of sin -1x is
-1 to 1 for the values of x.
Example 2: Find the value of sin-1(sin (π/6)).
Solution:
sin-1(sin (π/6) = π/6 (Using identity sin-1(sin (x) ) = x)

Mathematics Page 30 of 260 XII


Example 3: Find sin (cos-1 3/5).
Solution:
Suppose that, cos-1 3/5 = x
So, cos x = 3/5
We know, sin x=1–cos2

So, sin x=1–925=45 This implies, sin x = sin (cos-1 3/5)

Example 4.Write the value of

tan-1 (√3) – cot-1 (- √3). (All India 2019,13)


Answer:
We have, tan-1 (√3) – cot-1 (-√3)
= tan-1 (√3) – {π – cot-1 (√3)} [∵ cot-1 (- x) = π – cot-1 x; x ∈ R]
= tan-1 √3 – π + cot-1 √3
= (tan-1 √3 + cot-1 √3) – π
= π2 – π = – π2 [∵ tan-1x + cot-1x = π2; x ∈ R]

Which is the required principal value.

Example 5.Find the principal value of


tan-1√3 – sec-1 (- 2). (CBSE 2018 C; All India 2012)
Answer:

Mathematics Page 31 of 260 XII


We have, tan-1√3 – sec-1 (- 2)

Example 6.If sin (sin-11/5 + cos-1x) = 1, then find the value of x. (Delhi 2014)
Answer:

Mathematics Page 32 of 260 XII


Example 7. Write the value of cos-1(−1/2) + 2 sin-1(1/2). (Foreign 2014)
Answer:

Mathematics Page 33 of 260 XII


Example8. Write the principal value of tan-1[sin(−π/2)]. (All India 2014C)
Answer:

Example 9. Find the value of the following.


cot (π/2 – 2 cot-1√3) (All India 2014C)
Answer:
First, use cot(π/2−θ) = tan θ, then put
cot-1 √3 = π/6 and simplify it .

Example 10.Write the principal value of the following.


[cos-1√3/2+cos-1(−1/2)] (Delhi 2013C)

Mathematics Page 34 of 260 XII


Answer:

MULTIPLE CHOIUCE QUESTIONS :


Question 1.
If tan-1 (cot θ) = 2θ, then θ is equal to
(a) π/3
(b) π/4
(c) π/6
(d) None of these
Answer:
(c) π/6
Question 2.
If sin-1 x – cos-1 x = π/6, then x =
(a) 1/2
(b) √3/2
(c) −1/2
(d) -√3/2
Answer:
(b) √3/2
Question 3.
If sin-1(x2 – 7x + 12) = nπ, ∀ n ∈ I, then x =
(a) -2
(b) 4
Mathematics Page 35 of 260 XII
(c) -3
(d) 5
Answer:
(b) 4
Question4.
cos-1 x + sin-1 x = π, then the value of x is
(a) 1/2
(b) 1/√2
(c) √3/2
(d) -√3/2
Answer:
(c) √3/2
Question5.If sin-1 x – cos-1 x = π/6, then x =
(a) ½
(b) 1/√2
(c) √3/2
(d) -√3/2
Answer:
(c) √3/2
Question6.If tan-1 (cot θ) = 2θ, then θ is equal to
(a) π/3
(b) π/4
(c) π/6
(d) None of these
Answer:
(c) π/6

Mathematics Page 36 of 260 XII


Question 7.

Answer:
(d) −π/6
Questio8 .

Answer:
(b) π//3
Question 9.

Answer:
(b) π/3

Mathematics Page 37 of 260 XII


Question 10.

Answer:
(a) π/4
Question 11.

Answer:
(b) 3π/4

JEE QUESTIONS
Question 1: A value of x satisfying the equation sin[cot−1(1 + x)] = cos[tan-1x],
is:
(a) 1/2
(b) -1
(c) 0
(d) -1/2
Answer: (d)
Solution:
Given equation is: sin[cot −1(1 + x)] = cos[tan-1x]
Let cot−1(1 + x)] = a => cot a = 1 + x …(1)

Mathematics Page 38 of 260 XII


And tan-1x =b => x = tan b …(2)
Solve (1) for a in terms of sin function:
cot a = 1 + x
We know, cosec a = √(1+cot2a) = √(1 + (1 + x)2 = √(x2 + 2x + 2)
Also, sin a = 1/cosec a
⇒ sin a = 1/√(x2 + 2x + 2)
Or a = sin-1[1/ √(x2 + 2x + 2)]
Solve (2) for b in terms of cos function:
x = tan b
We know, sec b = √(1+tan2b) = √(1+x2)
Also, cos b = 1/sec b = 1/√(1+x2)
Or b = cos-1[1/√(1+x2)]
Given equation =>
sin(sin-1[1/ √(x2 + 2x + 2)]) = cos[cos-1(1/√(1+x2))]
⇒ 1/ √(x2 + 2x + 2) = 1/√(1+x2)
Squaring both sides and solving, we get
2x = -1
Or x = -1/2
Question 2: 1 + cot2 (sin-1 x) =
(a) 1/2x
(b) x2
(c) 1/x2
(d) 2/x
Solution:

Mathematics Page 39 of 260 XII


Let sin-1 x = θ
So sin θ = x
1 + cot2 θ = cosec2 θ
= 1/x2
Hence option c is the answer.
Question 3: If α = 3 sin-1(6/11) and β = 3 cos-1 (4/9) where the inverse trig
functions take only the principal values, then the right option is
(a) cosβ > 0
(b) cos(α + β) > 0
(c) sin β < 0
(d) cosα < 0
Answer: (b), (c) and (d)
Solution:
α = 3 sin-1 (6/11) > 3 sin-1 (6/12) > 3 sin-1 (1/2) > π / 2
β = 3 cos-1 (4/9) > 3 cos-1 (4/8) > 3 cos-1 (1/2) > π
α + β > 3π/2
sin β < 0,cos α < 0,cos (α + β) > 0
Question 4: If f'(x) = tan-1(sec x + tan x), -π/2 < x < π/2, and f(0) = 0 then f(1)
is equal to
(a) (π+1)/4
(b) (π+2)/4
(c) ¼
(d) (π-1)/4
Answer: (a)
Solution:

Mathematics Page 40 of 260 XII


f'(x) = tan-1(sec x + tan x) = tan-1(1/cosx + sin x/cos x) = tan-1[(1+sin x)/cos x]

f'(x) = π/4 + x/2


⇒ f(x) = (π/4) x + x2/4 + c
At f(0) = 0 => c = 0
At f(1) = π/4 + 1/4 = (π+1)/4
Question 5: The value of

|x| < 1/2, x ≠ 0, is equal to


(a) π/4 + (1/2) cos-1x2
(b) π/4 + cos-1x2
(c) π/4 – (1/2) cos-1x2
(d) π/4 – cos-1x2
Mathematics Page 41 of 260 XII
Answer: (a)
Solution: Let x2 = cos 2A => A = (1/2) cos-1 (x2) …(1)
tan−1[1+cos 2A+1−cos 2A1+cos 2A–1−cos 2A]
=tan−1[2cos2 A+2sin2 A2cos2A–2sin2A]
=tan−1[2cosA+2sinA2cosA–2sinA]
=tan−1[1+tan A1–tan A]
We know tan π/4 = 1; using in the above equation, we get
= tan-1(tan(π/4 + A))
= π/4 + A
Using (1)
= π/4 + (1/2) cos-1(x2)
Question 6:
The value of cot[∑n=123cot−1(1+∑k=1n2k)] is
(a) 23/25
(b) 25/23
(c) 23/24
(d) 24/23
Answer: (b)
Solution:
cot[∑n=123cot−1(1+∑k=1n2k)]
=cot[∑n=123cot−1(1+2×n(n+1)2]
=cot[∑n=123cot−1(n2+n+1)]
=cot[∑n=123tan−1(n+1−n1+n(1+n))]
=cot[∑n=123tan−1(n+1)–tan−1n]
=cot[tan−1(24)–tan−1(1)]
=cot[tan−12325]
=cot[cot−12523]
= 25/23

Mathematics Page 42 of 260 XII


Question 7: Let f: [0, 4π] -> [0, π] be defined by f(x) = cos-1(cos x). The
number of points x Є [0, 4π] satisfying the equation f(x) = (10-x)/10 is
(a) 1
(b) 2
(c) 3
(d) None of these
Answer: (c)
Solution:
Draw graph for f(x) = cos-1(cos x) and f(x) = (10-x)/10

Both equations intersect at three different points, so the number of


solutions is 3.

Mathematics Page 43 of 260 XII


Chapter - 3
Matrices

SOME IMPORTANT THEORITICAL CONCEPTS


A matrix is a rectangular array of m  n numbers arranged in m rows and n
columns.
a11 a12 …………a1n 
a a 22 …………a 2n 
A  21
OR A = [a ij ]mn , where i = 1, 2,…., m ; j =
 
 
a m1 a m2 …………a mn  mn
1, 2,….,n.
* Row Matrix : A matrix which has one row is called row matrix. A  [a ij ]1n
* Column Matrix: A matrix which has one column is called column matrix.
A  [a ij ]m 1 .
* Square Matrix: A matrix in which number of rows are equal to number of
columns, is called a square matrix A  [a ij ]m  m
* Diagonal Matrix: A square matrix is called a Diagonal Matrix if all the
elements, except the diagonal elements are zero. A  [a ij ]n  n , where a ij
= 0 ,i j.
a ij 0 ,i = j.
* Scalar Matrix: A square matrix is called scalar matrix if all the elements,
except diagonal elements are zero and diagonal elements are some non-
zero quantity.
a ij
A  [a ij ]n  n , where a ij = 0 ,i j.  , i = j.
* Identity or Unit Matrix : A square matrix in which all the non diagonal
elements are zero and diagonal elements are unity is called identity or unit
matrix.
* Null Matrices : A matrices in which all element are zero.

Mathematics Page 44 of 260 XII


* Equal Matrices : Two matrices are said to be equal if they have same order
and all their
corresponding elements are equal.
* Transpose of matrix : If A is the given matrix, then the matrix obtained by
interchanging the rows and columns is called the transpose of a matrix.\
* Properties of Transpose :
If A & B are matrices such that their sum & product are defined, then
 
(i). A T
T
 A (ii). A  B  A T  BT
T

(iii). ( KA) T  K.A T where K is a scalar.


(iv). AB  B T A T
T
.
* Symmetric Matrix : A square matrix is said to be symmetric if A = AT
i.e. If A  [a ij ]m  m , then a ij  a ji for all i, j. Also elements of the symmetric
matrix are symmetric about the main diagonal
* Skew symmetric Matrix : A square matrix is said to be skew symmetric if
AT = - A.
If A  [a ij ]m  m , then a ij  a ji for all i, j, and diagonal elements are 0.
*Singular matrix:A square matrix ‘A’ of order ‘n’ is said to be singular, if | A|
= 0.
𝑎 𝑏] 𝑒 𝑓
*Let A = [ and B = [ ] are two matrices, then
𝑐 𝑑 𝑔 ℎ
𝑎+𝑒 𝑏+𝑓
(i) Addition of matrices : A + B = [ ]
𝑐+𝑔 𝑑+ℎ
𝑎−𝑒 𝑏−𝑓
(ii) Subtraction of matrices : A - B = [ ]
𝑐−𝑔 𝑑−ℎ
𝑎𝑒 + 𝑏𝑔 𝑎𝑓 + 𝑏ℎ
(iii) product of matrices : AB = [ ]
𝑐𝑒 + 𝑑𝑔 𝑐𝑓 + 𝑑ℎ

*Properties of Product of matrices:


1. If A & B are two matrices, then product AB is defined, if Number of
column of A = number of rows of B.
i.e. A  [a ij ] m  n , B  [b j k ] n  p then AB = AB  [Cik ] m  p .

Mathematics Page 45 of 260 XII


2. Product of matrices is not commutative. i.e. AB  BA.
3. Product of matrices is associative. i.e A(BC) = (AB)C
4. Product of matrices is distributive over addition.
*Inverse of a matrix: (i) A-1 exists only when |𝐴| ≠ 0, i.e. A is non-singular.
1
(ii) A-1 = |𝐴| (adj A)
*Properties of inverse: (i) A A-1 = A-1A = I (ii) (AB)-1 = B-1 A-1
1
(iii) (A-1)-1 = A (iv) |𝐴−1 | = |𝐴| .
(v) (AT)-1 = (A-1)T
*Properties of adjoint of a matrix: (i) adj (AB) = adj A. adj B
(ii) A (adj A) = (adj A) A = |𝐴| I
(iii) |𝑎𝑑𝑗 𝐴| = |𝐴|n-1 ,where A is a non-singular matrix of
order n.

MULTIPLE CHOICE QUESTIONS

𝑐𝑜𝑠𝛼 𝑠𝑖𝑛𝛼
1. If A = [ ] , 𝑎𝑛𝑑 𝐴 + 𝐴𝑇 = 𝐼 then the value of 𝛼 is
−𝑠𝑖𝑛𝛼 𝑐𝑜𝑠𝛼
𝜋 𝜋 𝜋 𝜋
(A) (B) 3 (C) 4 (D) 6
2

1 , 𝑤ℎ𝑒𝑛 𝑖 ≠ 𝑗
2. If A = [aij] is a square matrix of order 2 such that a ij = { ,
0 , 𝑤ℎ𝑒𝑛 𝑖 = 𝑗
then A2 is,

1 0 1 1 1 0 1 0
(A) [ ] (B) [ ] (C) [ ] (D) [ ]
1 0 1 0 1 1 0 1

3. Total number of possible matrices of order 2X3 with each entry 1 or 0 is

A) 6 B) 32 C) 36 D) 64

4. If A is a square matrix such that A2= A, then (I+A) 2 – 3 A is :

A) I (B) 3 I (C) A (D) 2A

Mathematics Page 46 of 260 XII


5. If A and B are matrices of order 3×m and 3×n respectively such that m =
n, then order of 2A + 9B is

A) 3×m B) 3×3 C) m×n D) n×m

4 3
6. If A=[ ] and A2 - xA + yI=O, then (x, y) is:
2 5

A) (3, 7) B) (9, 14) C) (5, 14) D) (3, 14)

2𝑥 0 3 0
7. If A=[ ] and A-I = [ ] , then x=?
𝑥+2 4𝑥 − 1 4 6

A) 1 B) 2 C) ½ D) -2

4 −3
8. If A and B are 2 X 2 square matrices and A+B = [ ]and A-B =
1 6
−2 1
[ ], then AB=?
5 2
5 −4] 7 −5] 7 −1 7 −1
𝐴) [ B) [ C) [ ] D)[ ]
1 2 1 −5 5 −5 −5 5
−2 3
9. If A=[ ], then |A-1| =?
1 1
A) -5 B) -1/5 C) 1/25 D) 25

3 −1
10. If |A|=3 and A-1=[ ], then adj A=?
−5/3 2/3

9 3 9 −3 −9 3 9 −3
𝐴) [ ] B) [ ] C) [ ] D) [ ]
−5 −2 −5 2 5 −2 5 −2

ANSWERS / SOLUTIONS :

1. (B) Hint. 2 cos 𝛼 = 1

0 1
2. (D) Hint. A = [ ]
1 0

Mathematics Page 47 of 260 XII


3. (D)

Hint As matrix of order 2X3 contains 6 elements. Each element can be


selected in 2 ways

(it can be either 1 or 0)

Hence ,all the six entries can be chosen in 26 = 64 ways.

So, required number of matrices is 64.

4. (A) Hint. Use A2= A , AI = A.

5. (A) Hint. Matrices of same order can be added .

6. (B)

Hint. put value of A and I in L.H.S. and then compare with zero matrix in
R.H.S.

7. (B)

8. (A)

Hint Add two given equations and get A then put value of A in equation
first to get B

9. (B)
1
10. (B) Hint A-1 = |𝐴| (adj A

CASE BASED/ SITUATION BASED


CASE STUDY 1:
A manufacture produces three stationery products Pencil, Eraser and
Sharpener which he sells in two markets. Annual sales are indicated below

Mathematics Page 48 of 260 XII


Market Product in Numbers

Pencil Eraser Sharpener

A 10,000 2,000 18,000

B 6,000 20,000 8,000

If the unit Sale price of Pencil, Eraser and Sharpener are Rs. 2.50, Rs. 1.50
and Rs. 1.00 respectively, and unit cost of the above three commodities are
Rs. 2.00, Rs. 1.00 and Rs. 0.50 respectively, then

Based on the above information answer the following:

1. Total revenue of market A

a. Rs. 64,000
b. Rs. 60,400
c. Rs. 46,000
d. Rs. 40600
2. Total revenue of market B

Mathematics Page 49 of 260 XII


a. Rs. 35,000

b. Rs. 53,000

c. Rs. 50,300

d. Rs. 30,500

3. Cost incurred in market A

a. Rs. 13,000

b. Rs.30,100

c. Rs. 10,300

d. Rs. 31,000

4. Profit in market A and B respectively are

a. (Rs. 15,000, Rs. 17,000)

b. (Rs. 17,000, Rs. 15,000)

c. (Rs. 51,000, Rs. 71,000)

d. ( Rs. 10,000, Rs. 20,000)

5. Gross profit in both market

a. Rs.23,000

b. Rs. 20,300

c. Rs. 32,000

d. Rs. 30,200

ANSWERS

Mathematics Page 50 of 260 XII


1. Rs. 46,000

2. Rs. 53,000

3. RS.31,000

4. (Rs.15, 000, Rs.17, 000)

5. Rs. 32,000

CASE STUDY 2:

Three schools DPS, CVC and KVS decided to organize a fair for collecting
money for helping the flood victims. They sold handmade fans, mats and
plates from recycled material at a cost of Rs. 25, Rs.100 and Rs. 50 each
respectively. The numbers of articles sold are given as

School/ Article DPS CVC KVS

Home Made Fans 40 25 35

Mats 50 40 50

Plates 20 30 40

Mathematics Page 51 of 260 XII


Based on the information given above, answer the following questions:

1. What is the total money (in Rupees) collected by the school DPS?

a. 700

b. 7,000

c. 6;125

d. 7875

2. What is the total amount of money (in Rs.) collected by schools CVC and
KVS?

a. 14,000

b. 15,725

c. 21,000

d. 13,125

3. What is the total amount of money collected by all three schools DPS, CVC
and KVS?

a. Rs. 15,775

b. Rs. 14,000

c. Rs. 21,000

d. Rs. 17,125

4. If the number of handmade fans and plates are interchanged for all the
schools, then what is the total money collected by all schools?

a. Rs. 18,000

Mathematics Page 52 of 260 XII


b. Rs. 6,750

c. Rs. 5,000

d. Rs. 21,250

5. How many articles (in total) are sold by three schools?

a. 230

b. 130

c. 430

d. 330

ANSWERS

1. (b) 7000

2. (a) 14000

3. (c) Rs.21000

4. (d) 21250

5. (d) 330

QUESTIONS CARRYING TWO OR THREE MARKS EACH

1. If A is a square matrix such that A2 = I then find the value of ( A – I )3 + ( A


+ I )3 – 7A .

2. If B is skew symmetric matrix, write whether the matrix (ABA T) is


Symmetric or skew symmetric.

3. Construct a 3 × 2 matrix whose elements are given by a ij = |𝑖 − 3𝑗|

Mathematics Page 53 of 260 XII


4. For the following matrices A and B, verify that: (AB)T = BTAT,

1
where A = [−4] , B = [−1 2 1]
3
3 −1
2 1 4
5. If A = [ ] and B = [2 2 ] , find AB and BA.
4 1 5
1 3
1 2 3 2 3 1
6. If A = [ ] , B= [ ] , then find (i) A –2 B. (ii) 5A + 3B
3 1 3 1 0 2

7. Give example of matrices A & B such that AB = O, but BA ≠ O, where O is a


zero matrix and A, B are both non zero matrices.

2 0 1
8. If A = [2 1 3], then find the value of A2 – 3A + 2I
1 −1 0

9. Find the value of ( y-x ) from the following equation:

𝑥 5 3 −4 7 6
2[ ]+[ ]=[ ].
7 𝑦−3 1 2 15 14
6 0 18 9
10. Find the matrix P satisfying the matrix equation: P[ ]=[ ]
2 3 34 15
1 2 0 0
11. For what value of x : [1 2 1][2 0 1] [2] = O?
1 0 2 𝑥

 3  2  4
12. Express A =  3  2  5 as the sum of a symmetric and a skew-
 1 1 2 
symmetric matrix.

SOLUTIONS / ANSWERS

1. Ans: A. Hint :- A3 = A2 A = IA = A
Mathematics Page 54 of 260 XII
2. Ans: Skew Symmetric. Hint:- show that (ABAT)T = - (ABAT).

2 5
3. Ans: [1 4]
0 3
2 2 7
12 12
5. Ans:- AB = [ ] , BA = [12 4 18]
19 13
14 4 19
−3 −4 1 11 19 18
6. Ans:- (i) A –2 B = [ ] (ii) 5A + 3B = [ ]
1 1 −1 18 5 21
5 −1 2 6 0 3 2 0 0
2
8. Ans:- A – 3A + 2I = [9 −2 5 ] − [6 3 9 ] + [0 2 0] =
0 −1 −2 3 −3 0 0 0 2
1 −1 −1
[−3 −3 −4]
−3 2 −4

9. Ans :- 7

2𝑥 + 3 6 7 6
Hint:- [ ] = [ ] ⇒ x = 2, y = 9 ⇒ y - x =
15 2𝑦 − 4 15 14
9 - 2 = 7.

2 3
10. Ans:- P = [ ]
4 5
a b]
Hint:- Take P = [ to make matrix multiplication possible then
c d
multiply L.H.S. and compare it with R.H.S. ⇒ a = 2, b = 3, c = 4, d = 5 i.e. P =
2 3
[ ]
4 5
11. Ans:- x = -1.

0
Hint:- [6 2 4] 2] = O
[ ⇒ 4 + 4x = 0 ⇒ x = -1.
𝑥

Mathematics Page 55 of 260 XII


12. Ans:-

 3  2  4  3 3  1
  
Sol. Let A   3  2  5  A   2  2 1 
 1 1 2   4  5 2 
1 1
Let A  (A  A)  (A  A)  P  Q
2 2
 6 1  5  3 1 / 2  5 / 2
P  (A  A)   1  4  4   1 / 2  2  2 
1 1  
2 2
 5  4 4   5 / 2  2 2 
 3 1 / 2  5 / 2
P   1 / 2  2  2   P  P is a symmetric matrix .

 5 / 2  2 2 
0  5  3  0  5 / 2  3 / 2 
Q  (A  A)  5 0  6  5 / 2
1
 3 
1
0
2 2
3 6 0  3 / 2 2 0 
 0 5 / 2 3 / 2

Q   5 / 2 0 3   Q  Q is a skew symmetric matrix .
 3 / 2  3 0 
 3 1 / 2  5 / 2  0  5 / 2  3 / 2  3  2  4
P  Q   1 / 2  2  2   5 / 2
 0  3    3  2  5  A
 5 / 2  2 2  3 / 2 2 0   1 1 2 

HOT QUESTIONS

1. If A and B are symmetric matrices of same order and X = AB + BA and Y =


AB – BA, then (XY)T is equal to

(A) XY (B) YX (C) -YX (D) None of these

Mathematics Page 56 of 260 XII


1 −1 1 −1
2. If A = [ ] and B = [ ] , then A8 equals
−1 1 −1 1
(A) 4B (B) 128B (C) -128B (D) -64B

3. If A is a skew-symmetric matrix of order 3, then the matrix A4 is

(A) symmetric (B) skew-symmetric (C) diagonal (D) None of these

4. Let A and B be two 2x2 matrices. Consider the statements

(i) AB = O ⇒ A = O or B = O

(ii) AB = I2 ⇒ A = B -1

(iii) (A + B)2 = A2 + 2AB + B2

Then

(A) (i) is false, (ii) and (iii) are true

(B) (i) and (iii) are false, (ii) is true

(C) (i) and (ii) are false, (iii) is true

(D) (ii) and (iii) are false, (i) is true

Answers:

1. Ans. (C) Hint:- (XY)T = YT XT

= (AB - BA)T (AB +BA)T

= (BTAT - ATBT) (BTAT + ATBT)

= (BA - AB) (BA + AB)

= - (AB - BA) (AB + BA) = - YX

2. Ans. (B) Hint:- find A2 = AA ,then A4 = A2 A2 , then A8 = A4 A4


Mathematics Page 57 of 260 XII
3. Ans. (C) Hint:- (AB)T = BT AT and AT = - A

4. Ans. (B) {∵ matrix multiplication is not commutative, definition of


identity

JEE mains / Advanced pattern


2 −1 −1
√3𝑖−1
1. Let A = [1 0 −1] and B = A - I. If 𝜔 = 2 , then the number of
1 −1 0
elements in the set { n ∈ {1,2,3,….,100}: A n + (𝜔B)n = A + B} is equal
to………………..

(i) 15 (ii) 17 (iii) 50 (iv) 80

1 0 𝑎
2. If A = [ 1 1 0] ,where a ∈ N from 1 to 50 and ∑50
𝑎=1| 𝑎𝑑𝑗 𝐴| = 100 K,
−1 0 1
then the value of K is…………..
1723 1717 1823 1821
(i) (ii) (iii) (iv)
2 2 4 4

ANS. 1. (ii)

ANS. 2. (iv)

Solutions:

2 −1 −1
1. A = [1 0 −1] ⇒ A2 = A ⇒ An = A ∀ n ∈ {1,2,3,….,100}
1 −1 0
1 −1 −1
Now B = A - I = [1 −1 −1] ⇒ B2 = - B ⇒ B3 = B ⇒ B5 = B ⇒ B99 = B
1 −1 −1

So Bn = B ∀ n ∈ {1,3,5…………..99}
Mathematics Page 58 of 260 XII
Also 𝜔 3k = 1,

So, required number = common of {1,3,5…………..99}and {3,6,9…………..99} =


17

2. ∑50
𝑎=1| 𝑎𝑑𝑗 𝐴| = 100 K {∵ | 𝑎𝑑𝑗 𝐴| = |𝐴|n-1 =
|𝐴|2 = (a+1)2

⇒ ∑50 2
𝑎=1(𝑎 + 1) = 100 K

⇒ 22 + 32 +42 +……512 = 100 K

⇒ (12 + 22 + 32 +42 +……512) - 12 = 100 K


1821
⇒ K= 4

Mathematics Page 59 of 260 XII


Chapter - 4
Determinants
* Determinant :
To every square matrix A=[aij] we can assign a unique number called
determinant of a matrix.
𝑎11 𝑎12
* Determinant of a matrix of order 2x2, Let 𝐴 = [𝑎
21 𝑎22 ] is given by |𝐴| =
𝑎11 𝑎12
|𝑎 𝑎22 | = 𝑎11 𝑎22 − 𝑎12 𝑎21
21
𝑎11 𝑎12 𝑎13
* Determinant of a matrix of order 3x3, Let 𝐴 = [𝑎21 𝑎22 𝑎23 ] be a
𝑎31 𝑎32 𝑎33
𝑎11 𝑎12 𝑎13
square matrix of order 3 then |𝐴| = |𝑎21 𝑎22 𝑎23 | and its value is
𝑎31 𝑎32 𝑎33
given by expanding it along first row
𝑎22 𝑎23 𝑎21 𝑎23 𝑎21 𝑎22
Det A = (-1)1+1a11|𝑎 𝑎 | + (-1)1+2a12|𝑎 1+3
𝑎33 | + (-1) a13|𝑎31 𝑎32 |
32 33 31

3 −2 5
2 −1 1 −1
For Example 1 2 −1| = (-1)1+1 3 |
| | + (-1)1+2 (-2) | | + (-
4 7 0 7
0 4 7
1 2
1)1+3 5 | |
0 4
= 3(14+4) +2(7-0) +5(4-0) = 54+14+20 = 88
* 𝐴𝐼 = Transpose of 𝐴
* If we interchange any two rows (columns) then sign of determinant
changes.
* If any two rows (or Columns) are identical or proportional, then value of
determinant is zero.
* If we multiply each element of a row or a column of a determinant by
constant K, then value of determinant is multiplied by K.
Mathematics Page 60 of 260 XII
* Multiplying a determinant by K means multiply elements of only one row
(or columns) by K.
* If 𝐴 = [𝑎𝑖𝑗 ]3X3 then |𝐾𝐴| = 𝐾 3 |𝐴|
* If elements of a row or a column in a determinant can be expressed as
sum of two or more
elements, then the given determinant can be expressed as sum of two or
more determinants.

* Area of a triangle with vertices (𝑥, 𝑦), (𝑥2 , 𝑦2 ) and (𝑥3 , 𝑦3 ) is given by ∆=
𝑥1 𝑦1 1
1
|𝑥2 𝑦2 1|
2
𝑥3 𝑦3 1
* Cofactor of 𝑎𝑖𝑗 is given by 𝐴𝑖𝑗 = (−1)𝑖+𝑗 𝑀𝑖𝑗 where 𝑀𝑖𝑗 is the minor of an
element.
* Let A=[aij] be a square matrix of order ≥ 2, then the adjoint of A is the
transpose of the matrix [Aij] where Aij is the cofactor of the element aij in
|A|. It is denoted by adj.A
* If n is the order of the matrix then |adj. A| = |A|n-1
* If A & B are square matrices of same order, then |AB| = |A| |B|

* 𝐴(𝑎𝑑𝑗 𝐴) = (𝑎𝑑𝑗 𝐴)𝐴 = |𝐴|𝐼 where 𝐴 is square matrix of order 𝑛.


* A square matrix 𝐴 is said to be singular or non-singular according as |𝐴| =
0 or |𝐴| ≠ 0
* 𝐴𝐵 = 𝐵𝐴 = 𝐼 where 𝐵 is a square matrix, then 𝐵 is called inverse of 𝐴.
* A square matrix 𝐴 has inverse if and only if 𝐴 is non singular.

Mathematics Page 61 of 260 XII


𝑎𝑑𝑗.𝐴
A-1 = |𝐴|

*Solving equations by matrix method

If 𝑎1 𝑥 + 𝑏1 𝑦 + 𝑐1 𝑧 = 𝑑1
𝑎2 𝑥 + 𝑏2 𝑦 + 𝑐2 𝑧 = 𝑑2
𝑎3 𝑥 + 𝑏3 𝑦 + 𝑐3 𝑧 = 𝑑3 ;
Then these equations can be written as 𝐴𝑋 = 𝐵, where
𝑎1 𝑏1 𝑐1 𝑥 𝑑1
𝐴 = [𝑎 2 𝑏2 𝑐2 ] 𝑋 = [𝑦 ] 𝐵 = [𝑑2 ]
𝑎3 𝑏3 𝑐3 𝑧 𝑑3
𝐴𝑋 = 𝐵
𝑋 = 𝐴−1 𝐵
𝐴𝑑𝑗 𝐴
= × 𝐵 Provided |𝐴| ≠ 0
|𝐴|

IMPORTANT QUESTIONS FOR CBSE EXAMINATIONS :

1  1 1 
Q1. . Given A= 2 1  3 Find A-1 and hence solve the system of
1 1 1 
equations:

x + 2y + z = 4; -x + y + z = 0 ; x-3y + z = 2

4 2 2 4 2 2
Solution: A =10 , Adj A=  5 0 5  A =  5 0 5 
  1 1 
10
 1  2 3  1  2 3

AT X =B  
 X= AT
1
  9 2
B  X= A1 B  x= , y = , z =
T

5 5
7
5
Mathematics Page 62 of 260 XII
1  1 0   2 2  4
   
Find BA where A  2 3 4 , B    4 2  4  . Using the result
Q2. 0 1 2 2 1 5 
   

solve: x – y = 3 ; 2x + 3y + 4z = 17 ; y + 2z = 7

6 0 0
 
Solution: BA =  0 6 0  = 6 I.
0 0 6 

1
Post multiplying by A-1 we get B = 6 A-1. Hence A-1 = B.
6
1  1 0   x   3 
    
 2 3 4   y   17 
0 1 2  z  7 
    
A X = B.
Pre-multiply by A-1, we get X = A-1 B.
x  2 2  4 3 
  1   
 y   4 2  4  17 
=

z  6  2 1 5  7 
    
12  2 
1     . Hence x= 2, y = -1, z = 4
  6   1
= =
6   4 
 24   

Q3. Using matrix method solve the following system of equations for x, y
and z.
2x-3y+3z=10, x+y+z = 10, 3x-y+2z= 13
Solution: Given equations can be in written is matrix form AX=B

 2  3 3  x 10  2  3 3  x 10
1 1 1   y  = 10 where A  1 1 1 X =  y  B  10
           
3  1 2  z  13 3  1 2  z  13

Mathematics Page 63 of 260 XII


2 3 3 3 3  6
A = 1 1 1 = -9  0 ,SO A 1 exists & Adj A =  1  5 1   A 1 =
 
3 1 2  4  7 5 
adj ( A)
X = 2, Y = 3, Z = 5 .
A

2 −3 5
Q4. If 𝐴 = [3 2 −4], find 𝐴−1 . Using 𝐴−1 solve the system of
1 1 −2
equations 2𝑥 − 3𝑦 + 5𝑧 = 11, 3𝑥 + 2𝑦 − 4𝑧 = −5, 𝑥 + 𝑦 − 2𝑧 = −3.

Solution: follow same steps as Q1.

2 −3
Q5. Show that A=[ ] satisfies the equation x2 – 6x + 17 = 0. Hence find
3 4
A-1
2 −3
Solution: We have, A= [ ]
3 4
2 −3 2 −3
A2=A.A=[ ][ ] = [−5 −18]
3 4 3 4 18 7
2 −3 12 −18 17 0
6A=6[ ]=[ ] and 17 I=17[ ]
3 4 18 24 0 17
0 0
𝐴2 − 6𝐴 + 17𝐼=[ ]
0 0
Hence matrix A satisfies the equation, x2 – 6x +17=0.

A2 -6A = -17I2
A-1 = ( 1/17 )(6I2 - A)
6 0 2 −3 4 3
=(1/17) {[ ]−[ ]} = 1/17[ ]
0 6 3 4 −3 2

CASE BASED QUESTIONS

Q1. Two schools A and B wants to award their selected students on the
values of sincerity, truthfulness and helpfulness. The school A wants to
award Rs. 𝑥 each, Rs. 𝑦 each and Rs. 𝑧each for three respective values to 3, 2
and 1 students respectively with a total award money of Rs. 2200. School B

Mathematics Page 64 of 260 XII


wants to spend Rs. 3100 to award its 4, 1 and 3 students on the respective
values(by giving the same award money to the three values as before). If the
total amount for one prize on each value is Rs. 1200.Based on the above
information answer the following questions.

(i) Value of the determinant formed by the coefficients.


(ii) Find A-1
(iii) Values of x,y and z.

Solution: 3x+2y+z=2200 ; 4x+y+3z=3100 ; x+y+z= 1200

|A| = 5

3 2 1 2200 𝑥
A=[4 1 3 ; B=[3100 ; X=[𝑦]
] ]
1 1 1 1200 𝑧

2 1 −5
A-1= 1/5. [ 1 −2 5 ]
−3 1 5
So x=300,y=400,z=500

Q 2. Area of a triangle whose vertices are (x1,y1),(x2' y2) and (x3, y3) is given by
the determinant
1 𝑥1 𝑦1 1
∆= |𝑥2 𝑦2 1|
2 𝑥 𝑦 1
3 3
Since, area is a positive quantity, so we always take the absolute value of the
determinant Δ. Also, the area of the triangle formed by three collinear
points is zero.
Based on the above information, answer the following questions
(i) Find the area of the triangle whose vertices are (-2, 6), (3, -6) and (1, 5).

a) 30 sq units b) 35 sq units c) 40 sq. units d) 15.5 sq. units


(ii) If the points (2, -3), (k, -1) and (0, 4) are collinear, then find the value of
4k
Mathematics Page 65 of 260 XII
140 40
a) 4 b) c) 8 d)
7 7

(iii) If the area of a triangle ABC, with vertices A(1, 3), B(0, 0) and C(k, 0) is 3
sq. units, then a value of k is

(a) 2 (b) 3 (c) 4 (d) 5


(iv) Using determinants, find the equation of the line joining the points A(1,
2) and B(3, 6).

a) y=2x b) x=3y c) y = x d) 4x-y=5

1. Solution: (i) (d) : Let be the area of the triangle then,


1 −2 6 1
𝐴𝑟𝑒𝑎 = | 3 −6 1|
2
1 5 1
=1/2|−2(−6−5)−6(3−1)+1(15+6)|=1/2|−2(−6−5)−6(3−1)+1(15+6)| [Expanding
along R1]
⇒Δ=1/2|43−12|=15.5sq. units ⇒Δ=1/2|43−12|=15.5sq. units
(ii) (d) : The given points are collinear.
∴ Area of triangle = 0
On expnding k=40/7
(iii) (a) : Area of ΔABC=3 sq. units ΔABC=3 sq. units [Given]
1 3 1
1
|0 0 1| = ± 3 on solving k=±2.
2
𝑘 0 1
(iv) (a) : Let Q(x, y) be any point on the line joining
A(1, 2) and B(3, 6). Then, area of ΔABQ=0ΔABQ=0
on solving we get 2x=y

Q3. Gaurav purchased 5 pens, 3 bags and 1 instrument box and pays Rs. 16.
From the same shop, Dheeraj purchased 2 pens, 1 bag and 3 instrument
boxes and pays Rs.19, while Ankur purchased 1 pen, 2 bags and 4
instrument boxes and pays Rs.25.

Mathematics Page 66 of 260 XII


Using the concept of matrices and determinants, answer the following
questions.
(i) The cost of one pen is
(a) Rs.2 (b) Rs.5 (c) Rs.1 (d) Rs.3
(ii) What is the cost of one pen and one bag?

(a) Rs.3 (b) Rs.5 (c) Rs.7 (d) Rs.8


(iii) What is the cost of one pen and one instrument box?

(a) Rs.7 (b) Rs.6 (c) Rs.8 (d) Rs.9


(iv) Which of the following is correct?

(b) Determinant is a number associated


(a) Determinant is a square matrix.
to a matrix
(c) Determinant is a number associated to a
(d) All of the above
square matrix
Solution:
Let the cost of 1 pen =Rs. x, the cost of 1 bag =Rs.y, and the cost of 1
instrument box =Rs. z
According to the question, we have
5x + 3y + z = 16, 2x + y + 3z = 19, x + 2y + 4z = 25
This system of equation can be written as AX = B
X=A-1B
On solving these equations we get x = 1,y= 2, z = 5
Hence, cost of one pen, one bag and an instrument box is Rs. 1,Rs. 2 andRs.
5 respectively.
(i) (c) : Cost of one pen isRs. 1.
(ii) (a) : Cost of one pen. and one bag =Rs. (1 + 2) =Rs. 3
(iii) (b) : Cost of one pen and one instrument box =Rs. (1 + 5) =Rs. 6
Mathematics Page 67 of 260 XII
(iv) (c) : According to the definition of determinant, determinartt is a
number associated to a square matrix.

(Multiple Choice Questions)


Each question carries 1 mark
Q1. . If A is a square matrix of order 3, |𝐴′| = −3, then |𝐴𝐴′| =

(a) 9 (b) -9 (c) 3 (d) -3

Solution: Using |AB| = |A| |B| and |A|= |A’|


|𝐴𝐴′| =|A| |A’| = (-3)(-3) = 9 (a)

2 4 2𝑥 4
Q2. If | | = | |,ℎ𝑒𝑛 𝑡ℎ𝑒 𝑝𝑜𝑠𝑠𝑖𝑏𝑙𝑒 value(s) of ‘x’ is/are
5 1 6 𝑥
(a) 3 (b) √3 (c) -√3 (d) √3, −√3
Solution: 2 − 20 = 2𝑥2 − 24 ⟹ 2𝑥2 = 6 ⟹ 𝑥2 = 3 ⟹ 𝑥 = ±√3 (d)
Q3. If A is a square matrix of order 3 and |A| = 5, then |𝑎𝑑𝑗𝐴| =
1
(a) 5 (b) 25 (c) 125 (d) 5
Solution: |𝑎𝑑𝑗𝐴| = |𝐴|n-1⟹ |𝑎𝑑𝑗𝐴| = 25

Q4. If A is a 3x3 matrix such that |A|= 8 , then |3A| equals

(a) 8 (b) 24 (c) 72 (d) 216


Solution: |𝐾𝐴| = 𝐾 3 |𝐴|
|3A|= 33|A| = 27x8=216
−2 0 0
Q5. If A is a square matrix order 3 such that A(adj.A) = [ 0 −2 0 ] then
0 0 −2
|adj.A| is equal to
(a) -2 (b) -4 (c) 4 (d) -8
Solution: 𝐴(𝑎𝑑𝑗 𝐴) = (𝑎𝑑𝑗 𝐴)𝐴 = |𝐴|𝐼
|A.adj.A|= -8
|A| |adj.A| = -8
|A| |A|2 =-8

Mathematics Page 68 of 260 XII


|A|= -2
Now |adj.A| = |A|2 =(-2)2 = 4 (c)

Q6. If A ,B,C are non singular matrices of same order then (AB -1C)-1 =
(a) CBA-1 (b) C-1B-1A-1 (c) C-1 B A-1 (d) C-1BA
Solutions: (AB-1C)-1 = C-1 (B-1)-1A-1
= C-1 B A-1 (c)

Q7. If A2-A+I =0 then A-1 is equal to


(a) A+I (b) A-I (c) A+2I (d) I-A
Solution: pre multiplying by A-1 both sides
A-I+A-1=0
A-1= I - A (d)

Q8. Value of k, for which


𝑘 8]
A= [
4 2𝑘
is a singular matrix is
(a) 4 (b) -4 (c) ±4 (d) 0
Solution:
Given that matrix A is singular.
Thus, the determinant of A is 0.
So,
|A|=0
k(2k) – 8(4) = 0
2k2 – 32 = 0
2k2 = 32
k2 = 16
k = ±4 ( c )

Q9. The area of a triangle with vertices (–3, 0), (3, 0) and (0, k) is 9 sq. units.
The value of k will be
(a) 9 (b) 3 (c) -9 (d) 6
Mathematics Page 69 of 260 XII
Solution:
The formula of area of the triangle with vertices (x 1, y1), (x2, y2), (x3, y3) is
given by:
𝑥1 𝑦1 1
1
∆= 2 |𝑥2 𝑦2 1|Thus, the area of a triangle with vertices (–3, 0), (3, 0) and
𝑥3 𝑦3 1
(0, k) is:
Δ=9
{given}
⇒ -3(0 – k) – 0+ 1(3k – 0) = 18
⇒ 3k + 3k = 18
⇒ 6k = 18
⇒ k = 3 (b)
Q10. Which of the following is correct?
(a) Determinant is a square matrix.
(b) Determinant is a number associated with a matrix.
(c) Determinant is a number associated with a square matrix.
(d) None of these
Correct option: (c) Determinant is a number associated with a square matrix.

ASSERTION-REASON BASED QUESTIONS


In the following questions, a statement of assertion (A) is followed by a
statement of Reason (R). Choose the correct answer out of the following
choices.
Q11.

Assertion [A]: Minor of element 6 in the matrix is 3.

Mathematics Page 70 of 260 XII


Reason [R]: Minor of an element aij of a matrix is the determinant obtained
by deleting its ith row.
a) Both A and R are individually true and R is the correct explanation of A.
b) Both A and R are individually true and R is not the correct explanation of
A.
c) 'A' is true but 'R' is false
d) Both A and R are false.

Minor of element
∴Given Assertion [A] is false Also we know that minor of an element aij of a
matrix is the determinant obtained by deleting its ith row and jth column. ∴
Given Reason (R) is also false ∴ Both Assertion [A] and Reason (R) are false
Hence option (e) is the correct Answer.

Q12. Assertion [A]: Value of k for which area of the triangle with vertices (1,
1), (0, 2), (k, 0) is 3 sq. units are 4 and 8.

Reason [R]: Area of the triangle with vertices (x1,y1),(x2,y2)(x3,y3) is

A) Both A and R are individually true and R is the correct explanation of A.


B) Both A and R are individually true and R is not the correct explanation of
A.
C) 'A' is true but 'R' is false
D) 'A' is false but 'R' is true

Mathematics Page 71 of 260 XII


 Solution: Given: Area of

triangle,
⇒(2−0)−(0−k)+(0−2k)=±
⇒2+k−2k=±
⇒−k=±6−2
⇒−k=6−2;−k=−6−2
⇒−k=4;−k=−8
⇒k=−4;k=8

∴ Assertion [A] is false Also Reason (R) is true Hence option [D] is correct
answer.

Mathematics Page 72 of 260 XII


Chapter – 5

Continuity and Differentiability

Continuity at a Point: A function f(x) is said to be continuous at a point


x = a if the function is defined at a and the value of the function at x=a is
equal to the limit of the function at a
ie
Left hand limit of f(x) at (x = a) = Right hand limit of f(x) at (x = a) = Value
of f(x) at (x = a)
i.e. if at x = a, LHL = RHL = f(a)
where, LHL = lim− 𝑓(𝑥)and RHL = lim+ 𝑓(𝑥)
𝑥→𝑎 𝑥→𝑎

EXAMPLE

f(x)=1/(x−1)
At x=1 we have:

1/(1−1) = 1/0 = undefined

So there is a "discontinuity" at x=1

So f(x) = 1/(x−1) over all Real Numbers is NOT continuous

Let's change the domain to x>1

So f(x) is continuous

In other words f(x) does not include the value x=1, so it is continuous.

Note: To evaluate LHL of a function f(x) at (x = o), put x = a – h and to find


RHL, put x = a + h.

Continuity in an Interval: A function y = f(x) is said to be continuous in


an interval (a, b), where a < b if and only if f(x) is continuous at every
point in that interval.

Mathematics Page 73 of 260 XII


● Every identity function is continuous.
● Every constant function is continuous.
● Every polynomial function is continuous.
● Every rational function is continuous.
● All trigonometric functions are continuous in their domain.

● Standard Results of Limits

Algebra of Continuous Functions :


Suppose f and g are two real functions, continuous at real number c.
Then,
● f + g is continuous at x = c.
● f – g is continuous at x = c.
● f.g is continuous at x = c.
● cf is continuous, where c is any constant.
● (f/g) is continuous at x = c, [provide g(c) ≠ 0]
Suppose f and g are two real valued functions such that (fog) is defined at
c. If g is continuous at c and f is continuous at g (c), then (fog) is
continuous at c.
If f is continuous, then |f| is also continuous.

Differentiability: A function f(x) is said to be differentiable at a point x =


a, if
Left hand derivative at (x = a) = Right hand derivative at (x = a)
i.e. LHD at (x = a) = RHD (at x = a),

Mathematics Page 74 of 260 XII


Note: Every differentiable function is continuous but every continuous
function is need not to be differentiable.
Differentiation: The process of finding a derivative of a function is
called differentiation.

Rules of Differentiation :
Sum and Difference Rule: Let y = f(x) ± g(x).Then, by using sum and
difference rule, its derivative is written as

Product Rule: Let y = f(x) g(x). Then, by using product rule, it’s
derivative is written as

EXAMPLE :
Suppose we want to differentiate y = x 2 cos 3x.
We identify f(x)= u as x 2 and g(x)=v as cos 3x. u = x 2 ,v = cos 3x
We now write down the derivatives of each of these functions. du/ dx =
2x and dv/ dx = −3 sin 3x We now put all these results into the given
formula: dy/ dx = u dv/ dx + v du/ dx = x 2 × (−3 sin 3x) + cos 3x × 2x
dy/ dx = x(−3x sin 3x + 2 cos 3x)

Quotient Rule: Let y = f(x)/g(x); g(x) ≠ 0, then by using quotient rule, it’s
derivative is written as

Mathematics Page 75 of 260 XII


EXAMPLE:
Suppose we want to differentiate y = cosx / x 2 .
We have identified u as cos x and v as x 2 . So u = cos x and v = x 2
We now write down the derivatives of these two functions. du/ dx = − sin
x & dv /dx = 2x
We now put all these results into the given formula: dy /dx =( v du/ dx −
u dv/ dx) /v 2
So dy/ dx ={ x 2 · (− sin x) − cos x · 2x}/ (x 2 ) 2
dy/ dx = −x(x sin x + 2 cos x) /x 4 = −(x sin x + 2 cos x)/ x 3

Chain Rule: Let y = f(u) and u = f(x), then by using chain rule, we may
write

EXAMPLE :
Suppose we want to differentiate y = cos x 2 .
Let u = x 2 so that y = cos u.
It follows immediately that du /dx = 2x and dy /du = − sin u
The chain rule says dy/ dx = (dy/ du) × (du/ dx )
so dy /dx = − sin u × 2x = −2x sin x2
Logarithmic Differentiation: Let y = [f(x)]g(x) ..(i)
So by taking log (to base e) we can write Eq. (i) as log y = g(x).log f(x).
Then, by using chain rule

EXAMPLE:
y=xx, x>0.
First we take logarithms of the left and right side of the equation:
Mathematics Page 76 of 260 XII
Log y= log xx,
⇒logy=x log x.
Now we differentiate both sides meaning that y is a function of x:
(logy)′ = (xlogx)′,
⇒(1/y)⋅y′=x′logx+x(logx)′
,⇒y′ =y(1⋅logx+x⋅1/x)
,⇒y′=y(logx+1),
⇒y′=y(logx+1),
⇒y′=xx(logx+1),where x>0.

Differentiation of Functions in Parametric Form: A relation


expressed between two variables x and y in the form x = f(t), y = g(t) is
said to be parametric form with t as a parameter, where

Whenever dx/dt≠0
Note: dy/dx is expressed in terms of parameter only without directly
involving the main variables x and y.

EXAMPLE
x=2t+1, y=4t−3.
dx/dt = (2t+1)’ =2,
dy/dt = (4t−3)′ =4.
Consequently,
dy/dx={dy/dt}/{dx/dt}=4/2
=2
Second order Derivative: It is the derivative of the first order
derivative.

Mathematics Page 77 of 260 XII


EXAMPLE
Let y =3x4−2x3+4x2−5x+1.
Take the first derivative using the power rule and the basic
differentiation rules:
y′=12x3−6x2+8x-5.
The second derivative is expressed in the form
y′′=36x2−12x+8

Mathematics Page 78 of 260 XII


Some Standard Derivatives

Some useful substitutions for finding Derivatives Expression :

Mathematics Page 79 of 260 XII


1 The function f(x)=[x],where [x] is the greatest integer function, is
continuous at

(a) 4 (b) -2 (c) 1 (d) 1.5

2 f(x)=|x|+|x-1|is
(a) Continuous at x=0 as well as at x=1
(b) continuous at x=1 but not at x=0
(c) continuous at x=0 but not at x=1
(d) Discontinuous at x=0 as well as at x=1
3 If xy . yx = 16, then the value of dy/dx at (2, 2) is
(a) -1 (b) 0 (c) 1 (d) none of these

4 If y = (tan x)sin x, then dy/dx is equal to


(a) sec x + cos x (b) sec x + log tan x
(c) (tan x) sin x (d) None of these

5 For what valve of A function f is defined by 𝑓(𝑥) =


1−𝑐𝑜𝑠𝑥
{ } , 𝑤ℎ𝑒𝑛 𝑥 ≠ 0
{ 𝑥2 } is continuous at x=0
𝐴, 𝑤ℎ𝑒𝑛 𝑥 = 0

Mathematics Page 80 of 260 XII


1 2 1
(a) 2 (b) (c) (d)
2 3 3

6 A function f(x) is said to be continuous everywhere if

(a) It is continuous at x=0 (b) It is differentiable at x=0

(c) It is continuous at 2 points (d) It is continuous at


every real number
𝑑𝑦
7 IF y=log 𝑠𝑖𝑛𝑥 𝑠𝑖𝑛𝑥 then 𝑑𝑥 𝑖𝑠

(a) Cos x (b) Log sinx (c) 0 (d) 1


𝑑𝑦
8. If y= 36log6 𝑥 then 𝑑𝑥 𝑖𝑠

(a) 2x (b) 6x (c) 36x (d) Log6x


𝑑𝑦
9 If y =√𝑠𝑖𝑛𝑥 + 𝑦, then is equals to:
𝑑𝑥

𝑐𝑜𝑠 𝑥 𝑐𝑜𝑠 𝑥 𝑠𝑖𝑛 𝑥 𝑠𝑖𝑛 𝑥


(a) 2𝑦−1 (b) (c) (d)
1−2𝑦 1−2𝑦 2𝑦−1

𝑠𝑖𝑛 𝑥+𝑐𝑜𝑠 𝑥 𝑑𝑦
10 y= tan−1 (𝑐𝑜𝑠 𝑥−𝑠𝑖𝑛 𝑥 ) then is equals to:
𝑑𝑥

(a) 0 (b) 1/2 (c) 1 (d) 2

11 The number of discontinuous functions f(x) on [-2, 2] satisfying x2


+ y2 = 4 is

(a) 0 (b) 1 (c) 2 (d) >2

12 For what value of x, f(x) = |2x – 7| is not derivable?

(a) 0 (b) 1 (c) 2/7 (d) 7/2


𝑑𝑦
13 What is the value of 𝑑𝑥 if x=𝑡 2 , and y= 𝑡 3

Mathematics Page 81 of 260 XII


2 3
(a) t (b) 𝑡 (c) 0 (d) 2 𝑡
3

3𝜋
14 If 𝑓(𝑥 ) = |𝑐𝑜𝑠 𝑥 |, 𝑓𝑖𝑛𝑑 𝑓 ′ ( 4 )

√3 1 − √3
(a) (b) (c) (d) 𝑛𝑜𝑛𝑒 𝑜𝑓 𝑡ℎ𝑒𝑠𝑒
2 √2 2

15 Derivative of log 7 𝑙𝑜𝑔𝑥 w.r. to x is


1 1 1 1
(a) 𝑙𝑜𝑔7.𝑙𝑜𝑔𝑥 (b) (c) (𝑑)
𝑥 𝑙𝑜𝑔7.𝑙𝑜𝑔𝑥 7.𝑙𝑜𝑔𝑥 𝑥𝑙𝑜𝑔7

𝑑𝑦
16 If x=a(Cos𝜃 + 𝜃𝑆𝑖𝑛𝜃)) and y= a(sin𝜃 − 𝜃𝑐𝑜𝑠𝜃)) then 𝑑𝑥 is equal to

(a) Cot𝜃 (b) tan𝜃 (c) aCot𝜃 (d)


atan𝜃

17. If f(x) = x² sin(1/x) where x ≠ 0, then the value of the function f(x)
at x = 0, so that the function is continuous at x = 0 is
(a) 0 (b) -1 (c) 1 (d) None of these

18. The derivative of ex3 with respect to log x is

(a) ee3 (b) 3x22ex3 (c) 3x3ex3 (d) 3x2ex3+ 3x2

19. If f(x) = cos x, cos 2 x, cos 4 x, cos 8 x, cos 16 x, then the value of
f'(π/4) is
(a) 1 (b) √2 (c) 1√2 (d) 0

20. The function f(x) = e|x| is


(a) continuous everywhere but not differentiable at x = 0
(b) continuous and differentiable everywhere
(c) not continuous at x = 0
(d) None of these

Mathematics Page 82 of 260 XII


ANSWERS CONTENT BASED MCQ

1. (d) 1.5
Hint. Greatest integer function are continuous at all non integral
points
2. (a) Continuous at x=0 as well as at x=1
Hint:- I x-a I function are continuous at x = a .
3. (a)-1
Hint:-Using log
Logxy + log yx =log 16
ylogx + xlog y =log 16
Diff wrt x
y/x +logx dy/dx + (x/y) .dy/dx + log y =0
dy/dx= - (logy +y/x)/(logx +x/y)
putting x=2 ,y=2
dy/dx =-1
4. (d) None of these Hint:-Taking log and solving
dy/dx=(tanx)sinx{cosxlogtanx +sec x }
1 1−𝑐𝑜𝑠𝑥 2𝑠𝑖𝑛 2 𝑥/2
5. (b) Hint:-lim x→0 = lim x→0 =1/2
2 𝑥2 𝑥2
6. (d) It is continuous at every real number
𝑑
7. (c) 0 Hint:-log 𝑒 𝑒 = 1 and 𝑑𝑥 (1) =0
𝑑
8. (a) 2x Hint:-𝑎log𝑎 𝑥 = 𝑥 so y = x2 and 𝑑𝑥 (x2) =2x

𝑐𝑜𝑠 𝑥
9. (a) Hint:- y2 = sinx +y so 2y dy/dx =cosx +dy/dx
2𝑦−1
𝑐𝑜𝑠 𝑥
,so dy/dx = 2𝑦−1
𝑠𝑖𝑛 𝑥+𝑐𝑜𝑠 𝑥 𝜋
10. (c) 1 Hint:-𝑐𝑜𝑠 𝑥−𝑠𝑖𝑛 𝑥 = 𝑡𝑎𝑛(4 + 𝑥 )

11. (a) 0 Hint:-x2 + y2 = 4 is continuous at all points in


[-2,2].
12. (d) 7/2 Hint:-|𝑥 − 𝑎| is not differentiable at x = a
3 3
13. (d) 𝑡 Hint:-dx/dt = 2t , dy/dt = 3t2 so dy/dx = 𝑡
2 2

Mathematics Page 83 of 260 XII


1 𝑑 𝑥
14. (b) Hint:-𝑑𝑥 |𝑥 | = |𝑥|
x≠ 0
√2
1 𝑙𝑜𝑔 𝑥
15. (b) Hint:-log𝑎 𝑥 =
𝑥 𝑙𝑜𝑔7.𝑙𝑜𝑔𝑥 𝑙𝑜𝑔𝑎
16. (b) tan𝜃
17. (a) 0 Hint:-𝑥 2 𝑠𝑖𝑛1/𝑥 = 0 . (some finite value) =0
𝑑𝑦 𝑑𝑦/𝑑𝑥
18. (c) 3x3ex3 Hint:-𝑑𝑢=𝑑𝑢/𝑑𝑥

19. (d)0
Hint:-log y = log g(x)
1 𝑑𝑦 𝑑
so,𝑦 = 𝑑𝑥 𝑔(𝑥 )
𝑑𝑥
𝑑𝑦 𝑑
so 𝑑𝑥 = 𝑦 𝑑𝑥 𝑔(𝑥)
But y=cos x, cos 2 x, cos 4 x, cos 8 x, cos 16 x
And y(π/4)=0 because cot2x at (π/4) =0
So dy/dx =0
20. (a) continuous everywhere but not differentiable at x = 0
Hint:- |x| is not differentiable at x=0

ASSERTION AND REASON BASED QUESTIONS

For below questions two statements are given ,one labelled


ASSERTION and other labelled REASON. Select the correct answer to
these questions from the codes (A),(B),(C) and (D) as given below

(A) Both Assertion and Reason are correct and Reason is the
correct explanation for Assertion
(B) Both Assertion and Reason are correct and Reason is not the
correct explanation for Assertion
(C) Assertion is correct but Reason is incorrect
(D) Assertion is incorrect but Reason is correct

Q1:

Mathematics Page 84 of 260 XII


ANSWER

Correct option is C
(C) Assertion is correct but Reason is incorrect

Explanation

Assertion
f(0)=0
lim x→0 x2 sin(1/x) = (0)2× (finite value)

=0
∴ It is continuous at x=0
Reason:
h(x)=x2 is continuous
but g(x) is not continuous
lim x→0 sin(1/x) = not defined
not continuous.

Q2. Assertion (A) : The function f(x) = I x I is discontinuious at x = 0


Reason (R) : the function f(x) = IxI is non differentable at x = 0
ANSWER
(D) Assertion is incorrect but Reason is correct
EXPALNATION
it is clear from the graph f(x) is continuous at x = 0 but not
differentiable

Mathematics Page 85 of 260 XII


Q3. Assertion (A) : The function f(x) = √x-2 + √2-x is continuious
at x = 2
Reason (R) : the function f(x) is a point function

ANSWER
D) Assertion is incorrect but Reason is correct
EXPLANATION :
√X – 2 is defined for X ≥ 2
AND √2 – x is defined for X ≤ 2
So f(x) is defined only at x=2
So f(2) =0
Hence f(x) is discontinious at x=2
Also f(x) is a point function

Q4.
ASSERTION : If y = 5cosx-3sinx then
d2y/dx2 = y
REASON: If y=f(x) then dy/dx is obtained by differentiating y with
respect to x and d2y/dx2 is obtained by differentiating dy/dx with
respect to x
ANSWER

(D) Assertion is incorrect but Reason is correct

EXPLANATION
Y= 5cosx -3sinx
dy/dx = -5sinx - 3 cosx
d2y/dx2 = -5cosx + 3 sinx = -y

CASE STUDY BASED QUESTIONS :


Q1: A function f(x) is said to be continuous at a point x = c if the function
is defined at c
Mathematics Page 86 of 260 XII
And the value of the function at x=c is equal to the limit of the function at
c
i.e.
lim 𝑥 → 𝑐 f(x)= f(c)
If it is not continuous at c we say f is discontinuous and c is called point
of discontinuity of f .
Based on the above information answer the following questions

(i)The number of point of discontinuity at f(x)= [x] in [3,7] is


(a)4 (b)5 (c)6 (d)8
(ii)Suppose f and g are two real functions continuous at a real number c
then:
(a) f + g is continuous at x = c.
(b) f + g is discontinuous at x = c.
(c) f + g may or may not be continuous at x=c
(d) none of these
(iii) The value of k so that the given function f(x) is continuous at x = 2
f(x) ={𝑘𝑥2 𝑤ℎ𝑒𝑛 𝑥 ≤ 2 3 𝑤ℎ𝑒𝑛 𝑥 ≥ 2 }
(a)1 (b)1/4 (c)3/4
(d)5/4

(iv) The value of k so that the given function f(x) is continuous at x = 5


fx)={𝑘𝑥 + 1 𝑤ℎ𝑒𝑛 𝑥 ≤ 5 3𝑥 − 1 𝑤ℎ𝑒𝑛 𝑥 ≥ 5 }
(a)3/5 (b)1/5 (c)4/5
(d)9/5

(v) If f(x) = IxI is continious and g(x) = sin x is continuous then


(a) sin IxI is continuous
(b) sin IxI is discontinuous
(c) sin IxI may or may not continuous
(d) none of these

ANSWERS ( Q1)
Mathematics Page 87 of 260 XII
(i) (b)5

(ii) (a) f + g is continuous at x = c.

(iii) (c)3/4
(iv) (d), 9/5
(v) (a) sin IxI is continuous

Q2. Let f(x) be a real valued function then its right hand derivative
and left hand derivative is given by

Also a
function f(x) is said to be differentiable at x = a if its RHD and
LHD at x = a exist and equal. For the function
𝑥2 3𝑥 13
f(x) ={|𝑥 − 3| 𝑥 ≥ 1 − + 𝑥 <1}
4 2 4

Answer the following questions :

(i) RHD of f(x) at x=1 is


(a)1 (b)-1 (c)0 (d)2

(ii) LHD of f(x) at x=1 is


(a)1 (b)-1 (c)0 (d)2

(iii) f(x) is non differentiable at x =


(a)1 (b)2 (c)3 (d)4

(iv)The value of f’ (2) is


(a)1 (b)-1 (c)2 (d)4

Mathematics Page 88 of 260 XII


(v) the value of f’(-1) is
(a)1 (b)-1 (c)2 (d)-2

ANSWERS (Q2)
(I) (b)-1
(II) (b)-1
(III) (c)3
(IV) (b)-1
(V) (d)-2

Content based Short answer type questions (2 marks) :

Q1. Determine the value of the constant ‘k’ so that the function

Is continuous at x=0.

Q2:- If y = x|x|, find dy/dx for x < 0.


Answer:
We have, y = x|x|
When, x < 0, then |x| = – x
∴ y = x(- x) = – x2
⇒ dy/dx = – 2x

Mathematics Page 89 of 260 XII


Q3 .Find the value of k, so that the function

is continuous at x = 0.

SOLUTION.

Q 4:-. If y = cos (√3x), then find dy/dx.


Answer:
Given, y = cos (√3x)
Differentiating w.r.t x, we get

Q5:-. Differentiate e√3x, with respect to x.


Answer:

Mathematics Page 90 of 260 XII


Let y = e√3x

Content based Short answer type questions (3 marks) :

Q1 . Write the derivative of sin x with respect to cos x.


Solution:-
Let u = sin x
On differentiating both sides w.r.t. X, we get
du/dx = cos x ……. (i)
Also, let v = cos x
On differentiating both sides w.r.t. x, we get
dv/dx = – sin x ……… (ii)
Now, du/dv=(du/dx)×(dx/dv)=−cosx/sinx [from Eqs. (i) and (ii)]
∴ du/dv = – cot x
𝑑𝑦 𝑥+𝑦
Q2. If log (x2 + y2) = 2 tan-1(y/x) show that 𝑑𝑥 = 𝑥−𝑦
Answer:
log (x2 + y2) = 2 tan-1(y/x)
on differentiating both sides w.r.t. x, we get

Mathematics Page 91 of 260 XII


1+𝑐𝑜𝑠𝑥
Q3.:- Differentiate tan-1 with respect to x.
𝑠𝑖𝑛𝑥
Answer:

Mathematics Page 92 of 260 XII


Long answer type questions (5 marks) :

Q1. If y = a sin x + b cos x, then prove that y2 + (dy/dx)2 = a2 + b2.


Answer: First, we differentiate the given expression with respect to x
and get first derivative of y. Then, put the value of y and first derivative
of y in LHS of given expression and then solve it to get the required RHS.
To prove y2 + (dy/dx)2 = a2 + b2
Given, y = a sin x + b cos x ….. (ii)
On differentiating both sides of Eq. (ii) w.r.t. x,
we get
dy/dx = a cos x – b sin x
Now, Let us take LHS of Eq. (i).
Here, LHS = y2 + (dy/dx)2
On putting the value of y and dy/dx , we get
LHS = (a sin x + b cos x)2 + (a cos x – b sin x)2
= a2 sin2 x + b2 cos2 x + 2ab sin x cos x + a2 cos2 x + b2 sin2 x – 2ab sin x
cos x
= a2 sin2 x + b2 cos2 x + a2 cos2 x + b2 sin2 x
= a2 (sin2 x + cos2 x) + b2 (sin2 x + cos2 x)
= a2 + b2 [∵ sin2 x + cos2 x = 1]
= RHS
Hence proved.

Q 2. If sin y = x cos (a + y), then show that


𝑑𝑦 𝑐𝑜𝑠 2 (a+y))
=
𝑑𝑥 𝑐𝑜𝑠𝑎
𝑑𝑦
Also, show that = cos a, when x = 0.
𝑑𝑥
Answer:

Mathematics Page 93 of 260 XII


Q3. If xy – yx = ab, find dy/dx.
Answer:

Mathematics Page 94 of 260 XII


Questions asked in JEE :
Q1:-

Q.2:-

Q3:-

Mathematics Page 95 of 260 XII


Answers of the questions asked in JEE :
Q1:- D
Q.2:- A
Q.3:- B

Mathematics Page 96 of 260 XII


Chapter - 6

Application of Derivatives
SOME IMPORTANT CONCEPTS

RATE OF CHANGE

Whenever one quantity y varies with another quantity x, satisfying some


𝑑𝑦
rule y = f (x), then 𝑑𝑥 represents the rate of change of y with respect to x and
𝑑𝑦
represents the rate of change of y with respect to x at x= x0.
𝑑𝑥
X=x0

INCREASING AND DECREASING FUNCTIONS

Let I be an open interval contained in the domain of a real valued function f.


Then f is said to be

(i) Increasing on I if x1 < x2 in I ⇒ f (x1) ≤ f (x2) for all x1, x2 ∈ I.


(ii) Strictly increasing on I if x1 < x2 in I ⇒ f (x1) < f (x2) for all x1, x2 ∈ I.
(iii) Decreasing on I if x1 < x2 in I ⇒ f (x1) ≥ f (x2) for all x1, x2 ∈ I.
(iv) Strictly decreasing on I if x1 < x2 in I ⇒f (x1) > f (x2) for all x1, x2 ∈ I.

Let f be continuous on [ a, b] and differentiable on the open interval (a, b).


Then

(i) f is increasing in [a, b] if f ′(x) > 0 for each x ∈ (a, b)


(ii) f is decreasing in [a, b] if f ′(x) < 0 for each x ∈ (a, b)
(iii) A function will be increasing (decreasing) in R if it is so in every
interval of R.
Let f be a function defined on an interval I. Then

(a) f is said to have a maximum value in I, if there exists a point c in


Mathematics Page 97 of 260 XII
I such that f (c) ≥ f (x), for all x ∈ I. The number f (c) is called the
maximum value of f in I and the point c is called a point of maximum
value of f in I.
(b) f is said to have a minimum value in I, if there exists a point c in I such
that f (c) ≤ f (x), for all x ∈ I. The number f (c), in this case, is called the
minimum value of f in I and the point c, in this case, is called a point of
minimum value of f in I.
(c) f is said to have an extreme value in I if there exists a point c in I such
that f (c) is either a maximum valueor a minimum value of f in I. The number
f (c), in this case, is called an extreme value of f in I and the point c is called
an extreme point.

ABSOLUTE MAXIMA AND MINIMA

Let f be a function defined on the interval I and c ∈ I. Then

(a) f(c) is absolute minimum if f (x) ≥ f(c) for all x ∈ I.


(b) f(c) is absolute maximum if f(x) ≤ f(c) for all x ∈ I.
(c) c ∈ I is called the critical point of f if f ′(c) = 0
(d) Absolute maximum or minimum value of a continuous function f on
[a, b] occurs at a or b or at critical points (i.e. at the points where f ′ is zero)
If c1 ,c2, … , cn are the critical points lying in [a , b], then
absolute maximum value of f = max{f(a), f(c1), f(c2), … , f(cn), f(b)}

absolute minimum value of f = min{f(a), f(c1), f(c2), … , f(cn), f(b)}.

LOCAL MAXIMA AND MINIMA

A function f is said to have a local maxima or simply a maximum value at x =


a if f(a ± h) ≤ f(a) for sufficiently small h.

A function f is said to have a local minima or simply a minimum value at x = a


if f(a ± h) ≥ f(a).

Mathematics Page 98 of 260 XII


FIRST DERIVATIVE TEST:

A function f has a maximum at a point x = a if

(i) f ′(a) = 0, and


(ii) f ′(x) changes sign from + ve to –ve in the neighbourhood of a (points
taken from left to right).
However, f has a minimum at x = a if

(i) f ′(a) = 0, and


(ii) f ′(x) changes sign from –ve to +ve in the neighbourhood of a.
If f ′(a) = 0 and f‘(x) does not change sign, then f(x) has neither
maximum nor minimum and the point a iscalled point of inflation.

The points where f ′(x) = 0 are called stationary or critical points. The
stationary points at which the function attains either maximum or
minimum values are called extreme points.

SECOND DERIVATIVE TEST


(i) A function has a maxima at x= a if f ′(x) 0 and f ′′ (a) <0
(ii) A function has a minima at x = a if f ′(x) = 0 and f ′′(a) > 0.
(iii) The test fails, if f'(c) = 0 and f”(c) = 0.

EXAMPLES:
1. The radius of a circle is increasing at the rate of 0.7 cm/sec. what is the rate of
increase of its circumference ?
Answer. The circumference of the circle (C) with radius (r) is given by
C = 2𝜋r
Therefore, the rate of change of circumference with respect to time is given by,
𝑑𝐶 𝑑𝐶 𝑑𝑟
𝑑𝑡
= 𝑑𝑟
. 𝑑𝑡

𝑑𝐶 𝑑 (2Π r) 𝑑𝑟 𝑑𝑟
= . = 2𝜋.
𝑑𝑡 𝑑𝑟 𝑑𝑡 𝑑𝑡

Mathematics Page 99 of 260 XII


𝑑𝑟
It is given that 𝑑𝑡
= 0.7 cm/s
𝑑𝐶
Therefore = 2 𝜋 (0.7) = 1.4 𝜋 cm/s
𝑑𝑡

2. The volume of a sphere is increasing at the rate of 8 cm3/s. Find the rate at
which its surface area is increasing when the radius of the sphere is 12 cm.
Answer: let r be the radius. V be the volume and S be the surface area of sphere
𝑑𝑉
Then, we have 𝑑𝑡 = 8 cm3/s
4
V = 3 𝜋r3
𝑑𝑉 4 𝑑𝑟
𝑑𝑡
= 3
𝜋 . 3r2. 𝑑𝑡
𝑑𝑟
8 = 4 𝜋r2 .
𝑑𝑡

. 𝑑𝑟 2
𝑑𝑡
= 𝛱𝑟2 cm/s

Now, S = 4𝜋r2
𝑑𝑆 𝑑
𝑑𝑡
= 𝑑𝑡 (4𝜋r2)
𝑑𝑟
= 8𝜋r.
𝑑𝑡
2
= 8𝜋r . 𝛱𝑟2

16
=
𝑟
𝑑𝑆 16 4
𝑑𝑡
= 12 = 3 cm2 / s.

r = 12

1. Show that the function f(x) = 4x3 – 18x2 + 27x – 7 is always increasing on R.
Answer:
We have, f(x) = 4x3 – 18x2 + 27x – 7
On differentiating both sides w.r.t. x, we get
f’(x) = 12x2 – 36x + 27
⇒ f'(x) = 3(4x2 -12x + 9)

Mathematics Page 100 of 260 XII


⇒ f'(x) = 3(2x – 3)2
⇒ f(x) > 0
⇒ For any x ∈ R, (2x – 3)2 > 0
Since, a perfect square number cannot be negative.
Given function f(x) is an increasing function on R.

4 . Find the intervals in which the function


𝑋4
f(x) = 4
– x3 – 5x2 + 24x + 12 is
(i) strictly increasing
(ii) strictly decreasing.
Answer:
𝑋4
We have, f(x) = – x3 – 5x2 + 24x + 12
4
On differentiating both sides w.r.t. x, we get
f’(x) = x3 – 3x2 – 10x + 24
= (x – 2) (x2 – x – 12)
= (x – 2) (x2 – 4x + 3x – 12)
= (x – 2) (x(x – 4) + 3(x – 4)
= (x – 2) (x – 4) (x + 3)
Now, put f'(x) = 0, which gives x = 2, 4 and -3 The points x = - 3 , x = 2 and x = 4
divides the whole real line into four disjoint intervals namely , (-∞,-3), (-3, 2), (2, 4),
(4, ∞)

for x ∈ (-∞, – 3), f'(x) < 0 for x ∈ (-3, 2), f’(x) > 0

for x ∈ (2, 4), f'(x) < 0 and for x ∈ (4, ∞), f'(x) > 0
∴ f(x) is strictly increasing in the intervals (-3, 2) and (4, ∞), and strictly
decreasing in the intervals (-∞, -3) and (2, 4).

5. Of all the closed cylindrical cans (right circular), which enclose a given
volume of 100 cm3, find the dimensions of the can which has the minimum
surface area?

Mathematics Page 101 of 260 XII


Let r and h be the radius and height of the cylinder, respectively. Then,
Volume (V) of the cylinder = πr2 h
⟹ 100 = πr2 h
⟹ h = 100/ πr2
Surface area (S) of the cylinder = 2 πr2 + 2 πr h = 2 πr2 + 2 πr × 100/ πr2
200
S = 2 πr2 + 𝑟
𝑑𝑆 200
= 4 πr - 𝑟 2
𝑑𝑟
For maximum or minimum, we must have
𝑑𝑆
=0
𝑑𝑟
200
4 πr - =0
𝑟2

4 πr3 = 200
50 1/3
r=( )
𝛱
𝑑2 𝑆 400
Now, =4π+
𝑑𝑟 2 𝑟3
𝑑2 𝑆 50 1/3
Therefore, 𝑑𝑟 2 ˃ 0, when r = ( )
𝛱
50 1/3
Thus, the surface area is minimum when r = ( ) .
𝛱

MULTIPLE CHOICE QUESTIONS:


1. The number that exceeds its square by the greatest amount is:
1
a) -1 b) 0 c) d) 1
2

solution.
Let the number be x, then y = x – x2
𝑑𝑦
= 1 – 2x
𝑑𝑥
𝑑𝑦 = 0
For maxima or minima, 𝑑𝑥
1
Therefore, x = 2

Mathematics Page 102 of 260 XII


𝒅𝟐 𝒚
and =-2˂0
𝒅𝒙𝟐

𝟏
Answer is option (c), x = 𝟐
2. The sides of an equilateral triangle are increasing at the rate of 2
cm/s. The rate at which the area increases when side is 8 cm is
a) 8 cm2/s b) 10 cm2/s
c) 8√3 cm2/s d) 𝟏𝟎√𝟑 cm2/s
solution:
Let the side of an equilateral triangle be x cm
𝒅𝒙
And = 2 cm /s
𝒅𝒕
√𝟑 2
Area of equilateral triangle, A = x
𝟒
𝒅𝑨 √𝟑 𝒅𝒙 √𝟑
On differentiating, 𝒅𝒕 = . 2x = .2. 8 . 2 = 8√𝟑 cm2/s
𝟒 𝒅𝒕 𝟒
Answer is option (c). 8√𝟑 cm2/s
3. A ladder, 5 m long, standing on a horizontal floor, leans against a
vertical wall. If the top of the ladder slides downwards at the rate of
10cm/sec, then the rate at which the angle between the floor and ladder is
decreasing when lower end of ladder is 2 meters from the wall is:
(a) 1/10 radian/sec (b) 1/20 radian/sec
(c) 20 radian/sec (d) 10 radian/sec
Sol. Let the angle between floor and the ladder be ɵ
A

B y
Let AB=x cm and BC=y cm
𝑥 𝑦
So sin ɵ= and Cos ɵ=
500 500
Then x=500 Sin ɵ and y=500 Cos ɵ

𝑑𝑥
Also 𝑑𝑡 = 10cm/ s

Mathematics Page 103 of 260 XII


𝒅ɵ
500 Cos ɵ. 𝒅𝒕 = 10cm/ s
𝒅ɵ 𝟏𝟎 𝟏
= 𝟓𝟎𝟎 𝐜𝐨𝐬 ɵ = 𝟓𝟎 𝑪𝒐𝒔ɵ
𝒅𝒕

For y= 2m =200 cm
𝒅ɵ 𝟏 𝟏𝟎
=𝟓𝟎. 𝒚 =
𝒅𝒕 𝒚
𝟓𝟎𝟎
𝟏𝟎 𝟏
= 𝟐𝟎𝟎 = 𝟐𝟎 rad/ s

Answer is option (b)

4. On the interval [0,1] the function x25 (1-x)25 takes its maxima
value at the point:
(a) 0 ( b) ½ ( c ) 1/3 ( d) ¼
25 25
Sol: f(x) = x (1-x)
f(x) = x25(75) (1-x)74(-1)+25x24(1-x)75
For maxima and minima,
-75x25(1-x)74+25x24(1-x)75=0
25x24(1-x)74[(1-x)-3x] =0
𝟏 𝟏
Either x =0 or x =1 or x = and f(x) is maximum at x =
𝟒 𝟒
Answer is option (d)

5. The maximum volume (in cu. m) of the right circular cone having slant
height 3 m is
(a) 3√𝟑 𝝅 (b) 6 𝝅
𝟒
(c) 2√𝟑 𝝅 (d) 𝟑 𝝅
𝟏
Sol. V= 𝟑 𝝅 r2 h, r2+h2= 9
𝟏
V= 𝝅 h (9-h2)
𝟑
𝒅𝑽 𝟏
= 𝟑 𝝅 (9-3h2) =0
𝒅𝒉
(9-3h2) =0
h2 =3, h = √𝟑

Mathematics Page 104 of 260 XII


𝟏
V = 𝟑 𝝅 (6) √𝟑 = 2√𝟑 𝝅
Answer is option (c)
6. The height of a right circular cylinder of maximum volume inscribed in a
sphere of radius 3 is
(a ) 2√𝟑 (b) √𝟑 (c ) √𝟔
𝟐
(d) √𝟑
𝟑
Sol. (a)

h = 2r Sinɵ
a = 2r Cosɵ
V = 𝝅 (r Cosɵ)2(2r Sinɵ)
V =2 𝝅 r3 Cos2ɵ Sinɵ
𝐝𝐕
= 𝝅 r3 (-2 Cosɵ Sin2ɵ + Cos3ɵ) =0
𝐝ɵ
𝟏
or tanɵ = √𝟐
𝟏
Hence h = 2x3x = 2√𝟑
√𝟑
Answer is option (a)

7. A spherical iron ball of 10 cm radius is coated with a layer of ice of


uniform thickness that melts at a rate of 50 cm3/ min. When the thickness of
ice is 5 cm, then the rate (in cm/ min) at which of the thickness of ice
decreases, is

Mathematics Page 105 of 260 XII


𝟓 𝟏 𝟏 𝟏
(a) (b) 𝟓𝟒𝛑 ( c ) 𝟑𝟔𝛑 (d)𝟏𝟖𝛑
𝟔𝛑

Sol. It is given that, a spherical iron ball of 10 cm radius is coated with a layer
of ice of uniform thickness, so let the thickness be ‘x’ cm, then the volume of
the ball is
𝟒
V= 𝟑 𝝅 (10+x)3h
On differentiating w.r.t. ’t’, we get
𝐝𝐕 𝐝𝐱
= 4 𝝅 (10+x)2 𝐝𝐭 ,
𝐝𝐭
Where, t is the time in min.
𝐝𝐕
Given, = -50 cm3/min
𝐝𝐭
Now when x = 5cm, then
𝐝𝐱
-50 = 4 𝝅 (10+5)2 𝐝𝐭

𝐝𝐱 −𝟓𝟎 𝟏
= 𝟒 𝝅 (𝟐𝟐𝟓) = cm/ min
𝐝𝐭 𝟏𝟖𝛑
Answer is option (d)
8. f(x) =x5 -5x4 +5 x3 + 1 has
a) Two maximum and two minimum value
b) Two maximum and one minimum value
c) One maximum and one minimum value
d) None of these

Sol: f(x) = x5 -5x4 +5 x3 + 1


fˈ(x) = 5x4-20x3+15x2
For maxima and minima fˈ(x) =0
fˈ(x) =0, So 5x2 ( x2-4x+3) = 0
5 x2 (x-3)(x-1) = 3, Hence x =0,1,3
Fˈˈ(x) = 20x3 – 60x2 + 30x
Fˈˈ(0) =0

Mathematics Page 106 of 260 XII


Fˈˈ(1) = 20-60+30 =-10 <0
Fˈˈ(3) = 20x27 - 60x9 + 30x3 =540-540+90 >0
Hence F(x) is maxima at x =1 and maximum value is F(x) = 2 and
F(x) is minima at x =3 and minimum value F(x) =-26

Answer is option (c)


9. The least value of a such that the function f given f(x) = x2 + ax +
1 is strictly increasing on (1, 2) is
(a) -2 (b) 4 (c ) 1 (d) -4
Solution. We have,
f(x) = x2 + ax + 1
fˈ(x) = 2x + a
Now, function f will be increasing in (1, 2), if fˈ(x) > 0 in (1, 2)
fˈ(x) > 0
⇒ 2x + a > 0
⇒ 2x > -a
−a
⇒x> 2
Therefore, we must find the least value of a such that
−a
x> 2
, when x ∈ (1, 2)

Thus, the least value of a for f to be increasing on (1, 2) is given by,


−a
=1
2

⇒ a = -2

Answer is option (a)

10. The rate of change of volume of a sphere with respect to its surface area,
when the radius is 2 cm , is –
(a) 4 cm3 / cm2 (b) 2 cm3/ cm2
( c ) ½ cm3/ cm2 (d) 1 cm3/ cm2
4
sol. V = 3 𝜋r3
dV
= 4𝜋r2
dr
Mathematics Page 107 of 260 XII
And S = 4𝜋r2
dS
= 8𝜋r
dr
dV r
= 2 = 1 at r = 2
dS

Answer is option (d)

Assertion & Reasoning

In each of the question given below, there are two statements marked
as Assertion (A) and Reason (R). Mark your answer as per the codes
provided below:

a). Both A and R are true, and R is the correct explanation of A.

b). Both A and R are true, but R is not the correct explanation of A.

c). A is true but R is false.

d). A is false but R is true.

1. Assertion (A): The rate of change of area of a circle with respect to its
radius r when r = 6 cm, is 12 𝜋 cm2/ cm
Reason (R): The rate of change of area of a circle with respect to its radius r
dA
is dr , Where A is the area of the circle.
Answer: a
2. Assertion (A): f(x) = tan x –x is always increasing
dy
Reason (R): Any function y = f(x) is increasing if dx > 0
Answer: a
3. Assertion (A): f(x) = x4 is decreasing in the interval (0, ∞)
dy
Reason (R): Any function y = f(x) is decreasing if dx < 0
Answer: d

4 Assertion (A): Both sin x and cos x are decreasing functions in the
π
interval ( 2 ,𝜋 )

Mathematics Page 108 of 260 XII


Reason (R): If a differentiable function decreases in (a, b) then its derivative
also decreases in (a, b).
Answer: d
SHORT ANSWER TYPE QUESTION :
1. A spherical ball of salt is dissolving in water in such a manner that the rate
of decrease of the volume at any instant is proportional to the surface.
Prove that the radius is decreasing at a constant rate.
Solution: Given, a spherical ball of salt
Then, the volume of ball V = 4/3 πr3 where r = radius of the ball
Now, according to the question we have
dV
∝ S, where S = surface area of the ball
dt
d
(4/3 πr3) ∝ 4𝜋 r2
dt
dr
4𝜋 r2 dt ∝ 4𝜋 r2
dr 4𝜋 r2

dt 4𝜋 r2
dr
= k.1, where k is the proportionality constant
dt
Hence the radius of ball is decreasing at a constant rate.

2. A kite is moving horizontally at a height of 151.5 meters. If the speed of kite


is 10 m/s, how fast is the string being let out, when the kite is 250 m away
from
The boy who is flying the kite. The height of boy is 1.5 m.

Given,

Mathematics Page 109 of 260 XII


Height of the Kite (h) = 151.5 m
Speed of the kite (V) = 10 m/s
Let FD be the height of the kite and AB be the height of the kite and AB be
the height of the boy.
Now, let AF = x m
So, BG = AF = x
And, dx/dt = 10 m/s
From the figure, it is seen that
GD = DF – GF = DF – AB
= (151.5 – 1.5) m = 150 m [As AB = GF]
Now, in ∆ BDG
BG2 + GD2 = BD2 (By Pythagoras Theorem)
x2 + (150)2 = (250)2
x2 + 22500 = 62500
x2 = 62500 – 22500 = 40000
x = 200 m
Let initially the length of the string be y m
So, in ∆ BDG
BG2 + GD2 = BD2
x2 + (150)2 = y2AAAAAAA
Differentiating both sides w.r.t., t, we have
dx dy
2x. + 0 = 2y.
dt dt
dy x dx
= y . dt
dt

dy 200
= 250 . 10 = 8 m/sec
dt
Mathematics Page 110 of 260 XII
3. A rectangular sheet of fixed perimeter with sides having their lengths in the
ratio 8:15 is converted into an open rectangular box by folding after
removing squares of equal area from all four corners. If the total area of
removed squares is 100, the resulting box has maximum volume. Find the
lengths of sides of the rectangular sheet.

Sol. Let side of rectangular sheets L=15x and B = 8x


Volume of rectangular box = (15x-2a) (8x-2a) X a
= (4a3-46xa2+120xa)
dv
= 12a2 -92ax+120
da
dv
=0
da
3a2-23ax+30x= 0
(3a-5x) (a-6x) = 0
a 5
a = 6x or =3
x
Area of removed squares = 4a2
4a2 = 100
a=5
5
therefore x = 3 or x = 6
d2 v
again da2 < 0 at x = 3
therefore, volume is maximum when x = 3
Thus B = 8x = 24 and L = 15x = 45
4. A man of height 1.5 m is moving away from a lamp post at the rate of 1.2
Mathematics Page 111 of 260 XII
m/s. If the height of lamp post be 4.5m find the rate at which shadow of the
man is increasing.
dy
Given = 1.2 m/s
dt

Sol. As 𝜟 AOB ̴ Δ NMB therefore


𝟒.𝟓 𝐱+𝐲
=
𝟏.𝟓 𝐱
3x = x+y
x = y/2
𝐝𝐱 𝟏 𝐝𝐲
= 𝟐 𝐝𝐭
𝐝𝐭
𝐝𝐱 𝟏
Therefore, = (1.2) = 0.6 m/s
𝐝𝐭 𝟐
5. Find the intervals in which the function
f(x) = 20 – 9x + 6x2 – x3 is
(i) Strictly increasing.
(ii) Strictly decreasing
Sol. Given function is f(x) = 20 – 9x + 6x2 – x3.
On differentiating both sides w.r.t. x, we get f'(x) = – 9 + 12x – 3x2
On putting f'(x) = 0, we get
-9 + 12x – 3x2 = 0
⇒ -3(x2 – 4x + 3) = 0
⇒ -3(x – 1)(x – 3) = 0
⇒ (x – 1)(x – 3) = 0
Mathematics Page 112 of 260 XII
x – 1 = 0 or x – 3 = 0 ⇒ x = 1 or 3
Now, we find intervals in which f(x) is strictly increasing or strictly
decreasing.

Interval f(x) = – 3 (x -1) (x – Sign of f'(x) Nature of f(x)


3)

x<1 (-) (-) (-) – ve Strictly decreasing

1< x < 3 (-) (+) (-) + ve Strictly increasing

x>3 (-) (+) (+) – ve Strictly decreasing

6. Find the intervals in which the function given by f(x) = sinx – cosx, 0 ≤ x ≤
2π is strictly increasing or strictly decreasing.
Sol:
Given function is f(x) = sin x – cos x, 0 ≤ x ≤ 2π
On differentiating both sides w.r.t. x, we get f'(x) = cos x + sin x
On putting f'(x) = 0, we get
Cos x + Sin x = 0
⇒ Sin x = - Cos x
Sin x
⇒ = -1
Cos x
⇒ tan x = -1
3π 7π
For x ɛ [0,2𝜋 ], tan x =tan or tan x = tan
4 4
3π 7π
X= ,
4 4
Interval f'(x) = cos x + sin x Sign of Nature of f(x)
f'(x)

3π π π π
0<x< 4 At x = 2 , Cos 2 + Sin 2 = 1 +ve Strictly

Mathematics Page 113 of 260 XII


increasing

3π 7π 5π 5π 5π
<x< 4 At x = , Cos + Sin = -ve Strictly
4 6 6 6
−√3+1 decreasing
2

7π 23π 23π 23π


<x<2𝜋 At x = , Cos + Sin +ve Strictly
4 12 12 12
>0 increasing

LONG ANSWER TYPE QUESTION


1. A tank with rectangular base and rectangular sides, open at the top is to
be constructed, so that its depth is 2 m and volume is 8 m 3. If building of
tank cost ₹ 70 per sq m for the base and ₹ 45 per sq m for sides. What is the
cost of least expensive tank?
Sol:
Let x m be the length, y m be the breadth and h=2m be the depth of the
tank.
Let ₹ H be the total cost for building the tank. Now, given that h = 2 m and
volume of tank = 8 m3.
Clearly, area of the rectangular base of the tank
= length × breadth = xy m2

and the area of the four rectangular sides


= 2 (length + breadth) × height
= 2 (x + y) × 2= 4 (x + y) m2

∴ Total cost, H = 70 × xy + 45 × 4 (x + y)
⇒ H = 70xy + 180 (x + y) …………(i)

Also, volume of tank = 8 m3


⇒ l × b × h = 8 ⇒ x × y × 2 = 8 ⇒ y = 4/x …..(ii)

Mathematics Page 114 of 260 XII


On putting the value of y from Eq. (ii) in Eq. (i), we get
4 4
H = 70x ( )+ 180 (x + )
x x

4
H = 280 + 180 (x + x ) …..(iii)

On differentiating both sides w.r.t. x, we get


dH 4
= 180 (1- x2 )
dx

dH
For maxima and minima put =0
dx

4
180 (1- x2 ) = 0

4
1- x2 = 0

x2 = 4 ⇒x = 2

d2 H 8
Also = 180 x x3
dx2

d2 H
For x = 2 >0
dx2

⇒ H is least at x = 2.
Also, the least cost = 280 + 180(2 + 4/2)
= 280 + 180 × 4 = 280 + 720 = ₹ 1000
Hence, the cost of least expensive tank is Rs. 1000.

2. Show that the semi-vertical angle of the right circular cone of given
𝟏
surface area and maximum volume is sin-1(𝟑).
Solution:

Let be the radius and be the height of the cone and semi-vertical
angle be

Mathematics Page 115 of 260 XII


Total Surface area of cone (S) =

1
Volume of cone (V) = 𝜋x2y
3

= [Using quotient rule]

……….(ii)

Now

Mathematics Page 116 of 260 XII


[height cannot be negative]

Since, , therefore, Volume is maximum at

Now Semi-vertical angle of the cone,

3. Show that the height of the cylinder of maximum volume that can be inscribed
in a

2R
sphere of radius R is . find the
√3

maximum volume.

Sol. A sphere of fixed radius (R) is given.


Let r and h be the radius and the height of the cylinder respectively.

Mathematics Page 117 of 260 XII


From the given figure, we have
The volume (V) of the cylinder is given by V = 𝜋 𝑟2 h

V =2𝜋 𝑟 2 √𝑅2 − 𝑟2
dV = 2𝜋 𝑟2 (−2𝑟)
4 𝜋r √𝑅2 − 𝑟2 +
dr 2√𝑅2 − 𝑟2

dV = 4 𝜋r (𝑅2 − 𝑟2 )− 2𝜋r3
dr √𝑅2 − 𝑟2

dV 4 𝜋r𝑅2 – 6 𝜋𝑟3
=
dr √𝑅2 − 𝑟2

dV
Now, =0
dr

4 𝜋r𝑅2 – 6 𝜋𝑟3 =
0
√𝑅2 − 𝑟2

2R2
r2 =
3

d2 V
Now =
dr2
(−2𝑟)
√𝑅2 − 𝑟2 (4 𝜋 𝑅2 − 18 𝜋𝑟2 )–(4 𝜋 𝑅2 − 6 𝜋𝑟3 )
2√ 𝑅2 − 𝑟2
(R2−r2 )

=
(R2 −r2 )(4 𝜋 𝑅2 − 18 𝜋𝑟2 )+ 𝑟(4 𝜋 𝑅2 − 6 𝜋𝑟3 )
(R2 −r2 )3/2

4 𝜋 𝑅4 − 22 𝜏 𝑟2 𝑅2 + 12 𝜋 𝑟4 + 4 𝜋 𝑟2 𝑅2
= (R2−r2 )3/2

2 R2 d 2 V
Now it can be observed that at r2 = 3
, dr2 < 0

2 R2
The volume is the maximum when r2 = 3
,

2 R2 2R
when r2 = , the height of the cylinder is
3 √3

Mathematics Page 118 of 260 XII


Hence, the volume of the cylinder is the maximum when the height of the
2R
cylinder is .
√3

4. An isosceles triangle inscribed in the circle with radius a. Show that the
π
area of triangle is maximum when θ = 6 .

1
Therefore, Area of the Triangle ABC i.e. ∆ = BC.AD
2
1
= 2 2aSin 2𝜃 (a+aCos2𝜃)

= a2 Sin 2𝜃 (1+Cos2𝜃)

1
⇒ ∆ = a2 Sin 2𝜃 + a2 Sin 4𝜃
2

d∆
Therefore, = 2a2 Cos2𝜃 + 2a2 Cos4𝜃
d𝜃

= 2a2 (Cos2𝜃 + Cos4𝜃)

d∆
d𝜃
= 0 ⇒ Cos 2𝜃 = - Cos4𝜃 = Cos (𝜋 − 4𝜃)

π
Therefore, 2𝜃 = 𝜋-4𝜃 ⇒ 𝜃 = 6
Mathematics Page 119 of 260 XII
d2 ∆ π
dθ2
= 2a2 (-2Sin 2𝜃 - 4Sin 4𝜃) <0 (at 𝜃 = 6 )

π
Therefore, Area of triangle is maximum when 𝜃 =
6

CASE STUDY BASED QUESTIONS

1. A cylindrical container of inner radius r and height h is to be made from certain


solid material with the following constraints, it has fixed inner volume of V
mm3, has a 2mm thick solid wall and is open at the top. The bottom of the
container is solid circular disc of thickness 2mm and is of radius equal to the
outer radius of the container.
Based on the above information, answer the following questions.

i) Volume of the material (V1) is:

a). V1 = 𝜋(r + 2 )2 . 2 + 𝜋(r + 2 )2h – 𝜋r2h

b). V1 = 𝜋(r + 2 )2h – 𝜋r2h

c). V1 = 𝜋r2h

d). V1 = 𝜋(r + 2 )2 . 2 - 𝜋(r + 2 )2h – 𝜋r2h

ii) Express Volume of the material (V1) in terms of V and r .

iii) If the volume of the material used to make the container is minimum when
V
the inner radius of the container is 10mm, then find the value of 250 π
.

sol. As , V = 𝜋r2h

Volume of material, V1 = 𝜋(r + 2 )2 . 2 + 𝜋(r + 2 )2h – 𝜋r2h

V1 = 2𝜋 (r + 2 )2 + 4𝜋ℎ( r + 1 )

V
V1 = 2𝜋 ( (r + 2 )2 + 2( r + 1 ) . πr2 )
Mathematics Page 120 of 260 XII
Given volume is maximum when r = 10 mm

dV 1
= 0 when r = 10
dr

dV 1 −1 2 V
= 2𝜋 (2 (r + 2) + 2( - ). )=0
dr r2 r3 π

2V −10−2
24 + π
( 103
)=0

V = 103 𝜋

V
Therefore, =4
250π

2. A wire of length 36 m is cut into two pieces, one of the pieces is bent to
form a square of side x and the other is bent to form a circle of radius r,
such that the sum of the areas of the square and the circle is minimum.
Based on the above information, answer the following questions.

a). find the relation between x and y.


b). Express the combined area (A) of square and circle in terms of x.
c). find the value of y for which area (A) is minimum.
or
4
If the circumference of the circle is k, find the value of ( π + 1) k.

Sol. As 4x + 2𝜋y = 36
A = x2 + 𝜋y2
1
A = x2 + π( 18 – 2x )2
dA 2
dx
= 2x + π (18 – 2x) ( -2)
dA
=0
dx
36
x = π+4
d2 A
dx2
>0
Mathematics Page 121 of 260 XII
18
y=
π+4

also, circumference of circle = k


k = 2𝜋y
4 4 18
therefore, value of ( π + 1) k = ( π + 1) 2 𝜋 (π+4 ) = 36

3. A telephone company in a town has 500 subscribers on its list and


collects fixed charges of Rs. 300 per subscriber per year. The company
proposes to increase the annual subscription and it is believed that of
every increase of Rs. 1, one subscriber will discontinue the service.
Based on the above information, answer the following questions
1. If the company increases the annual subscription by Rs. x, express the total
revenue of the company in terms of x.
2. Find the value of x for which the revenue is maximum.
3. Find the maximum revenue of the company.
Sol. As the company increases the annual subscription by Rs. x,
So, x subscribers will discontinue the service.
Therefore, total revenue R(x) = (500 – x) (300 + x)
R(x) = - x2 + 200x + 150000
On differentiating w.r.t. x, Rˈ(x) = -2x + 200
Now, Rˈ(x) = 0
x = 100
and Rˈˈ(x) = -2 <0
So, R is maximum when x = 100.
Therefore, maximum revenue = (500 -100) (300 + 100) = 160000
4. A toy consists of a rectangular parallelopiped shaped box and a spherical ball
inside it. The sides of the box are x, 2x, x/3, while the radius of the ball is r

Mathematics Page 122 of 260 XII


Based on the above information, answer the following questions.
a) If sum of the surface areas of the box and ball are given to be constant S,
then express x in terms of S.
b) find the value of r for which volume V, the sum of the volume of
parallelepiped box and sphere is minimum.
c) find the relation between the length of the box and radius of the sphere.
or
find the minimum value of the volume V.
Sol. As sum of the surface area of the box and sphere = S
x x
S = 2 (x. 2x + 2x. + . x) + 4 𝜋r2
3 3
2 2
S = 6x + 4 𝜋r
𝑆−4 𝜋r2
x= √ 6

Now, sum of the volume of the parallelepiped box and sphere = V


x 4
V = x (2x) ( ) + 𝜋r3
3 3
2 𝑆−4 𝜋r2 4
V= ( )3/2 + 𝜋r3
3 6 3
dV = 1
4 𝜋r [ r - √𝑆 − 4 𝜋r2 ]
dr 3√6
dV =
For maxima and minima, dr
0
− 4 𝜋r
3√6
√𝑆 − 4 𝜋r2 =
- 4 𝜋r298

𝑆 − 4 𝜋r2 = 54r2
𝑆
r= √54+4𝜋

𝑆−4 𝜋r2
since, x2=
6
2
2
x = (54r + 4 𝜋r 2
- 4 𝜋r2 ) / 6
x2 = 9r2
x = 3r
Mathematics Page 123 of 260 XII
2 4
As V = x3 + 𝜋r3
3 3
2 4
Minimum value of V = 3(3r )3 + 3 𝜋r3
4
= 18r3 + 3 𝜋r3
4
= (18 + 3
𝜋) r3
4 𝑆
= (18 + 𝜋) ( ) 3/2
3 54+4𝜋

Mathematics Page 124 of 260 XII


Chapter – 7
Integrals
Important formulas:

x n 1
 x dx  c ;
n
1.
n 1

2.  dx  x  c ;
3
2 2
3.  x dx 
3
x c ;

 e dx  e c;
x x
4.

ax
 a dx  c;
x
7.
log a

dx
8.  x
 log x  c

9.  sin xdx   cos x  c ;


10.  cos xdx  sin x  c ;
 sec xdx  tan x  c ;
2
11.


12. sec x. tan xdx  sec x  c ;

13.  cos ecx. cot xdx   cos ecx  c ;



14. cos ec xdx   cot x  c
2

Mathematics Page 125 of 260 XII


dx
15.  1 x 2
 sin 1 x  c   cos 1 x  c ;

dx
16. x x 12
 sec 1 x  c   cos ec 1 x  c

dx
17. 1 x 2
 tan 1 x  c   cot 1 x  c ; 18.

 sec xdx  log sec x  tan x  c ;


19.  cos ec xdx  log cos ecx  cot x  c ; 20.

 tan xdx  log sec x  c   log cos x  c


21.  cot xdx  log sin x  c   log cos ecx  c ; 22.

dx 1 ax
a 2
x 2

2a
log
ax
c

dx 1 xa
23. x a 22

2a
log
xa
 c; 24.

dx 1 1 x
 a 2  x 2  a tan a  c ;
dx x
25. x x2  a2
 sec 1
a
c 26.

dx x
 a x2 2
 sin 1
a
c;

dx
27.  x a 2 2
 log x  x 2  a 2  c ; 28.

dx
 a x
2 2
 log x  a 2  x 2  c ;

Mathematics Page 126 of 260 XII


x a2  x2 a2 x
 a  x dx   sin 1  c
2 2
29.
2 2 a

x x2  a2 a2
30.  x 2  a 2 dx 
2

2
log x  x 2  a 2  c ;

x x2  a2 a2
31.  x 2  a 2 dx 
2

2
log x  x 2  a 2  c

du

32. u.vdx  u vdx    dx  vdx OR

d
 { f ( x) .g ( x)}dx  f ( x)  g ( x)dx   {
dx
f ( x). g ( x)dx}dx  c

Preference for first function in integration “BY PARTS” should be “ILATE”


where

I= Inverse Trigonometry Functions, L= Logarithmic Functions, A= Algebraic


Functions

T= Trigonometric Functions, E= Exponential Functions

INTEGRATION BY PARTIAL FRACTIONS

Denominator q(x) Form of given fraction Format of Partial Fraction

(x+a) (x+b) P( x) A B

x  a x  b xa xb

(x+a) (x+b) (x+c) P( x) A B C


 
x  a x  b ( x  c) xa xb xc

Mathematics Page 127 of 260 XII


(x+a)2 (x+b) P( x) A B C
 
(x + a) 2 (x + b) x  a ( x  a) 2
xb

(x+a) (x2+b) P( x) A Bx  C
 2
(x + a) (x 2 + b) xa x b

( x 2  a)( x 2  b) P( x) A
 2
B
( x  a)( x 2  b)
2
x a x b
2

(x+a)(ax2+bx+c) P( x) A Bx  C
 2
where ax2+bx+c is (x + a)(ax 2 + bx + c) x  a ax  bx  c
non factorizable

. Properties of definite intrigation:

a
1.  f ( x).dx  0
a

b a
2. 
a
f ( x).dx    f ( x).dx
b

b b
3.  f ( x).dx   f (a  b  x).dx
a a

a a
4.  0
f ( x).dx   f (a  x).dx
0

b c b
5. 
a
f ( x).dx   f ( x).dx   f ( x).dx Here a < c < b
a c

a
6.  f ( x).dx 
a
0 if f ( x)   f ( x) i.e. f ( x) is odd function.

Mathematics Page 128 of 260 XII


2a
7.  f ( x).dx 
0
0 if f (2a  x)   f ( x)

a
2  f ( x).dx if f ( 2a  x )  f ( x )
0

b b
8. 
a
f ( x).dx   f ( z )dz
a

QUESTIONS (MCQ’S)

1.

(a) sin² x – cos² x + C


(b) -1
(c) tan x + cot x + C
(d) tan x – cot x + C

2. Given ∫ 2x dx = f(x) + C, then f(x) is

3 ∫cot²x dx equals to
(a) cot x – x + C
(b) cot x + x + C
(c) -cot x + x + C
(d) -cot x – x + C

Mathematics Page 129 of 260 XII


4.

(a) log |sin x + cos x|


(b) x
(c) log |x|
(d) -x

Mathematics Page 130 of 260 XII


Then the value of a is

(a) 0 (b) ½
( c)1 ( d) 2
8.

(a) 0 (b) ½
( c)1 ( d) 2

ANSWERS (MCQ ‘S)

ANSWER 1- CORRECT OPTION IS (d)

ANSWER 2- CORRECT OPTION IS (C)

ANSWER 3- CORRECT OPTION IS (d)

Mathematics Page 131 of 260 XII


∫ (cosec²x -1)dx = -cot x – x + C

ANSWER 4- CORRECT OPTION IS (d)

ANSWER 5- CORRECT OPTION IS (c)

ANSWER 6- CORRECT OPTION IS (c)

Mathematics Page 132 of 260 XII


ANSWER 7- CORRECT OPTION IS (b)

ANSWER 8- CORRECT OPTION IS (a)

COMPETENCY BAESD QUESTIONS (CASE BASED MCQ’S)

Read the given extract and answer the following questions

Q 1. Karan & Shahrukh were discussing about integration, in their discussion


following points were discussed.

( i) In integration when degree of numerator is larger than denominator then we


have to divide the numerator till degree of numerator is reduced when
comparison to denominator.

Mathematics Page 133 of 260 XII


( ii) While we have question in the form of 𝒂𝟑 + 𝒃𝟑 we can use identity to
factorize the term as (𝒂 + 𝒃)(𝒂𝟐 + 𝒃𝟐 − 𝒂𝒃)

( iii) if we have derivative available in any integration question we put the other
term or substitute the other term as t then proceed

On the basis of the above discussion answer the following


questions

𝒔𝒊𝒏𝟔 𝒙+𝒄𝒐𝒔 𝟔 𝒙
(i) ∫ 𝒔𝒊𝒏𝟐 𝒙𝒄𝒐𝒔𝟐𝒙
𝒅𝒙
(𝒂) 𝒕𝒂𝒏𝒙 − 𝒄𝒐𝒕 𝒙 − 𝒙 + 𝒄
(𝒃) 𝒕𝒂𝒏𝒙 𝒄𝒐𝒕 𝒙 + 𝟑𝒙 + 𝒄
(𝒄) 𝒕𝒂𝒏𝒙 − 𝒄𝒐𝒕 𝒙 − 𝟑𝒙 + 𝒄
(𝒅) 𝒕𝒂𝒏𝒙 𝒄𝒐𝒕 𝒙 − 𝒙 + 𝒄

ANSWER:- CORRECT OPTION IS ( C)


𝒙𝟑
(ii) ∫ 𝒙+𝟐 𝒅𝒙
𝒙𝟑
(𝒂)
𝟑
− 𝒙𝟐 + 𝟒𝒙 − 𝟖𝒍𝒐𝒈|𝒙 + 𝟐| + 𝒄
𝟑
𝒙
(𝒃) − 𝒙𝟐 + 𝒙 − 𝒍𝒐𝒈|𝒙 + 𝟐| + 𝒄
𝟑
𝒙𝟑
(𝒄) − 𝟐𝒙𝟐 + 𝒙 − 𝟖𝒍𝒐𝒈|𝒙 + 𝟐| + 𝒄
𝟑
𝒙𝟑
(𝒅) − 𝟑𝒙𝟐 + 𝒙 − 𝒍𝒐𝒈|𝒙 + 𝟐| + 𝒄
𝟑

ANSWER:- CORRECT OPTION IS ( a)

𝟐𝒙+𝟓
(iii) ∫ 𝒙𝟐+𝟓𝒙−𝟕 𝒅𝒙
(𝒂) 𝒍𝒐𝒈|𝟐𝒙𝟐 + 𝒙 − 𝟕| + 𝒄
(𝒃) 𝒍𝒐𝒈|𝒙𝟐 + 𝟓𝒙 + 𝟕| + 𝒄
(𝒄) 𝒍𝒐𝒈|𝒙𝟐 + 𝒙 − 𝟕| + 𝒄
(𝒅) 𝒍𝒐𝒈|𝒙𝟐 + 𝟓𝒙 − 𝟕| + 𝒄

ANSWER:- CORRECT OPTION IS ( d)

Mathematics Page 134 of 260 XII


𝒔𝒆𝒄𝟐𝒙
(iv) ∫ 𝟑+𝒕𝒂𝒏 𝒙 dx
(𝒂) 𝒍𝒐𝒈|𝟑 + 𝒄𝒐𝒕 𝒙| + 𝒄
(𝒃) 𝒍𝒐𝒈|𝟐 + 𝒕𝒂𝒏 𝒙| + 𝒄
(𝒂) 𝒍𝒐𝒈|𝟓 + 𝒄𝒐𝒕 𝒙| + 𝒄
(𝒂) 𝒍𝒐𝒈|𝟑 + 𝒕𝒂𝒏 𝒙| + 𝒄

ANSWER:- CORRECT OPTION IS ( d)

ASSERTION-REASON BASED QUESTIONS


In the following questions, a statement of assertion (A) is followed by a
statement of Reason (R). Choose the correct answer out of the following
choices.
(a) Both A and R are true and R is the correct explanation of A.
(b) Both A and R are true but R is not the correct explanation of A.
(c) A is true but R is false.
(d) A is false but R is true.
𝜋

QUESTION1: Assertion (A): ∫− 𝜋 𝑠𝑖𝑛3 𝑥 𝑑𝑥 = 0


2

2
𝑑
Reason (R): 𝑑𝑥 𝑠𝑖𝑛𝑥 = cos 𝑥
𝜋

QUESTION 2: Assertion (A): ∫− 𝜋 𝐶𝑂𝑆 3 𝑥 𝑑𝑥 = 0


2

2
Reason (R): Cos(-x)= Cos x i.c. cos x is even function
𝑥6
QUESTION 3: Assertion (A): ∫ 𝑥 5 𝑑𝑥 = +c
6
x n 1
 x dx  c
n
Reason (R):
n 1
𝑒 −2𝑥
QUESTION4: Assertion (A): ∫ 𝑒 −2𝑥 𝑑𝑥 = +𝑐
−2
Reason (R):  sin xdx   cos x  c

ANSWER 1- Correct answer is b as both statements are true but R is not


correct explanation of ANSWER 2- Correct answer is d as first statement
is false and second statement is true
ANSWER 3- Correct answer is a as both statements are true and R is
correct explanation of A
Mathematics Page 135 of 260 XII
ANSWER 4- Correct answer is b as both statements are true but R is not
correct explanation of A

LONG ANSWER TYPE QUESTIONS

QUESTION 1 :-

Integrate: ∫ sin3 x cos2x dx

Solution:

Given that, ∫ sin3 x cos2x dx


This can be written as:
∫ sin3 x cos2x dx = ∫ sin2 x cos2x (sin x) dx
=∫(1 – cos2x ) cos2x (sin x) dx —(1)
Now, substitute t = cos x,
Then dt = -sin x dx
Now, equation can be written as:
Thus, ∫ sin3 x cos2x dx = – ∫ (1-t2)t2 dt
Now, multiply t2 inside the bracket, we get
= – ∫ (t2-t4) dt
Now, integrate the above function:
= – [(t3/3) – (t5/5)] + C —(2)
Where C is a constant
Now, substitute t = cos x in (2)
= -(⅓)cos3x +(1/5)cos5x + C
Hence, ∫ sin3 x cos2x dx = -(⅓)cos3x +(1/5)cos5x + C

QUES 2:-

Mathematics Page 136 of 260 XII


ANSWER:-

= ∫(tan x – cot x) dx
= ∫ tan x dx – ∫ cot x dx
= log |sec x| – [- log|cosec x|] + C
= log |sec x| +log|cosec x| + C
= log |sec x ∙ cosec x| + C

QUESTION 3-

QUESTION 4-

ANSWER:

Mathematics Page 137 of 260 XII


QUESTION 5-

ANSWER :

QUESTION 6-

ANSWER:
Mathematics Page 138 of 260 XII
QUESTION 7-

ANSWER:

QUESTION 8:

ANSWER

Mathematics Page 139 of 260 XII


Mathematics Page 140 of 260 XII
QUESTION 9:-

ANSWER

Mathematics Page 141 of 260 XII


Mathematics Page 142 of 260 XII
10.

ANSWER 10.

QUESTION 11.

ANSWER 11.
Mathematics Page 143 of 260 XII
QUESTION 12.

ANSWER 12.

Mathematics Page 144 of 260 XII


Chapter - 8
Application of Integrals

Chapter Notes:-
y-axis
 Area between the curve y = f(x), x-axis
and x = a and x = b (shown in figure) is given by y = f(x)

b b
A = ∫a y dx = ∫a f(x) dx
O x=a x=b x-axis

y-axis
 If the area between the curve y = f(x), x-axis
and x = a and x = b lies below the x-axis then x=a x=b
O x-axis
b b
A= | ∫a y dx | = | ∫a f(x)dx |
y = f(x)

x = f(y)

 Area between the curve x = f(y), y-axis


and y = a and y = b (shown in figure) is given by
y=b

b b
A = ∫a x dy = ∫a f(y) dy y=a

O x-axis

y-axis

Some Formula used :

Mathematics Page 145 of 260 XII


𝐱𝐧+𝟏
∫ 𝐱 𝐧 𝐝𝐱 = +𝐜 (n  -1)
𝐧+𝟏

∫ 𝐜𝐨𝐬 𝐱 𝐝𝐱 = 𝐬𝐢𝐧 𝐱 + 𝐜

∫ 𝐬𝐢𝐧 𝐱 𝐝𝐱 = − 𝐜𝐨𝐬 𝐱 + 𝐜

𝐱 𝐚𝟐 𝐱
∫ √𝐚𝟐 − 𝐱 𝟐 dx = 𝟐 √𝐚𝟐 − 𝐱 𝟐 + 𝟐 𝐬𝐢𝐧−𝟏 (𝐚) + 𝐜
Graph of some functions Y
Y

O
X’ X
X’ X
O

Y’ Y’
Parabola: x2 = 4ay Parabola: x2 = - 4ay

Y Y

X’ X X’ O X
O

Y’ Y’
Parabola: y2 = 4ax Parabola: y2 = - 4ax

Y Y

X’ X X’ X
O O

Mathematics Page 146 of 260 XII


Y’ Y’ 2
Circle : 𝑥 2 + 𝑦 2 = 𝑟 2 𝑥2 𝑦
Note Ellipse : 𝑎2
+ 𝑏2 = 1
(𝒙 − 𝒉)𝟐 + (𝒚 − 𝒌)𝟐 = 𝒓𝟐
is a circle with centre at
(h,k) and radius r.
Y
Y
4
3
2
1
X’ X
O X 0
X
-4 -3 -2 -1 1 2 3 4
-1
-2
-3
Y’ -4
y = |x|
Solved Examples Y
y=|x-2|

1. Find area of the region bounded by curve y2 + x = 0 and x – y + 6 = 0.


Solution: -
Y
y2 = - x

y=2
X’ X
O
y=-3

Y’

The region bounded by curve y2 + x = 0 (parabola)


and x – y + 6 = 0 (straight line) is the shaded region
which shown in the figure.

Intersection points of the curves are


y2 + x = 0 and x – y + 6 = 0
 y2 + y - 6 = 0
 (y+3)(y-2) = 0
 y = -3 or y=2
 when y = -3 , x = -9 or when y = 2 , x = -4
Intersection points of the curves are (- 9, -3) and (- 4, 2)
Required area is
Mathematics Page 147 of 260 XII
2
A = | ∫−3{f(y) − g(y)}dy | {Where f(y) = y – 6 and g(y) = - y2}
2
= | ∫−3{(y − 6) − (−y 2 )}dy |
2
= | ∫−3{y 2 + y − 6}dy |
2
y3 y2
= |[ 3 + − 6y] |
2 −3

(2)3 (2)2 (−3) 3 (−3) 2


= |( + − 12) − ( + − 6x(−3))|
3 2 3 2

(2)3 (2)2 (−3) 3 (−3) 2


= |( + − 12) − ( + + 18)|
3 2 3 2

8 4 −27 9
= |(3 + 2 − 12) − ( + 2 + 18)|
3

125
= |− |
6

125
= sq unit
6

2. Find area of the region bounded by curve 9x2 + y2 ≤ 36 and 3x + y - 6 ≥ 0.

Solution: - Y
The region bounded by curve 9x + y ≤ 36 (ellipse) 2 2 (0,6)
and 3x + y - 6 ≥ 0 (straight line) is the shaded region
which shown in the figure. (2,0)
Intersection points of the curves are X’ X
O
9x2 + y2 = 36 and 3x + y - 6 = 0
 9x2 + (6-3x)2 = 36
 18x2 - 36x + 36 = 36
Y’
 18x2 - 36x = 0
 18x(x-2) = 0
 x = 0 or x = 2
when x = 0 , y = 6 or when x = 2 , y = 0
Intersection points of the curves are (0, 6) and (2, 0)
Required area is

Mathematics Page 148 of 260 XII


2 2
A = ∫0 f(x)dx − ∫0 g(x)dx {Where f(x) = 3√4 − x 2 and g(x) = 6 – 3x}
2 2
= 3 ∫0 √4 − x 2 dx − ∫0 (6 − 3x)dx
2
x 4 x 2 3x2
= 3 [2 √4 − x 2 + 2 sin−1 2] − [6x − ]
0 2 0

2 4 2 0 4 0 12
= 3 [ √4 − 22 + sin−1 − ( √4 − 02 + sin−1 )] − [12 − − (0)]
2 2 2 2 2 2 2

4 2
= 3 [0 + 2 sin−1 2 − 0] − [6]

= 3[2sin−1 1] − [6]

= 6sin−1 1 − 6

= −6
2

= (3π − 6)sq unit

Questions for practice


Short Answer Questions (2marks)
1. Find area of region bounded by y = x3, x axis, x = 1 and x = - 2
Solution: -
The region bounded by the curves shown in figure
Y
It is clear from figure the region from x = -2 to x = 0 is 3 y=x3
below the x-axis and the region from x = 0 to x = 1 is 2
1
above the x-axis. -2 -1
X
X 0
Required area -3 1 2 3
-1
0 1
A = |∫−2 f(x)dx | + ∫0 f(x)dx -2
-3
0 1 Y
A = |∫−2 x 3 dx | + ∫0 x 3 dx
0 1
x4 x4
A = |[ 4 ] | + [ 4 ]
−2 0
(−2) 4 14
A = |[0 − ]| + [ − 0]
4 4
16 1 16 1 17
A = |[− ]| + [ ] = + = sq unit
4 4 4 4 4

Mathematics Page 149 of 260 XII


2. Find area of region bounded by the curve y2 = 4x, y axis and y = 3
Solution: -
The region bounded by the curves shown in figure Y
Required area y=3
3 1 3 1 1
A = ∫0 f(y)dy = 4 ∫0 y 2 dy = 12 [y 3 ]30 = 12 [27 − 0] X’ O
X
9
A = 4 sq units
y2 = 4x
Y’
3. Find area of region bounded by curve y = x and x = 3 and x axis
Solution: - Y
The region bounded by the curves shown in figure
Required area
3 3 1 1 X’ X
A = ∫0 f(x)dx = ∫0 xdx = [x 2 ]30 = [9 − 0] O
2 2
9
A = 2 sq units x=3
Y’
1
2 )2
4. Find area of the region bounded by the curve y = (16 − x above x –
axis.

Solution: -
1
Given equation can be written as y = (16 − x 2 )2 = x2 + y2 = 16
which represent a circle of radius 4 and centre at origin.
The region bounded by the curves shown in figure Y
Required area
4 4
A = 2 ∫0 f(x)dx = 2 ∫0 √16 − x 2 dx
X’ X
x 16 x 4 (-4,0) O (4,0)
= 2 [2 √16 − x 2 + sin−1 4]
2 0
= 2[0 + 8 sin−1 1 − 0 − 8 sin−1 0]
8π Y’
= 2[ 2 ] 𝑥 2 + 𝑦 2 = 42
A = 8π sq units

Now try yourself :


5. Find area of region bounded by the curve y2 = 4x, and its latus rectum
above x – axis
Mathematics Page 150 of 260 XII
4
Ans 3 sq units {Hint: - latus rectum of the parabola y2 = 4ax is x = a}
6. Using Integration find area enclosed by the circle x 2 + y2 = 2
Ans 2π sq unit
7. Using Integration find area of the region bounded by curve y = x + 2 and
the lines x = 2, x = 3 and x axis.
9
Ans 2 sq units
8. Find area of the region bounded by curve y = x2 and the line y = 16
256
Ans 3 sq units
9. Find area of the region bounded by the curve y = cosx between x = 0 and x

Ans 2 sq units
10. Find area of the region bounded by the curve y = sinx between the
π
ordinates x = 0, x = 2 and the x-axis
Ans 1 sq units

Long answer type questions (3marks / 5 marks)


1. Find the area of the region lying in the first quadrant and enclosed by the
x – axis, the line y = x and the circle x2 + y2 = 32.
Solution: - Y
The region bounded by the curves shown in figure
(4,4)
Intersecting point of the line y = x and the
circle x2 + y2 = 32 are (4,4) and (-4,-4) X’
O
X
Required area
(-4,-4)

4 √ 4 2
A = ∫0 f(x)dx + ∫4 g(x)dx Y’
4 4 √2 𝑥 2 + 𝑦 2 = 32
= ∫0 xdx + ∫4 √32 − x 2 dx
1 x 32 x 4 √2
= 2 [x 2 ]40 + [2 √32 − x 2 + sin−1 4 2]
2 √ 4
1 −1 4 4
= 2 [16 − 0] + [0 + 16 sin 1 − 2 √32 − 16 − 16 sin−1 4 2]

16π 16π
= 8+[ 2 −8− 4 ]
A = 4π sq unit
2. Using integration find the area of the region : {(x, y) : 0 ≤ y ≤ x 2 , 0 ≤ y ≤ x,
0 ≤ x ≤ 2} 11/6
Mathematics Page 151 of 260 XII
Solution: -
The region bounded by the curves shown in figure
Y
Intersecting point of the line y = x, x = 2 and the (2,2)
x2 =y
Parabola y = x2 are (1,1) and (2,2) (1,1)
Required area
X’ X=1 X=2 X
1 2
O
A= ∫0 f(x)dx + ∫1 g(x)dx
1 2
= ∫0 x 2 dx + ∫1 xdx
x3
1
x2
2 Y’
= [3] + [2]
0 1
1 1
= 3 + [2 − 2]
11
= 6 sq unit
3. Using integration, find the area of the parabola y 2 = 4ax bounded by its
latus rectum. 8a2 /3

Solution: -
We know that latus rectum of parabola y2 = 4ax is x = a
The region bounded by the curves shown in figure
Required area
a Y
A = 2 ∫0 f(x)dx
a
= 2 ∫0 √4𝑎𝑥dx
3 a
X’ X
2x2 O x=a
= 4 √𝑎 [ ]
3
0 y2 = 4ax
3
8
= √𝑎 [𝑎 − 0] 2 Y’
3
8 𝑎2
= sq unit
3

4. Using integration, find the area of the region bounded by the lines x – y =
0, 3x – y = 0 and
x+ y = 12. 18

Solution: -
The region bounded by the curves shown in figure
Mathematics Page 152 of 260 XII
Intersecting point of the line y = x, y = 3x and
x+y = 12 are (0,0), (3,9) and (6,6)
Required area
Y
3 6 6
A= ∫0 f(x)dx + ∫3 g(x)dx − ∫0 h(x)dx (3,9)
(6,6)
3 6 6 O
= ∫0 3xdx + ∫3 (12 − x)dx − ∫0 xdx X’ x=3 x=6 X
3 6 6
x2 x2 x2
= 3 [ 2 ] + [12𝑥 − 2 ] − [ 2 ]
0 3 0
Y’
27 9
= + [72 − 18 − 36 + 2] − 18
2

= 18 sq unit

5. Using integration, find the smaller area enclosed by the circle x 2 + y2 = 4


and the line x + y = 2.

Solution: -
The region bounded by the curves shown in figure Y
Required area

2 2 (2,0)
A = ∫0 f(x)dx − ∫0 g(x)dx X’ X
O

2 2
A = ∫0 √4 − x 2 dx − ∫0 (2 − x)dx
Y’
2 2
x 4 −1 x] 𝑥2
= [2 √4 − x 2 + sin − [2x − ]
2 2 0 2 0

= [0 + 2 sin−1 1 − 0 − 2 sin−1 0] − [4 − 2 − 0 + 0]
A = (π − 2) sq units
6. Using integration, find the area of the greatest rectangle that can be
𝑥2 𝑦2
inscribed in an ellipse + =1
𝑎2 𝑏2

Solution: -
The region bounded by the curves shown in figure

Mathematics Page 153 of 260 XII


Now try yourself
7. Using integration, find the area of a triangle whose vertices are (1, 0), (2,
2) and (3, 1).
8. Using integration, find the area of the region bounded by the triangle
whose vertices are (2, – 2), (4, 5) and (6, 2).
9. Using integration, find the area of the region {(x, y) : x 2 + y2 ≤ 9, x + y ≥ 3}.

Mathematics Page 154 of 260 XII


10. Using integration, find the area of the region :{(x, y) : 9x2 + 4y2 ≤ 36, 3x +
2y ≥ 6}
11. Using integration, find the area of the region enclosed by the parabola y
= 3x2 and the
line 3x – y + 6 = 0.
12. Using integration, find the area of the region bounded by the lines 2x + y
= 4, 3x – 2y = 6 and
x – 3y + 5 = 0.
13. Using integration, find the area of triangle ABC bounded by the lines 4x –
y + 5 = 0, x + y – 5 = 0 and x – 4y + 5 = 0
14. Using the method of integration, find the area of the region bounded by
the lines 3x – 2y + 1 = 0, 2x + 3y – 21 = 0 and x – 5y + 9 = 0.
15. Using integration, find the area of the smaller region bounded by the
ellipse x2 / 9 + y2 / 4 = 1 and the line x/3 + x/2 = 1.
16. Using integration, find the area of the region bounded by the line y = 3x
+ 2, the x-axis and the ordinates x = – 2 and x = 1.
17. Find the area of the following region using integration :{(x, y) : y ≤|x| + 2,
y ≥ x 2}
18. Find the area of the region {(x, y) : 0 ≤ y ≤ x2, 0 ≤ y ≤ x + 2, – 1 ≤ x ≤ 3}.
19. If the area between the curves x = y2 and x = 4 is divided into two equal
parts by the line x = a, then find the value of a using integration.

Answer
3 9
7. 2 sq unit 8. 10 sq unit 9. (π − 2) sq unit
4

10. 2
− 3 sq unit
27 7 15
11. sq unit 12. 2 sq unit 13. sq unit
2 2
13
14. sq unit
2
3π 41 20
15. − 3 sq unit 16. sq unit 17. sq unit
2 6 3
15
18. sq unit
2
2
19. a = 4 sq unit
3

Mathematics Page 155 of 260 XII


CHAPTER -9
DIFFERENTIAL EQUATION
DEFINITION: An equation involving independent variable x, dependent
variable y and derivatives of dependent variable with respect to
𝑑𝑦 𝑑2 𝑦
independent variable , 𝑑𝑥 2 , etc is called the differential equation. For
𝑑𝑥
𝑑𝑦 𝑑2 𝑦 𝑑𝑦
example, 2𝑥 𝑑𝑥 − 3𝑦 = 5 and 4 𝑑𝑥 2 + (𝑑𝑥 )3 = 0 are differential equations.
ORDER OF DIFFERENTIAL EQUATION: The order of the differential equation
is the order of the highest order derivative of dependent variable w.r.t.
independent variable occurring in the differential equation. For example, in
𝑑2 𝑦 𝑑3 𝑦 𝑑𝑦
the differential equation 𝑥 2 𝑦 𝑑𝑥 2 + 𝑥 𝑑𝑥 3 − 𝑦 𝑑𝑥 = 0
𝑑3 𝑦
The highest order derivative is and its order is 3, therefore the order of
𝑑𝑥 3
differential equation is 3
NOTE: The order of differential equation is always a positive integer
DEGREE OF DIFFERENTIAL EQUATION: The highest power of the highest
order derivative involved in a differential equation when it is written as a
polynomial in derivatives and the derivatives are free from radicals and
fractions, is called the degree of differential equation
NOTE: 1.The degree of differential equation is always a positive integer
2. The degree of differential equation is not defined when it is not a
polynomial equation in derivatives
EXAMPLE: Determine the order and degree of the differential equation
𝑑𝑦 𝑑2 𝑦
[1 + (𝑑𝑥 )2 ]3/2 = 5 𝑑𝑥 2
Since this equation is not free from radicals
∴ Squaring on both sides
𝑑𝑦 𝑑2𝑦
[1 + (𝑑𝑥 )2 ]3 = 25(𝑑𝑥 2 )2
𝑑𝑦 𝑑𝑦 𝑑𝑦 𝑑2 𝑦
1+ (𝑑𝑥 )6 + 3(𝑑𝑥 )2 + 3(𝑑𝑥 )4 = 25(𝑑𝑥 2 )2
𝑑2 𝑦
The highest order derivative in the equation is and its order is 2
𝑑𝑥 2
therefore the order of deferential equation is 2
Mathematics Page 156 of 260 XII
The highest power of the highest order derivative is 2 therefore the degree
of differential equation is 2
EXAMPLE: Determine the order and degree of the differential equation
𝑑2 𝑦 𝑑𝑦
sin(𝑑𝑥 2 ) + 𝑑𝑥 = 9
𝑑2 𝑦 𝑑𝑦
sin(𝑑𝑥 2 ) + 𝑑𝑥 = 9
𝑑2 𝑦 𝑑𝑦
⇒ 𝑠𝑖𝑛 ( ) + = 9 which is not a polynomial equation in
𝑑𝑥 2 𝑑𝑥
derivatives, as it is not finite, so its degree is not defined
𝑑2 𝑦
The highest order derivative in the equation is and its order is 2
𝑑𝑥 2
therefore the order of differential equation is 2
LINEAR DIFFERENTIAL EQUATION: A differential equation is said to be
linear, if the dependent variable and its derivatives occur only in the first
degree and are not multiplied together.
In general, a linear differential equation of order n is of the form
𝑑𝑛 𝑦 𝑑𝑛−1𝑦 𝑑𝑛−2 𝑦 𝑑𝑦
𝑃0 + 𝑃1 + 𝑃2 + − − − − +𝑃𝑛−1 + 𝑃𝑛 𝑦 = 𝑄
𝑑𝑥 𝑛 𝑑𝑥 𝑛−1 𝑑𝑥 𝑛−2 𝑑𝑥
Whether 𝑃0 , 𝑃1 , 𝑃2 − − − − − 𝑃𝑛−1 , 𝑃𝑛 , 𝑄 are function of x or constants
𝑑𝑦 𝑑3 𝑦 y
The differential equations 𝑐𝑜𝑠𝑥 𝑑𝑥 + 𝑦 𝑠𝑖𝑛𝑥 = 1 𝑎𝑛𝑑 + x = x 2 logx are
𝑑𝑥 3
linear differential equations
𝑑𝑦
The differential equation 𝑦 𝑑𝑥 + 𝑥 = 4 is non linear differential equation,
𝑑𝑦
because the dependent variable 𝑦 and its derivative are multiplied
𝑑𝑥
together
GENERAL SOLUTION OF DIFFERENTIAL EQUATION: A solution of a
differential equation is called the general solution, if it contains as many
arbitrary constants as the order of the differential equation
PARTICULAR SOLUTION OF DIFFERENTIAL EQUATION: A solution obtained
by giving particular values to arbitrary constants in the general solution of a
differential equation is called particular solution. Particular solution is free
from arbitrary constants
METHODS FOR SOLVING THE DIFFERENTIAL EQUATIONS:

Mathematics Page 157 of 260 XII


1. Variable Separable Method:
𝑑𝑦
If = 𝑓(𝑥 ), 𝑡ℎ𝑒𝑛 𝑑𝑦 = 𝑓 (𝑥 ) . 𝑑𝑥 , ∴ ∫ 𝑑𝑦 = ∫ 𝑓(𝑥 ) 𝑑𝑥 + 𝑐 ,
𝑑𝑥
represent the general solution of D.E
𝑑𝑦 𝑑𝑦 𝑑𝑦
If = 𝑓(𝑦), 𝑡ℎ𝑒𝑛 = 𝑑𝑥 , ∴ ∫ 𝑓(𝑦) = ∫ 𝑑𝑥 + c, represent
𝑑𝑥 𝑓(𝑦)
the general solution of D.E
𝑑𝑦 𝑑𝑦 𝑑𝑦
If = 𝑓(𝑥 ). 𝑓 (𝑦), 𝑡ℎ𝑒𝑛 = 𝑓(𝑥) 𝑑𝑥 , ∴ ∫ = ∫ 𝑓(𝑥)𝑑𝑥 + c,
𝑑𝑥 𝑓(𝑦) 𝑓(𝑦)
represent the general solution of
EXAMPLE: Find the general solution of the following differential equation:
𝑑𝑦
= 𝑒 𝑥−𝑦 + 𝑥 3 𝑒 −𝑦
𝑑𝑥
𝑑𝑦
SOLUTION: 𝑑𝑥 = 𝑒 𝑥 𝑒 −𝑦 + 𝑥 3 𝑒 −𝑦
𝑑𝑦 𝑑𝑦
⇒ = (𝑒 𝑥 + 𝑥 3 )𝑒 −𝑦 [ = 𝑓 (𝑥 ). 𝑓 (𝑦 ) ]
𝑑𝑥 𝑑𝑥
𝑑𝑦
⇒ = (𝑒 𝑥 + 𝑥 3 ) 𝑑𝑥 [Separating the variables]
𝑒 −𝑦
𝑑𝑦
∴∫ = ∫(𝑒 𝑥 + 𝑥 3 ) 𝑑𝑥
𝑒 −𝑦
𝑥4 𝒙𝟒
⇒ 𝑒𝑦 = 𝑒 𝑥 + + 𝑐 ⇒ 𝒆𝒚 − 𝒆𝒙 − =𝒄
4 𝟒

2. Homogeneous Differential Equation:


Homogeneous Function: A function F(x, y) is a homogeneous function of
degree n if
𝑦 𝑥
F(x, y) = 𝑥 𝑛 𝑔 (𝑥 ) OR F(x, y) = 𝑦 𝑛 ℎ (𝑦) OR F (λx, λy) =λ𝑛 F(x,
y)
𝑦
Example: F(x, y) = 𝑐𝑜𝑠 𝑥 is a homogeneous function of degree zero
λ𝑦
F (λx, λy) = 𝑐𝑜𝑠 λ𝑥
𝑦
= λ0 𝑐𝑜𝑠 𝑥
= λ0 F(x, y)
Homogeneous Differential Equation: A differential equation of the form
𝑑𝑦
= F(x, y) is said to be homogeneous if F(x, y) is a homogeneous function
𝑑𝑥
of degree zero or F (λx, λy) =λ0 F(x, y)
Mathematics Page 158 of 260 XII
𝑑𝑦 𝑦
Example: x (𝑑𝑥 ) − 𝑦 + 𝑥𝑠𝑖𝑛( 𝑥 ) = 0 is homogeneous differential equation
𝑦
𝑑𝑦 𝑦−𝑥𝑠𝑖𝑛( )
𝑥
=
𝑑𝑥 𝑥
𝑦
𝑦−𝑥𝑠𝑖𝑛( )
𝑥
Let F(x, y) = 𝑥
λ𝑦
λ𝑦−λ𝑥𝑠𝑖𝑛( )
λ𝑥
F (λx, λy) = λ𝑥
𝑦
𝑦−𝑥𝑠𝑖𝑛( )
0 𝑥
= λ [ ]
𝑥
= λ0 F(x, y)
∴ 𝑔𝑖𝑣𝑒𝑛 𝑒𝑞𝑢𝑎𝑡𝑖𝑜𝑛 𝑖𝑠 𝑎 ℎ𝑜𝑚𝑜𝑔𝑒𝑛𝑒𝑜𝑢𝑠 𝑑𝑖𝑓𝑓𝑒𝑟𝑒𝑛𝑡𝑖𝑎𝑙 𝑒𝑞𝑢𝑎𝑡𝑖𝑜𝑛
𝒅𝒚 𝒚
STEPS FOR SOLVING HOMOGENEOUS DIFFERENTIAL EQUATION: = 𝝋( )
𝒅𝒙 𝒙
𝑑𝑦 𝑑𝑣
Step I Put y = 𝑣𝑥 and differentiate it w.r.t x to get 𝑑𝑥 = 𝑣 + 𝑥 𝑑𝑥
𝑑𝑦
Ste II Put the value of y and in the given differential equation and
𝑑𝑥
separate the variable v and x and integrate both sides
Step III Replace the value of v by y/x
This gives the general solution of differential equation
𝒅𝒙
STEPS FOR SOLVING HOMOGENEOUS DIFFERENTIAL EQUATION: =
𝒅𝒚
𝒙
𝝋( 𝒚 )
𝑑𝑥 𝑑𝑣
Step I Put x = 𝑣𝑦 and differentiate it w.r.t y to get 𝑑𝑦 = 𝑣 + 𝑦 𝑑𝑦
𝑑𝑥
Ste II Put the value of x and in the given differential equation and
𝑑𝑦
separate the variable v and y and integrate both sides
Step III Replace the value of v by x/y
This gives the general solution of differential equation

𝑦
EXAMPLE: Solve the differential equation: 𝑦 𝑑𝑥 + 𝑥𝑙𝑜𝑔 ( 𝑥 ) 𝑑𝑦 − 2𝑥 𝑑𝑦 =
0
𝑦
𝑦𝑑𝑥 = 2𝑥 𝑑𝑦 − 𝑥𝑙𝑜𝑔 ( 𝑥 ) 𝑑𝑦
𝑦
𝑦𝑑𝑥 = 𝑥[2 − 𝑙𝑜𝑔 ( 𝑥 )]𝑑𝑦
Mathematics Page 159 of 260 XII
𝑑𝑦 𝑦/𝑥 𝑦
= 𝑦 = 𝜑(𝑥 )
𝑑𝑥 [2 − 𝑙𝑜𝑔( )]
𝑥

∴ 𝑔𝑖𝑣𝑒𝑛 𝑒𝑞𝑢𝑎𝑡𝑖𝑜𝑛 𝑖𝑠 𝑎 ℎ𝑜𝑚𝑜𝑔𝑒𝑛𝑒𝑜𝑢𝑠 𝑑𝑖𝑓𝑓𝑒𝑟𝑒𝑛𝑡𝑖𝑎𝑙 𝑒𝑞𝑢𝑎𝑡𝑖𝑜𝑛


𝑑𝑦 𝑑𝑣
Let 𝑦 = 𝑣𝑥 ⇒ 𝑑𝑥 = 𝑣 + 𝑥 𝑑𝑥
𝑑𝑣 𝑣
∴ 𝑣+𝑥 =
𝑑𝑥 2 − 𝑙𝑜𝑔𝑣
𝑑𝑣 𝑣
⇒𝑥 = −𝑣
𝑑𝑥 2−𝑙𝑜𝑔𝑣
𝑑𝑣 𝑣𝑙𝑜𝑔𝑣−𝑣
⇒ 𝑥 𝑑𝑥 = 2−𝑙𝑜𝑔𝑣
2 − 𝑙𝑜𝑔𝑣 𝑑𝑥
⇒ 𝑑𝑣 =
𝑣(𝑙𝑜𝑔𝑣 − 1) 𝑥
2−𝑙𝑜𝑔𝑣 𝑑𝑥 1−(𝑙𝑜𝑔𝑣−1) 𝑑𝑥
⇒∫ 𝑣(𝑙𝑜𝑔𝑣−1) 𝑑𝑣 = ∫ ⇒ ∫ 𝑑𝑣 = ∫
𝑥 𝑣(𝑙𝑜𝑔𝑣−1) 𝑥
1 1 𝑑𝑥
⇒∫ 𝑣(𝑙𝑜𝑔𝑣−1) − 𝑣 𝑑𝑣 = ∫ 𝑥
⇒ log|(𝑙𝑜𝑔𝑣 − 1)| − log |𝑣| = log |𝑥| + 𝑙𝑜𝑔𝑐
𝑙𝑜𝑔𝑣−1
⇒log | | = log| 𝑐𝑥 |
𝑣
𝑙𝑜𝑔𝑣 − 1
⇒ = 𝑐𝑥
𝑣
𝑦
log( )−1 𝑦
𝑥
⇒ 𝑦 = 𝑐𝑥 ⇒ log (𝑥 ) − 1 = 𝑐𝑦
𝑥

3. LINEAR DIFFERENTIAL EQUATION OF FIRST ORDER:


Two types of linear differential equation of first order as given below:
𝑑𝑦
Type I A differential equation of the form 𝑑𝑥 + 𝑃𝑦 = 𝑄,where P and Q are
constants or function of x only
Integrating Factor (I.F.) = 𝑒 ∫ 𝑃 𝑑𝑥
Solution: 𝑦. 𝐼. 𝐹 = ∫(𝑄 × 𝐼. 𝐹) 𝑑𝑥 + C
𝑑𝑥
Type II A differential equation of the form 𝑑𝑦 + 𝑃𝑥 = 𝑄,whereP and Q are
constants or function of y only
Integrating Factor (I.F.) = 𝑒 ∫ 𝑃 𝑑𝑦
Solution: 𝑥. 𝐼. 𝐹 = ∫(𝑄 × 𝐼. 𝐹) 𝑑𝑦 + C
𝑑𝑦
EXAMPLE: Find the general solution of the D.E 𝑥 𝑑𝑥 − y = 𝑥 3
Mathematics Page 160 of 260 XII
𝑑𝑦
Solution: 𝑥 𝑑𝑥 − y = 𝑥 3
𝑑𝑦 𝑦 𝑑𝑦
⇒ 𝑑𝑥 − 𝑥 = 𝑥 2 [linear differential equation of first order of type I, +
𝑑𝑥
𝑃𝑦 = 𝑄]
1
𝑃 = − 𝑋 , 𝑄 = 𝑥2
1 −1 1
I.F. = 𝑒 − ∫𝑥 𝑑𝑥 = 𝑒 −𝑙𝑜𝑔𝑥 = 𝑒 𝑙𝑜𝑔𝑥 = 𝑥 −1 = 𝑥
1 1
General solution: 𝑦. 𝐼. 𝐹 = ∫(𝑄 × 𝐼. 𝐹) 𝑑𝑥 + C ⇒ 𝑦. 𝑥 = ∫(𝑥 2 × 𝑥) 𝑑𝑥 +C
𝑦 𝑥2 𝑥3
= +𝐶 ⇒ 𝑦 = + 𝐶𝑥
𝑥 2 2
EXAMPLE: Find the general solution of D.E 𝑦𝑒 𝑦 𝑑𝑥 = (𝑦 3 + 2𝑥𝑒 𝑦 )𝑑𝑦
Solution: 𝑦𝑒 𝑦 𝑑𝑥 = (𝑦 3 + 2𝑥𝑒 𝑦 )𝑑𝑦
𝑑𝑥 (𝑦 3+2𝑥𝑒 𝑦 )
⇒ 𝑑𝑦 = 𝑦𝑒 𝑦
𝑑𝑥 2𝑥
⇒ 𝑑𝑦 = 𝑦 2 𝑒 −𝑦 + 𝑦
𝑑𝑥 2
⇒ − 𝑥 = 𝑦 2 𝑒 −𝑦 [linear differential equation of first order of type II,
𝑑𝑦 𝑦
𝑑𝑥
[𝑑𝑦 + 𝑃𝑥 = 𝑄]
2
𝑃 = − 𝑦 , Q = 𝑦 2 𝑒 −𝑦
2
− ∫ 𝑑𝑦 −2 1
I.F = 𝑒 𝑦 = 𝑒 −2𝑙𝑜𝑔𝑦 = 𝑒 𝑙𝑜𝑔𝑦 = 𝑦 −2 =
𝑦2
1
General solution: 𝑥. 𝐼. 𝐹 = ∫(𝑄 × 𝐼. 𝐹) 𝑑𝑦 + C ⇒ 𝑥. 𝑦 2 =
1
∫(𝑦 2 𝑒 −𝑦 × 𝑦 2 )𝑑𝑦 + 𝐶
𝑥
⇒ = ∫ 𝑒 −𝑦 𝑑𝑦 + 𝐶
𝑦2

𝑥
⇒ = −𝑒 −𝑦 + 𝐶 ⇒ 𝑥 +𝑦 2 𝑒 −𝑦 = 𝐶𝑦 2
𝑦2
MULTIPLE CHOICE QUESTIONS:
𝑑 𝑑𝑦
Q.1 The sum of the order and degree of differential equation 𝑑𝑥 {(𝑑𝑥 )3} = 0 is
(a) 4 (b) 5 (c) 3 (d) none of these

Mathematics Page 161 of 260 XII


Q2. The sum of the order and degree of differential equation 𝑥√1 − 𝑦 2 𝑑𝑥 +
𝑦√1 − 𝑥 2 𝑑𝑦 = 0 is
(a) 3 (b) 2 (c) 4 (d) none of these
Q3. The sum of the order and degree of differential equation [1 +
𝑑𝑦 𝑑3 𝑦
(𝑑𝑥 )2 ]3/2 = 5(𝑑𝑥 3 )2
(a) 3 (b) 2 (c) 5 (d) 7
𝑑2 𝑦 𝑑𝑦 𝑑𝑦
Q4. The degree of differential equation (𝑑𝑥 2 )2 + (𝑑𝑥 )3 = 𝑥𝑠𝑖𝑛(𝑑𝑥 ) is
(a) 2 (b) 1 (c) 3 (d) not defined
(5)
𝑑𝑦 3
Q5. The degree of differential equation 𝑥 + 𝑑𝑦 = 𝑦 2 is
𝑑𝑥
𝑑𝑥

(a) 1 (b) 2 (c) 3 (d) not defined


𝑑𝑦
Q6. The general solution of the differential equation 𝑑𝑥 = 𝑒 𝑥+𝑦 is
(a) 𝑒 𝑥 + 𝑒 −𝑦 = 𝑐 (b) 𝑒 −𝑥 + 𝑒 𝑦 = 𝑐 ( c) 𝑒 −𝑥 + 𝑒 −𝑦 = 𝑐
(d) 𝑒 𝑥 + 𝑒 𝑦 = 𝑐
𝑑𝑦
Q7. The general solution of the differential equation 𝑥 𝑑𝑥 + 2𝑦 = 𝑥 2 is
𝑥 2+𝑐 𝑥2 𝑥 4+𝑐
(a) 𝑦 = (b) 𝑦 = +𝑐 (c) y = (d) y =
4𝑥 2 4 𝑥2
𝑥2 𝑐
+ 𝑥2
4
Q8. The number of arbitrary constants in the particular solution of
differential equation of third order is
(a) 1 (b) 2 (c) 3 (d) 0
𝑑𝑦
Q9. The general solution of the differential equation log (𝑑𝑥 ) = 𝑎𝑥 + 𝑏𝑦 is
𝑒 −𝑏𝑦 𝑒 𝑎𝑥 𝑒 −𝑏𝑦 𝑒 −𝑎𝑥 𝑒 𝑏𝑦 𝑒 𝑎𝑥
(a) = +𝑐 (b) = +𝑐 ( c) = +𝑐
−𝑏 𝑎 𝑏 𝑎 𝑏 𝑎
𝑒 𝑏𝑦 𝑒 −𝑎𝑥
(d) = +𝑐
𝑏 −𝑎
𝑑𝑦
Q10. The general solution of the differential equation + 𝑦 𝑡𝑎𝑛𝑥 = 𝑠𝑒𝑐𝑥
𝑑𝑥
is
(a) 𝑦 = 𝑐𝑜𝑠𝑥 + 𝑐𝑠𝑖𝑛𝑥 (b) 𝑦 = 𝑠𝑖𝑛𝑥 + 𝑐𝑐𝑜𝑠𝑥 (c ) 𝑦 = 𝑡𝑎𝑛𝑥 + 𝑐𝑐𝑜𝑡𝑥 (d)
𝑦 = 𝑐𝑜𝑡𝑥 + 𝑐𝑡𝑎𝑛𝑥
Mathematics Page 162 of 260 XII
𝑑𝑦
Q11. The integrating factor of differential equation (𝑥𝑙𝑜𝑔𝑥) + 𝑦 =
𝑑𝑥
2𝑙𝑜𝑔𝑥 is
(a) 𝑒 𝑥 (b) 𝑙𝑜𝑔𝑥 (c) log(𝑙𝑜𝑔𝑥) (d) x
𝑑𝑦
Q12. The general solution of the differential equation 𝑑𝑥 + 𝑎𝑦 = 𝑒 𝑚𝑥 is
(a) (𝑎 + 𝑚)𝑦 = 𝑒 𝑚𝑥 + 𝑐 (b) 𝑦𝑒 𝑎𝑥 =
𝑚𝑒 𝑚𝑥 + 𝑐
(c) 𝑦 = 𝑒 𝑚𝑥 + 𝑐𝑒 −𝑎𝑥 (d) 𝑦(𝑎 + 𝑚) =
𝑒 𝑚𝑥 + 𝑐𝑒 −𝑎𝑥
𝑑𝑦
Q13. The general solution of the differential equation 𝑑𝑥 = 𝑒 𝑦+𝑥 + 𝑒 𝑦−𝑥 is
(a) −𝑒 −𝑦 = 𝑒 𝑥 − 𝑒 −𝑥 + 𝑐 (b) 𝑦=
𝑒 𝑥 (𝑥 + 1) + 1 + 𝑐
(c) 𝑦 = 𝑒 𝑥 (𝑥 − 1) + 1 + 𝑐 (d) none of these
𝑑𝑦
Q14. The integrating factor of differential equation (1 − 𝑥 2 ) 𝑑𝑥 − 𝑥𝑦 = 1 is
𝑥
(a) −𝑥 (b) 1+𝑥 2
(c) √1 − 𝑥 2
1
(d) 2 log(1 − 𝑥 2 )
𝑑𝑥 𝑑𝑦
Q15. The general solution of the differential equation + = 0 is
𝑥 𝑦
1 1
(a) 𝑥 + 𝑦 = 𝑐 (b) 𝑥𝑦 = 𝑐 (c ) 𝑙𝑜𝑔𝑥 𝑙𝑜𝑔𝑦 = 𝑐
(d) 𝑥 + 𝑦 = 𝑐

𝑒 −2√𝑥 𝑦 𝑑𝑥
Q16. The integrating factor of differential equation [ − ] = 1 is
√𝑥 √ 𝑥 𝑑𝑦

(a) 𝑒 2√𝑥 (b) 𝑒 −2√𝑥


(c ) 𝑒 √𝑥 (d)
𝑒 −√𝑥
Q17. The integrating factor of differential equation 𝑦𝑑𝑥 − 𝑥𝑑𝑦 + 𝑙𝑜𝑔𝑥 𝑑𝑥 =
0 is
1
(a) 𝑥 (b) 𝑥 ( c) – 𝑥 (d) log x

𝑑2 𝑦 𝑑𝑦 𝑑2 𝑦
Q18. The degree of differential equation 𝑑𝑥 2 + 3(𝑑𝑥 )2 = 𝑥 2 log( 𝑑𝑥 2 ) is

Mathematics Page 163 of 260 XII


(a) 1 (b) 2 (c) 3 (d) not
defined
𝑑𝑦
Q19. The general solution of the differential equation (𝑒 𝑥 + 𝑒 −𝑥 ) 𝑑𝑥 =
(𝑒 𝑥 − 𝑒 −𝑥 ) is
(a) 𝑦 = log|𝑒 𝑥 + 𝑒 −𝑥 | + 𝑐 (b) 𝑦 =
log|𝑒 𝑥 − 𝑒 −𝑥 | + 𝑐
(c) 𝑦 = log |𝑒 𝑥 | + 𝑐 (d) 𝑦 = log |𝑒 −𝑥 | +
𝑐
𝑑𝑦
Q20. The general solution of the differential equation (𝑥 + 2𝑦 2 ) 𝑑𝑥 = 𝑦 is
1 1
(a) 𝑦 (b) (c ) logy (d) 𝑥
𝑦

ANSWER
𝑑
𝑑𝑦 𝑑𝑦 𝑑2 𝑦
Q1.(𝑐) {( )3 } =0⇒ 3(𝑑𝑥 )2 . 𝑑𝑥 2 = 0 , Order =2, degree =1 sum =
𝑑𝑥 𝑑𝑥
2+1 = 3
dy 𝑥√1−𝑦 2
Q2. (𝑏) 𝑥√1 − 𝑦 2 𝑑𝑥 + 𝑦√1 − 𝑥 2 𝑑𝑦 = 0 ⇒ =− , Order = 1,
dx 𝑦√1−𝑥 2
degree = 1 sum = 2
𝑑𝑦 𝑑3 𝑦 𝑑𝑦 𝑑3 𝑦
Q3. (𝑑 ) [1 + (𝑑𝑥 )2 ]3/2 = 5(𝑑𝑥 3 )2 ⇒ [1 + (𝑑𝑥 )2 ]3 = 25(𝑑𝑥 3 )4, Order = 3,
degree = 4 sum = 7
𝑑2𝑦 𝑑𝑦 𝑑𝑦
Q4. (𝑑 ) (𝑑𝑥 2 )2 + (𝑑𝑥 )3 = 𝑥𝑠𝑖𝑛(𝑑𝑥 ) Since it is not a polynomial equation
in derivative, so degree is not defined
𝑑𝑦 3 𝑑𝑦 𝑑𝑦
Q5. (b) 𝑥 + 𝑑𝑦 = 𝑦 2 ⇒ 𝑥( )2 + 3 = 𝑦 2 , degree = 2
𝑑𝑥 𝑑𝑥 𝑑𝑥
𝑑𝑥
𝑑𝑦 𝑑𝑦 dy dy
Q6.(𝑎) = 𝑒 𝑥+𝑦 ⇒ 𝑑𝑥 = 𝑒 𝑥 . 𝑒 𝑦 ⇒ 𝑒 𝑦 = ex 𝑑𝑥 ⇒ ∫ 𝑒 𝑦 = ∫ ex 𝑑𝑥 ⇒
𝑑𝑥
−𝑒 −𝑦 = ex + c ⇒ 𝑒 𝑥 + 𝑒 −𝑦 = c
2
𝑑𝑦 𝑑𝑦 2𝑦
Q7. (𝑑 ) 𝑥 + 2𝑦 = 𝑥 2 ⇒ + = 𝑥, I.F = 𝑒 ∫𝑥𝑑𝑥 = 𝑒 2𝑙𝑜𝑔𝑥 =
𝑑𝑥 𝑑𝑥 𝑥
𝑙𝑜𝑔𝑥 2 2
𝑒 =𝑥
x4 x2 c
Solution: 𝑦𝑥 2 = ∫ 𝑥. 𝑥 2 𝑑𝑥 ⇒ x 2 y = +c⇒y= + x2
4 4
Q8. (c) Number of arbitrary constants =3
Mathematics Page 164 of 260 XII
𝑑𝑦 dy dy dy
Q9. (a) log (𝑑𝑥 ) = 𝑎𝑥 + 𝑏𝑦 ⇒ dx = eax+by ⇒ dx = eax . eby ⇒ eby =
dy 𝑒 −𝑏𝑦 𝑒 𝑎𝑥
eax dx ⇒ ∫ by = ∫ eax dx ⇒ = +𝑐
e −𝑏 𝑎
𝑑𝑦 ∫ 𝑡𝑎𝑛𝑥 𝑑𝑥
Q10. (b) + 𝑦 𝑡𝑎𝑛𝑥 = 𝑠𝑒𝑐𝑥 I.F = 𝑒 = 𝑒 log( 𝑠𝑒𝑐𝑥) = 𝑠𝑒𝑐𝑥
𝑑𝑥
Solution: 𝑦. 𝑠𝑒𝑐𝑥 = ∫ 𝑠𝑒𝑐 2 𝑥 𝑑𝑥 ⇒ 𝑦. 𝑠𝑒𝑐𝑥 = 𝑡𝑎𝑛𝑥 + 𝑐 ⇒ y =
sinx + ccosx
1
𝑑𝑦 𝑑𝑦 1 2 ∫𝑥𝑙𝑜𝑔𝑥 𝑑𝑥
Q11.(𝑏) (𝑥𝑙𝑜𝑔𝑥 ) + 𝑦 = 2𝑙𝑜𝑔𝑥 ⇒ = 𝑦 = 𝑥 2, I.F. =𝑒 =
𝑑𝑥 𝑑𝑥 𝑥𝑙𝑜𝑔𝑥
𝑒 log(𝑙𝑜𝑔𝑥) = 𝑙𝑜𝑔𝑥
𝑑𝑦
Q12 (𝑑 ) + 𝑎𝑦 = 𝑒 𝑚𝑥 I.F. =𝑒 ∫ 𝑎𝑑𝑥 = 𝑒 𝑎𝑥 , Solution: 𝑦𝑒 𝑎𝑥 =
𝑑𝑥
∫ 𝑒 𝑎𝑥 . 𝑒 𝑚𝑥
𝑑𝑥 ⇒
𝑒 (𝑎+𝑚)𝑥
𝑦𝑒 𝑎𝑥 = ∫ 𝑒 (𝑎+𝑚)𝑥 𝑑𝑥 ⇒ 𝑦𝑒 𝑎𝑥 = + 𝑐 ⇒ y(a + m) =
(𝑎+𝑚)
emx + ce−ax
(7)
𝑑𝑦 𝑑𝑦 𝑑𝑦
Q13. (𝑎) = 𝑒 𝑦+𝑥 + 𝑒 𝑦−𝑥 ⇒ 𝑑𝑥 = 𝑒 𝑦 𝑒 𝑥 + 𝑒 𝑦 𝑒 −𝑥 ⇒ 𝑑𝑥 = 𝑒 𝑦 (𝑒 𝑥 + 𝑒 −𝑥 )
𝑑𝑥
dy dy
⇒ ey = (𝑒 𝑥 + 𝑒 −𝑥 )𝑑𝑥 ⇒ ∫ ey = ∫(𝑒 𝑥 + 𝑒 −𝑥 )𝑑𝑥 ⇒ −𝑒 −𝑦 = 𝑒 𝑥 −
𝑒 −𝑥 + 𝑐
x
𝑑𝑦 dy x 1 −∫ 𝑑𝑥
Q14. (𝑐 ) (1 − 𝑥 2 ) − 𝑥𝑦 = 1 ⇒ − 2y = 2 , I.F. = 𝑒
1−x2 =
𝑑𝑥 dx 1−x 1−x
1
log |1−𝑥 2 | 2)1/2
𝑒 2 = 𝑒 log(1−𝑥 = √1 − 𝑥 2
𝑑𝑥 𝑑𝑦 dy y 𝑑𝑦 𝑑𝑥 𝑑𝑦 𝑑𝑥
Q15 (𝑏) 𝑥
+ 𝑦
= 0 ⇒ dx = − x ⇒ 𝑦
=− 𝑥
⇒∫ 𝑦
= −∫ 𝑥
⇒ logy =
−logx + logc
⇒ log y + log x = log c ⇒ log xy = log c ⇒ xy = c
1
𝑒 −2√𝑥 𝑦 𝑑𝑥 dy 𝑦 𝑒 −2√𝑥 ∫ 𝑑𝑥
Q16 (𝑎) [ − ] = 1 ⇒ dx + = , I.F. = 𝑒 √𝑥 = 𝑒 2√𝑥
√𝑥 √ 𝑥 𝑑𝑦 √𝑥 √𝑥
𝑑𝑦 dy y
Q17. (b) 𝑦𝑑𝑥 − 𝑥𝑑𝑦 + 𝑙𝑜𝑔𝑥 𝑑𝑥 = 0 ⇒ 𝑦 − 𝑥 𝑑𝑥 + log 𝑥 = 0 ⇒ dx − x =
log x
x
1 1
1
I.F. = 𝑒 − ∫𝑥𝑑𝑥 = 𝑒 − log 𝑥 = 𝑒 𝑙𝑜𝑔𝑥 = 𝑥

Mathematics Page 165 of 260 XII


𝑑2 𝑦 𝑑𝑦 𝑑2 𝑦
Q18.(d) + 3(𝑑𝑥 )2 = 𝑥 2 log( 𝑑𝑥 2), It is not a polynomial equation in
𝑑𝑥 2
derivative so degree is not defined
𝑑𝑦 (𝑒 𝑥 −𝑒 −𝑥 )
Q19.(a) (𝑒 𝑥 + 𝑒 −𝑥 ) 𝑑𝑥 = (𝑒 𝑥 − 𝑒 −𝑥 ) ⇒ dy = (𝑒 𝑥 +𝑒 −𝑥 ) dx ⇒ ∫ dy =
(𝑒 𝑥 −𝑒 −𝑥 )
∫ (𝑒 𝑥 +𝑒 −𝑥 ) dx
⇒ y = log |(𝑒 𝑥 + 𝑒 −𝑥 )| + 𝑐
1 1
𝑑𝑦 𝑑𝑥 ∫ 𝑥 − 𝑑𝑦 𝑙𝑜𝑔
Q20.(b) (𝑥 + 2𝑦 2 ) 𝑑𝑥 = 𝑦 ⇒ 𝑑𝑦 − 𝑦 = 2𝑦, I.F. 𝑒 𝑦 = 𝑒 − log 𝑦 = 𝑒 𝑦 =
1
𝑦

ASSERTION –REASON BASED QUESTIONS


In the following questions, a statement of Assertion (A) is followed by a
statement of Reason (R)
Choose the correct answer out of the following choices.
(a) Both A and R is true and R is the correct explanation of A
(b) Both A and R is true and R is not the correct explanation of A
(c) A is true but R is false.
(d) A is false but R is true.
𝑑3 𝑦 𝑑𝑦
Q.1 Assertion (A): The degree of differential equation =1+√ 𝑖𝑠 2
𝑑𝑥 3 𝑑𝑥

Reason (R) : The degree is the highest power of the highest order
derivative involved in a differential equation when it is written as a
polynomial in derivatives and the derivatives are free from radicals and
fractions,
𝑑𝑦
Q2. Assertion (A): The solution of differential equation (𝑥 2 + 1) 𝑑𝑥 +
(𝑦 2 + 1) = 0
𝑖𝑠 𝑡𝑎𝑛 −1 𝑥 + 𝑡𝑎𝑛−1 𝑦 = 𝐶
𝑑3 𝑦
Reason (R) : The order of differential equation + 𝑥 = 0 𝑖𝑠 3
𝑑𝑥 3

Mathematics Page 166 of 260 XII


𝑑𝑥
Q.3 Assertion (A): The integrating factor of differential equation 𝑑𝑥 +
(𝑡𝑎𝑛𝑦)𝑥 = 𝑠𝑒𝑐 2 𝑦 𝑖𝑠 𝑠𝑒𝑐𝑦
𝑑𝑥
Reason (R) : Linear differential equation of the form + 𝑃𝑥 = 𝑄
𝑑𝑥
where P and Q are
Function of y or constant has integrating factor,𝐼𝐹 =
𝑒 ∫ 𝑃 𝑑𝑦

𝑑4 𝑦
Q.4 Assertion (A): The order and degree of differential equation =𝑦+
𝑑𝑥 4
𝑑𝑦
(𝑑𝑥 )4 are
4 and 1 respectively
𝑑𝑦 𝑑𝑦
Reason (R) : Degree of differential equation 𝑑𝑥 = sin (𝑑𝑥 ) is not
defined
Q.5 Assertion (A): The order of differential equation whose solution is 𝑦 =
𝑐1 𝑒 𝑥+𝑐2 + 𝑐3 𝑒 𝑥+𝑐4
Is 4
Reason (R) : Order of differential equation is equal to the number of
independent arbitrary
Constant mentioned in the solution of differential equation
SOLUTION
Q.1 (a) Both A and R is true and R is the correct explanation of A
Q.2 (b) Both A and R is true and R is not the correct explanation of A
Q.3 (a) Both A and R is true and R is the correct explanation of A
Q.4 (b) Both A and R is true and R is not the correct explanation of A
Q.5(d) A is false but R is true.

SHORT ANSWER TYPE QUESTIONS [2 MARKS]


𝑑2 𝑦 𝑑𝑦
Q.1 Verify that 𝑦 = 𝑒 −3𝑥 is a solution of differential equation + 𝑑𝑥 −
𝑑𝑥 2
6𝑦 = 0

Mathematics Page 167 of 260 XII


𝑑𝑦
Q.2 Find the particular solution of the differential equation 𝑑𝑥 = −4𝑥𝑦 2 ,
given that y (0) = 1
𝑑𝑦
Q.3 Find the general solution of the differential equation 𝑑𝑥 = 1 − 𝑥 + 𝑦 −
𝑥𝑦
𝑑𝑦
Q.4 Find the general solution of the differential equation 𝑒 𝑑𝑥 = (𝑥 + 3)
𝑑𝑦
Q.5 Find the particular solution of the differential equation = 𝑦 cot 2𝑥 ,
𝑑𝑥
𝜋
𝑦 ( 4) = 2

SOLUTIONS
dy 𝑑2 𝑦
Q.1 𝑦 = 𝑒 −3𝑥 ⇒ dx = −3e−3x and = 9𝑒 −3𝑥
𝑑𝑥 2
dy 𝑑2 𝑦
On putting the value of y, dx and 𝑑𝑥 2 in L.H.S. of the given equation
9𝑒 −3𝑥 − −3e−3x − 6𝑒 −3𝑥
= 0, hence 𝑦 = 𝑒 −3𝑥 is a solution of differential equation

𝑑𝑦 dy
Q.2 𝑑𝑥 = −4𝑥𝑦 2 ⇒ = −4x dx
y2
dy 1 x2
⇒ ∫ y2 = ∫ −4x dx ⇒ − y = −4 +c
2
y = 1, when x = 0
⇒ −1 = 0 + c ⇒ c = −1
1 1
∴ − y = −2x 2 − 1 ⇒ y = 2x2 +1 is the required solution
𝑑𝑦 𝑑𝑦 𝑑𝑦
Q.3 = 1 − 𝑥 + 𝑦 − 𝑥𝑦 ⇒ = (1 − 𝑥) + 𝑦(1 − 𝑥) ⇒ = (1 − 𝑥 )(1 +
𝑑𝑥 𝑑𝑥 𝑑𝑥
𝑦)
dy dy
⇒ 1+y = (1 − x)dx ⇒ ∫ 1+y = ∫(1 − x)dx
𝑥2
⇒ log |(1 + y)| = 𝑥 − +𝑐
2
𝑑𝑦
dy
Q.4 𝑒 𝑑𝑥 = (𝑥 + 3) ⇒ dx = log(x + 3) ⇒ dy = log(𝑥 + 3) 𝑑𝑥
⇒ ∫ dy = ∫ log(x + 3) dx
1
⇒ y = log(x + 3) . x − ∫ x+3 . x dx
Mathematics Page 168 of 260 XII
(x+3)−3
⇒ y = x log(x + 3) − ∫ dx ⇒ y = x log(x + 3) −
x+3
3
∫ 1 − x+3 dx
⇒ y = x log(x + 3) − x + 3 log|x + 3| + c
𝑑𝑦 dy dy
Q.5 = 𝑦 cot 2𝑥 ⇒ = cot 2x dx ⇒ ∫ = ∫ cot 2x dx
𝑑𝑥 y y
log |sin2x|
⇒ log|y| = +c
2
𝜋
𝑦 = 2 𝑤ℎ𝑒𝑛 𝑥 = 4
log1
⇒ log 2 = + c ⇒ c = log2
2
log |sin2x|
⇒ log|y| = + log2
2

SHORT ANSWER TYPE QUESTIONS [3 MARKS]


Q.1 Find the particular solution of the differential equation:
𝜋
𝑒 𝑥 𝑡𝑎𝑛𝑦 𝑑𝑥 + (2 − 𝑒 𝑥 )𝑠𝑒𝑐 2 𝑦 𝑑𝑦 = 0, given that 𝑦 = 4 𝑤ℎ𝑒𝑛 𝑥 = 0
𝑦 𝑑𝑦
Q.2 Find the general solution of the differential equation 𝑥𝑐𝑜𝑠 ( 𝑥 ) 𝑑𝑥 =
𝑦
𝑦𝑐𝑜𝑠 ( 𝑥 ) + 𝑥, 𝑥 ≠ 0
𝑥
Q3. Solve the differential equation 2𝑦𝑒 𝑥/𝑦 𝑑𝑥 + (𝑦 − 2𝑥𝑒 𝑦 ) 𝑑𝑦, 𝑦 ≠ 0
𝑑𝑦
Q.4 Find the general solution of the differential equation:(𝑦 − 𝑥 𝑑𝑥 ) =
𝑑𝑦
𝑎(𝑦 2 + 𝑑𝑥 )
Q.5 Solve the differential equation 𝑥𝑑𝑦 − 𝑦 𝑑𝑥 = √𝑥 2 + 𝑦 2 𝑑𝑥
𝑑𝑦 𝑦 𝑦
Q.6 Solve the differential equation − + 𝑐𝑜𝑠𝑒𝑐 = 0, 𝑦(1) = 0
𝑑𝑥 𝑥 𝑥
𝑑𝑦 𝜋
Q.7 Solve the differential equation𝑑𝑥 + 2𝑦 tanx = 𝑠𝑖𝑛𝑥, 𝑦 ( 3 ) = 0
Q.8 Find the general solution of the differential equation 𝑦𝑑𝑥 −
(𝑥 + 2𝑦 2 )𝑑𝑦 = 0

SOLUTIONS
𝑑𝑦 𝑒 𝑥 𝑡𝑎𝑛𝑦
Q.1 𝑒 𝑥 𝑡𝑎𝑛𝑦 𝑑𝑥 + (2 − 𝑒 𝑥 )𝑠𝑒𝑐 2 𝑦 𝑑𝑦 = 0 ⇒ 𝑑𝑥 = (𝑒 𝑥 −2)𝑠𝑒𝑐 2 𝑦

Mathematics Page 169 of 260 XII


𝑠𝑒𝑐 2 𝑦 𝑒𝑥 𝑠𝑒𝑐 2 𝑦 𝑒𝑥
⇒ dy = (𝑒 𝑥 −2) dx ⇒∫ dy = ∫ (𝑒 𝑥 −2) dx
tany tan y
𝑥
⇒ log|tan y|= log| (𝑒 − 2)| + 𝑐
𝜋
𝑦 = 4 𝑤ℎ𝑒𝑛 𝑥 = 0 ⇒ log|tan π/4|= log| (𝑒 0 − 2)| + 𝑐 ⇒ c = 0
⇒ log|tan y|= log| (𝑒 𝑥 − 2)| ⇒ tan y = (𝑒 𝑥 − 2)
𝑦 𝑑𝑦 𝑦 𝑦 𝑑𝑦 𝑦 𝑦
Q.2 𝑥𝑐𝑜𝑠 ( 𝑥 ) 𝑑𝑥 = 𝑦𝑐𝑜𝑠 ( 𝑥 ) + 𝑥 ⇒ 𝑐𝑜𝑠 ( 𝑥 ) 𝑑𝑥 = 𝑥 𝑐𝑜𝑠 ( 𝑥 ) + 1
𝑦 𝑦
𝑑𝑦 𝑐𝑜𝑠( )+1 𝑦
𝑥 𝑥
⇒ 𝑑𝑥 = 𝑦 = 𝜑 ( 𝑥 ) [Homogeneous D.E]
𝑐𝑜𝑠( )
𝑥
𝑑𝑦 𝑑𝑣
Let 𝑦 = 𝑣𝑥 ⇒ 𝑑𝑥 = 𝑣 + 𝑥 𝑑𝑥
𝑑𝑣 𝑣𝑐𝑜𝑠𝑣+1
⇒ 𝑣 + 𝑥 𝑑𝑥 = 𝑐𝑜𝑠𝑣
𝑑𝑣 𝑣𝑐𝑜𝑠𝑣+1 𝑑𝑣 1
⇒ 𝑥 𝑑𝑥 = − 𝑣 ⇒ 𝑥 𝑑𝑥 = cos 𝑣
𝑐𝑜𝑠𝑣
dx dx
⇒ cosv dv = ⇒ ∫ cosv dv = ∫
x x
y
⇒ sinv = log x + logc ⇒ sin x = 𝑙𝑜𝑔𝑐𝑥
𝑥
𝑥 (2𝑥𝑒 𝑦−𝑦)
dx
Q.3 2𝑦𝑒 𝑥/𝑦 𝑑𝑥 + (𝑦 − 2𝑥𝑒 ) 𝑑𝑦 ⇒ dy = 𝑦
2𝑦𝑒 𝑥/𝑦
𝑥
𝑥
(2 𝑒 𝑦−1)
dx 𝑦 𝑥
= = 𝜑 ( 𝑦) [Homogeneous D.E]
dy 2𝑒 𝑥/𝑦
𝑑𝑥 𝑑𝑣
Let 𝑥 = 𝑣𝑦 ⇒ 𝑑𝑦 = 𝑣 + 𝑦 𝑑𝑦
𝑑𝑣 2𝑣𝑒 𝑣 −1 𝑑𝑣 2𝑣𝑒 𝑣 −1
⇒ 𝑣 + 𝑦 𝑑𝑦 = ⇒ 𝑦 𝑑𝑦 = −𝑣
2𝑒 𝑣 2𝑒 𝑣
𝑑𝑣 −1 1
⇒ 𝑦 𝑑𝑦 = 2𝑒 𝑣 ⇒ 2𝑒 𝑣 𝑑𝑣 = − 𝑦 𝑑𝑦
1
⇒ 2 ∫ 𝑒 𝑣 𝑑𝑣 = − ∫ 𝑦 𝑑𝑦 ⇒ 2𝑒 𝑣 = − log|𝑦| + 𝑙𝑜𝑔𝑐
x
c
⇒ 2ey = log (y)
𝑑𝑦 𝑑𝑦 dy
Q.4 (𝑦 − 𝑥 𝑑𝑥 ) = 𝑎(𝑦 2 + 𝑑𝑥 ) ⇒ dx (a + x) = y(1 − ay)
dy dx dy dx
⇒ y(1−ay) = a+x ⇒ ∫ y(1−ay) = ∫ a+x
1−ay+ay dx 1 a dx
⇒∫ dy = ∫ a+x ⇒ ∫ y + 1−ay dy = ∫ a+x
y(1−ay)
⇒ log|y| − log|1 − ay) = log|𝑎 + 𝑥 | + 𝑙𝑜𝑔𝑐
Mathematics Page 170 of 260 XII
y
⇒ log |1−ay| = log |c(a + x)|
y
⇒ = c(a + x)
1−ay

Q.5 𝑥𝑑𝑦 − 𝑦 𝑑𝑥 = √𝑥 2 + 𝑦 2 𝑑𝑥
𝑑𝑦 𝑦+√𝑥 2 +𝑦 2
⇒ 𝑑𝑥 = 𝑥
𝑑𝑦 𝑦 𝑦 𝑦
⇒ = 𝑥 + √1 + ( 𝑥 )2 = 𝜑 ( 𝑥 ) [Homogeneous D.E]
𝑑𝑥
𝑑𝑦 𝑑𝑣
Let 𝑦 = 𝑣𝑥 ⇒ 𝑑𝑥 = 𝑣 + 𝑥 𝑑𝑥
𝑑𝑣
⇒ 𝑣 + 𝑥 𝑑𝑥 = 𝑣 + √1 + 𝑣 2
𝑑𝑣
⇒ 𝑥 𝑑𝑥 = √1 + 𝑣 2
dv dx dv dx
⇒ √1+𝑣 2 = ⇒ ∫ √1+𝑣 2 = ∫
x x
⇒ log |v + √1 + 𝑣 2 |= log| 𝑥| + log 𝑐
y 𝑦
⇒ log | x + √1 + ( 𝑥 )2 | = log |𝑐𝑥|

𝑦 𝑦
⇒ + √1 + ( 𝑥 )2 = 𝑐𝑥
𝑥
𝑑𝑦 𝑦 𝑦 𝑑𝑦 𝑦 𝑦 𝑦
Q.6 − + 𝑐𝑜𝑠𝑒𝑐 = 0 ⇒ = − 𝑐𝑜𝑠𝑒𝑐 = 𝜑 ( ) [Homogeneous D.E]
𝑑𝑥 𝑥 𝑥 𝑑𝑥 𝑥 𝑥 𝑥
𝑑𝑦 𝑑𝑣
Let 𝑦 = 𝑣𝑥 ⇒ 𝑑𝑥 = 𝑣 + 𝑥 𝑑𝑥
𝑑𝑣
⇒ 𝑣 + 𝑥 𝑑𝑥 = 𝑣 − 𝑐𝑜𝑠𝑒𝑐 𝑣
𝑑𝑣
⇒ 𝑥 𝑑𝑥 = −𝑐𝑜𝑠𝑒𝑐 𝑣
dv dx
⇒ =−
cosec v x
dv dx
⇒ −∫ =∫
cosec v x
dx
⇒ − ∫ sin v dv = ∫ ⇒ cos v = log|𝑥 | + 𝑙𝑜𝑔𝑐
x
y
⇒ cos x = log |cx|
𝑑𝑦 𝑑𝑦
Q.7 + 2𝑦 tanx = 𝑠𝑖𝑛𝑥 [ + 𝑃𝑦 = 𝑄 ]
𝑑𝑥 𝑑𝑥
𝑃 = 2 tan 𝑥, 𝑄 = sin 𝑥
Mathematics Page 171 of 260 XII
2
𝐼. 𝐹 = 𝑒 ∫ 2 tan 𝑥 𝑑𝑥 ⇒ e2log | sec x| ⇒ 𝑒 log(𝑠𝑒𝑐 𝑥) ⇒ 𝑠𝑒𝑐 2 𝑥
Solution: 𝑦 𝑠𝑒𝑐 2 = ∫ 𝑠𝑒𝑐 2 𝑥. 𝑠𝑖𝑛𝑥 𝑑𝑥
⇒ 𝑦 𝑠𝑒𝑐 2 = ∫ 𝑠𝑒𝑐 2 𝑥. 𝑠𝑖𝑛𝑥 𝑑𝑥 ⇒ 𝑦 𝑠𝑒𝑐 2 = ∫ 𝑠𝑒𝑐𝑥. 𝑡𝑎𝑛𝑥 𝑑𝑥
⇒ 𝑦 𝑠𝑒𝑐 2 = 𝑠𝑒𝑐𝑥 + 𝑐 ⇒ 𝑦 = 𝑐𝑜𝑠𝑥 + +𝑐 𝑐𝑜𝑠 2 𝑥
𝜋
𝑦 = 0 𝑤ℎ𝑒𝑛 𝑥 = 3
1 1
0 = 2 + 4 𝑐 ⇒ c = −2
⇒ 𝑦 = 𝑐𝑜𝑠𝑥 − 2 𝑐𝑜𝑠 2 𝑥
dx (𝑥+2𝑦 2 )
Q.8 𝑦𝑑𝑥 − (𝑥 + 2𝑦 2 )𝑑𝑦 = 0 ⇒ dy = 𝑦
dx x dx
⇒ dy − = 2y [ dy + Px = Q]
y
1
𝑃 = − 𝑦 , 𝑄 = 2𝑦
1 1
− ∫ 𝑑𝑦 log( ) 1
𝐼. 𝐹. = 𝑒 𝑦 ⇒ e−logy = e 𝑦 =y
1 1 x
Solution: 𝑥. y = ∫ 2y. y dy ⇒ y = 2y + c
⇒ x = 2y 2 + cy
LONG ANSWER TYPE QUESTIONS [5 MARKS]
Q.1 Solve the differential equation (1 + 𝑦 2 )𝑑𝑥 = (𝑡𝑎𝑛−1 𝑦 − 𝑥 )𝑑𝑦, 𝑦(1) =
0
𝑑𝑦
Q.2 Solve the differential equation 𝑥 𝑑𝑥 + 𝑦 − 𝑥 + 𝑥𝑦 𝑐𝑜𝑡𝑥 = 0, 𝑥 ≠ 0
𝑦 𝑦
Q.3 Solve the differential equation 𝑦 [𝑥𝑐𝑜𝑠 (𝑥 ) + 𝑦𝑠𝑖𝑛 (𝑥 )] 𝑑𝑥 =
𝑦 𝑦
𝑥 [𝑦𝑠𝑖𝑛 (𝑥 ) − 𝑥𝑐𝑜𝑠 (𝑥 )] 𝑑𝑦
𝑑𝑦
Q.4 Solve the differential equation √1 + 𝑥 2 + 𝑦 2 + 𝑥 2 𝑦 2 + 𝑥𝑦 𝑑𝑥 = 0
SOLUTION
dx (𝑡𝑎𝑛 −1 𝑦−𝑥)
Q.1 (1 + 𝑦 2 )𝑑𝑥 = (𝑡𝑎𝑛 −1 𝑦 − 𝑥 )𝑑𝑦 ⇒ dy = (1+𝑦 2 )
dx 𝑥 (𝑡𝑎𝑛 −1𝑦) dx
⇒ + (1+𝑦 2 ) = (1+𝑦 2 )
[ + Px = Q]
dy dy
1 (𝑡𝑎𝑛 −1 𝑦)
𝑃 = (1+𝑦 2 ) , Q= (1+𝑦 2)
1
∫ dy −1 𝑦
(1+𝑦2 )
𝐼. 𝐹 = 𝑒 = 𝑒 𝑡𝑎𝑛
Mathematics Page 172 of 260 XII
−1𝑦 (𝑡𝑎𝑛 −1𝑦) −1 𝑦
Solution: 𝑥𝑒 𝑡𝑎𝑛 =∫ (1+𝑦 2)
. 𝑒 𝑡𝑎𝑛 𝑑𝑦
1
𝐿𝑒𝑡 𝑡𝑎𝑛−1 𝑦 = 𝑡 ⇒ (1+𝑦 2 ) dy = dt
−1𝑦
𝑥𝑒 𝑡𝑎𝑛 = ∫ 𝑡𝑒 𝑡 𝑑𝑡
= 𝑡𝑒 𝑡 − ∫ 1. 𝑒 𝑡 𝑑𝑡
= 𝑡𝑒 𝑡 − 𝑒 𝑡 + 𝑐 = 𝑒 𝑡 (𝑡 − 1) + 𝑐
−1 −1
⇒ 𝑥𝑒 𝑡𝑎𝑛 𝑦 = 𝑒 𝑡𝑎𝑛 𝑦 ( 𝑡𝑎𝑛−1 𝑦 − 1) + 𝑐
𝑐
𝑥 = ( 𝑡𝑎𝑛−1 𝑦 − 1) + 𝑡𝑎𝑛−1𝑦
𝑒
c
𝑦 = 0 𝑤ℎ𝑒𝑛 𝑥 = 1 ⇒ 1 = (0 − 1) + 1 ⇒ 𝑐 = 2
2
⇒ 𝑥 = ( 𝑡𝑎𝑛−1 𝑦 − 1) + −1𝑦
𝑒 𝑡𝑎𝑛

𝑑𝑦 𝑑𝑦 𝑦
Q.2 𝑥 𝑑𝑥 + 𝑦 − 𝑥 + 𝑥𝑦 𝑐𝑜𝑡𝑥 = 0 ⇒ + 𝑥 + 𝑦 𝑐𝑜𝑡𝑥 = 1
𝑑𝑥
𝑑𝑦 1 𝑑𝑦
⇒ + (𝑥 + 𝑐𝑜𝑡𝑥) 𝑦 = 1 [ + 𝑃𝑦 = 𝑄 ]
𝑑𝑥 𝑑𝑥
1
𝑃 = (𝑥 + 𝑐𝑜𝑡𝑥) , 𝑄=1
1
𝐼. 𝐹 = 𝑒 ∫ (𝑥+𝑐𝑜𝑡𝑥) 𝑑𝑥 = 𝑒 log|𝑥|+log |𝑠𝑖𝑛𝑥| = 𝑒 log|( 𝑥𝑠𝑖𝑛𝑥)| = 𝑥𝑠𝑖𝑛𝑥
Solution: 𝑦. 𝑥𝑠𝑖𝑛𝑥 = ∫ 1. 𝑥𝑠𝑖𝑛𝑥 𝑑𝑥
= 𝑥(−𝑐𝑜𝑠𝑥 ) − ∫ 1. (−𝑐𝑜𝑠𝑥)𝑑𝑥
⇒ 𝑥𝑦𝑠𝑖𝑛𝑥 = −𝑥𝑐𝑜𝑠𝑥 + 𝑠𝑖𝑛𝑥 + 𝑐
𝑦 𝑦 𝑦 𝑦
Q.3 𝑦 [𝑥𝑐𝑜𝑠 (𝑥 ) + 𝑦𝑠𝑖𝑛 (𝑥 )] 𝑑𝑥 = 𝑥 [𝑦𝑠𝑖𝑛 (𝑥 ) − 𝑥𝑐𝑜𝑠 (𝑥 )] 𝑑𝑦
𝑦 𝑦 𝑦𝑦 𝑦
dy 𝑦[𝑥𝑐𝑜𝑠( )+𝑦𝑠𝑖𝑛( )] dy 𝑥𝑦[𝑐𝑜𝑠( )+ 𝑠𝑖𝑛( )]
𝑥 𝑥 𝑥𝑥 𝑥
⇒ dx = 𝑦 𝑦 ⇒ dx = 𝑦 𝑦 𝑦
𝑥[𝑦𝑠𝑖𝑛( )−𝑥𝑐𝑜𝑠( )] 𝑥.𝑥[ 𝑠𝑖𝑛( )−𝑐𝑜𝑠( )]
𝑥 𝑥 𝑥 𝑥 𝑥
𝑦 𝑦 𝑦
dy 𝑦 [𝑐𝑜𝑠(𝑥 )+ 𝑥𝑠𝑖𝑛(𝑥)] 𝑦
⇒ dx = [ ] = 𝜑 ( ) [Homogeneous D.E]
𝑥 [𝑦𝑠𝑖𝑛(𝑦)−𝑐𝑜𝑠(𝑦) ] 𝑥
𝑥 𝑥 𝑥
𝑑𝑦 𝑑𝑣
Let 𝑦 = 𝑣𝑥 ⇒ 𝑑𝑥 = 𝑣 + 𝑥 𝑑𝑥
𝑑𝑣 [𝑐𝑜𝑠𝑣+𝑣𝑠𝑖𝑛𝑣] 𝑑𝑣 [𝑐𝑜𝑠𝑣+𝑣𝑠𝑖𝑛𝑣]
𝑣+𝑥 = 𝑣 [ [𝑣𝑠𝑖𝑛𝑣−𝑐𝑜𝑠𝑣 ]] ⇒ 𝑥 = 𝑣 [ [𝑣𝑠𝑖𝑛𝑣−𝑐𝑜𝑠𝑣 ]] − 𝑣
𝑑𝑥 𝑑𝑥
𝑑𝑣 [2𝑣𝑐𝑜𝑠𝑣] [𝑣𝑠𝑖𝑛𝑣−𝑐𝑜𝑠𝑣 ] 𝑑𝑥
⇒ 𝑥 𝑑𝑥 = [ [𝑣𝑠𝑖𝑛𝑣−𝑐𝑜𝑠𝑣 ]] ⇒ [2𝑣𝑐𝑜𝑠𝑣]
𝑑𝑣 = 𝑥
1 [𝑣𝑠𝑖𝑛𝑣−𝑐𝑜𝑠𝑣 ] 𝑑𝑥 1 1 𝑑𝑥
⇒ ∫ [𝑣𝑐𝑜𝑠𝑣]
𝑑𝑣 = ∫ ⇒ ∫ [𝑡𝑎𝑛𝑣 − ]𝑑𝑣 = ∫
2 𝑥 2 𝑣 𝑥

Mathematics Page 173 of 260 XII


1
⇒ 2 [log|𝑠𝑒𝑐𝑣| − log|𝑣|] = 𝑙𝑜𝑥|𝑥 | + 𝑙𝑜𝑔𝑐
𝑠𝑒𝑐𝑣 𝑠𝑒𝑐𝑣
⇒ log|√ |= 𝑙𝑜𝑔 |cx| ⇒ √ = 𝑐𝑥
𝑣 𝑣

𝑦
𝑠𝑒𝑐 𝑦
⇒√ 𝑦
𝑥
= 𝑐𝑥 ⇒ 𝑠𝑒𝑐 𝑥 = 𝑐 2 𝑥𝑦
𝑥

𝑑𝑦 dy √(1+𝑥 2)+𝑦 2(1+𝑥 2 )


Q.4 √1 + 𝑥 2 + 𝑦 2 + 𝑥 2 𝑦 2 + 𝑥𝑦 𝑑𝑥 = 0 ⇒ dx = − 𝑥𝑦
dy √(1+𝑥 2 )(1+𝑦 2 ) y √1+x2
⇒ =− ⇒ dy = − 𝑑𝑥
dx 𝑥𝑦 √1+y2 x
y √1+x2
⇒∫ 2
dy = − ∫ 𝑑𝑥
√1+y x
y
𝐿𝑒𝑡 𝐼1 = ∫ dy
√1+y2
dt
Let 1 + y 2 = t ⇒ 2ydy = dt ⇒ ydy = 2
1 dt 1
𝐼1 = 2 ∫ = 2 . 2 √ 𝑡 = √𝑡
√t
∴ 𝐼1 = √1 + y 2
√1+x2 √1+x2
Let 𝐼2 = ∫ 𝑑𝑥 = ∫ 𝑥𝑑𝑥 [multiply and divide by x]
x x2

Let 1 + x 2 = v 2 ⇒ x 2 = v 2 − 1 ⇒ 2xdx = 2vdv ⇒ xdx = vdv


𝑣 𝑣2 (𝑣 2−1)+1 1
𝐼2 = ∫ v2 −1 𝑣 𝑑𝑣 = ∫ v2 −1 𝑑𝑣 = ∫ 𝑑𝑣 = ∫ 1 + v2 −1 𝑑𝑣
v2 −1
1 𝑣−1
𝐼2 = 𝑣 + log | |+𝑐
2 𝑣+1
1 √1+𝑥 2−1
⇒ 𝐼2 = √1+𝑥 2 + log | |+𝑐
2 √1+𝑥 2+1
1 √1+𝑥 2 −1
∴ Required solution: √1 + y 2 = −√1+𝑥 2 − 2 log |√1+𝑥 2 | + 𝑐
+1
1 √1+𝑥 2−1
⇒ √1 + y 2 + √1+𝑥 2 + 2 log |√1+𝑥 2 | = 𝑐
+1

QUESTIONS FOR J.E.E. (mains/advanced)


𝑑𝑦
Q.1 𝐿𝑒𝑡 𝑦 = 𝑦(𝑥 )𝑏𝑒 𝑡ℎ𝑒 𝑠𝑜𝑙𝑢𝑡𝑖𝑜𝑛 𝑜𝑓 𝑑𝑖𝑓𝑓𝑒𝑟𝑒𝑛𝑡𝑖𝑎𝑙 𝑒𝑞𝑢𝑎𝑡𝑖𝑜𝑛: 𝑠𝑖𝑛𝑥 𝑑𝑥 +
𝑦𝑐𝑜𝑠𝑥 = 4𝑥,
Mathematics Page 174 of 260 XII
𝜋 𝜋
𝑥 ∈ (0, 𝜋), 𝑖𝑓 𝑦 ( 2 ) = 0, 𝑡ℎ𝑒𝑛 𝑦 ( 6 ) 𝑖𝑠 𝑒𝑞𝑢𝑎𝑙 𝑡𝑜:
4 −8 −8 −8
(a) 𝜋2 (b) 9 𝜋2 (c ) 𝜋2 (d) 𝜋2 (J.E.E
9 √3 √3 9 9
(mains)2018)
Q.2 𝐼𝑓 𝑦 = 𝑦(𝑥)𝑠𝑎𝑡𝑖𝑠𝑓𝑖𝑒𝑠 𝑡ℎ𝑒 𝑑𝑖𝑓𝑓𝑒𝑟𝑒𝑛𝑡𝑖𝑒𝑙 𝑒𝑞𝑢𝑎𝑡𝑖𝑜𝑛 ∶

8√𝑥 (√9 + √𝑥) 𝑑𝑦 = [√4 + √9 + √𝑥 )]−1 𝑑𝑥,𝑥 > 0 𝑎𝑛𝑑 𝑦(0) =


√7 𝑡ℎ𝑒𝑛 𝑦(256) =
(a) 9 (b) 3 (c) 80 (d) 16 (J.E.E
(Advanced) 2017)
𝑑𝑦
Q.3 𝐼𝑓 (2 + 𝑠𝑖𝑛𝑥 ) 𝑑𝑥 + (𝑦 + 1)𝑐𝑜𝑠𝑥 = 0 𝑎𝑛𝑑 𝑦(0) =
𝜋
1 𝑡ℎ𝑒𝑛 𝑦 ( 2 ) 𝑖𝑠 𝑒𝑞𝑢𝑎𝑙 𝑡𝑜
1 4 1 2
(a) − 3 (b) 3 ( c) 3 (d) − 3 (J.E.E
(mains)2017)
Q.4 𝐼𝑓 𝑡ℎ𝑒 𝑐𝑢𝑟𝑣𝑒 𝑦 =
𝑓(𝑥 )𝑝𝑎𝑠𝑠𝑒𝑠 𝑡ℎ𝑟𝑜𝑢𝑔ℎ 𝑡ℎ𝑒 𝑝𝑜𝑖𝑛𝑡 (1, −1)𝑎𝑛𝑑 𝑠𝑎𝑡𝑖𝑠𝑓𝑖𝑒𝑠 𝑡ℎ𝑒 𝐷. 𝐸.
1
𝑦(1 + 𝑥𝑦)𝑑𝑥 = 𝑥𝑑𝑦, 𝑡ℎ𝑒𝑛 𝑓 (− 2) 𝑖𝑠 𝑒𝑞𝑢𝑎𝑙 𝑡𝑜
4 2 4 2
(a) − 5 (b) 5 ( c) 5 (d) − 5 (J.E.E
(mains)2016)
𝑑𝑦 𝑥𝑦
Q.5 𝐿𝑒𝑡 𝑦 = 𝑓 (𝑥 )𝑏𝑒 𝑡ℎ𝑒 𝑠𝑜𝑙𝑢𝑡𝑖𝑜𝑛 𝑜𝑓 𝑑𝑖𝑓𝑓𝑒𝑟𝑒𝑛𝑡𝑖𝑎𝑙 𝑒𝑞𝑢𝑎𝑡𝑖𝑜𝑛: 𝑑𝑥 + 𝑥 2 −1 =
𝑥 4+2𝑥
in (-1, 1)
√1−𝑥 2
√3/2
Satisfying f(0) =0 then∫−√3/2 𝑓 (𝑥 )𝑑𝑥 𝑖𝑠 𝑒𝑞𝑢𝑎𝑙 𝑡𝑜
𝜋 √3 𝜋 √3 𝜋 √3 𝜋 √3
(a) 3 − (b) 3 − ( c) 6 − (d) 6 − (J.E.E
2 4 4 2
(mains)2015)
SOLUTIONS
𝑑𝑦 𝑑𝑦 4𝑥
Q.1 (𝑐 ) 𝑠𝑖𝑛𝑥 𝑑𝑥 + 𝑦𝑐𝑜𝑠𝑥 = 4𝑥 ⇒ 𝑑𝑥 + 𝑦𝑐𝑜𝑡𝑥 = 𝑠𝑖𝑛𝑥
I.F =𝑒 ∫ 𝑐𝑜𝑡𝑥 𝑑𝑥 =𝑒 log(𝑠𝑖𝑛𝑥) = 𝑠𝑖𝑛𝑥
4𝑥
𝑦𝑠𝑖𝑛𝑥 = ∫ 𝑠𝑖𝑛𝑥 𝑠𝑖𝑛𝑥 𝑑𝑥 ⇒ 𝑦𝑠𝑖𝑛𝑥 = 2𝑥 2 + 𝑐
Mathematics Page 175 of 260 XII
𝜋 𝜋2 𝜋2
𝑦 = 0 𝑤ℎ𝑒𝑛 𝑥 = 2 , ∴ 0 = +𝑐⇒c = −
2 2
𝜋2
𝜋2 2𝑥 2 −
∴ 𝑦𝑠𝑖𝑛𝑥 = 2𝑥 2 − ⇒y= 2
2 sinx
𝜋2 𝜋2
𝜋 2( )− 8𝜋2
36 2
𝑤ℎ𝑒𝑛 𝑥 = 6 , 𝑦 = 1 = −( )
9
2

Q.2 (b) 8√𝑥 (√9 + √𝑥) 𝑑𝑦 = [√4 + √9 + √𝑥 )]−1 𝑑𝑥


dy 1 1
⇒ dx = ⇒ dy = dx
8√𝑥(√9+√𝑥)√4+√9+√𝑥 ) 8√𝑥(√9+√𝑥)√4+√9+√𝑥 )
1 1
⇒ ∫ dy = ∫ dx
8
√𝑥 (√9 + √𝑥) √4 + √9 + √𝑥 )
1 1 1
𝐿𝑒𝑡 √9 + √𝑥 = 𝑡 ⇒ ×2 𝑑𝑥 = 𝑑𝑡 ⇒ dx =
2√9+√𝑥 √𝑥 √𝑥(√9+√𝑥)

4dt
4 1
⇒ 𝑦 = 8∫ 𝑑𝑡 ⇒ 𝑦 = √4 + 𝑡 + 𝑐 ⇒ 𝑦 = √4 + √9 + √𝑥 + 𝑐
√4+𝑡

𝑦 = √7 𝑤ℎ𝑒𝑛 𝑥 = 0 ∴ √7 = √4 + √9 + √0 + 𝑐 ⇒ √7 = √7 + 𝑐 ⇒ c =
0

⇒ 𝑦 = √4 + √9 + √𝑥 + 0 ∴ 𝑤ℎ𝑒𝑛 𝑥 = 256, 𝑦 = √4 + √9 + √256 = 3


𝑑𝑦 dy cosx cosx
Q3.(c) (2 + 𝑠𝑖𝑛𝑥 ) 𝑑𝑥 + (𝑦 + 1)𝑐𝑜𝑠𝑥 = 0 ⇒ dx + (2+𝑠𝑖𝑛𝑥) y = − (2+𝑠𝑖𝑛𝑥)
cosx
∫(2+𝑠𝑖𝑛𝑥)dx
𝐼. 𝐹 = 𝑒 = 𝑒 log(2+𝑠𝑖𝑛𝑥) = (2 + 𝑠𝑖𝑛𝑥)
cosx
∴ 𝑦(2 + 𝑠𝑖𝑛𝑥 ) = − ∫ × (2 + sinx) dx
(2 + 𝑠𝑖𝑛𝑥 )
𝑦(2 + 𝑠𝑖𝑛𝑥 ) = −𝑠𝑖𝑛𝑥 + 𝑐
𝑦 = 1𝑤ℎ𝑒𝑛 𝑥 = 0 ∴ 1(2 + 0) = −0 + 𝑐 ⇒ c = 2
−𝑠𝑖𝑛𝑥+2
𝑦= 2+𝑠𝑖𝑛𝑥
𝜋 −1+2 1
∴ 𝑤ℎ𝑒𝑛 𝑥 = 𝑦= ⇒y=3
2 2+1

Mathematics Page 176 of 260 XII


Q.4(c) 𝑦(1 + 𝑥𝑦)𝑑𝑥 = 𝑥𝑑𝑦 ⇒ ydx + xy 2 dx = xdx ⇒ ydx − xdy +
xy 2 dx = 0 (divide by 𝑦 2 )
ydx−xdy x
⇒ = −xdx ⇒ d (y) = −xdx (Integrating on both sides)
y2
x x2
(y) = − + c Since it is passes through (1, -1)
2
1 1
∴ −1 = − 2 + c ⇒ c = − 2
x x2 1 x x
∴ (y) = − − ⇒ y = − x2 1
⇒ f(x) = − x2 1
2 2 + +
2 2 2 2
1
1 (− ) 4
2
∴ 𝑓 (− 2) = − 1 1 =5
+
8 2

𝑑𝑦 𝑥𝑦 𝑥 4+2𝑥
Q.5 (b) + 1−𝑥 2 = √1−𝑥 2
𝑑𝑥
𝑥 1 2 )]
𝑑𝑥
𝐼. 𝐹 = 𝑒 ∫𝑥2 −1 = 𝑒 2[log(1−𝑥 = √1 − 𝑥 2
𝑥 4+2𝑥
𝑦√1 − 𝑥 2 = ∫ √1−𝑥 2 × √1 − 𝑥 2 𝑑𝑥
𝑥5
⇒ 𝑦√1 − 𝑥 2 = + 𝑥2 + 𝑐
5
𝑥5 𝑥5
+𝑥 2 +𝑐 +𝑥 2+𝑐
5 5
⇒y= ⇒ f(x) =
√1−𝑥 2 √1−𝑥 2
Since f (0) = 0 ⇒ c = 0
𝑥5
+𝑥 2
5
∴ f(x) = √1−𝑥 2
2𝑥5
√3/2 5 +𝑥 1 √3/2 x5 √3/2 𝑥2
⇒ ∫−√3/2 √1−𝑥 2 dx = 5 ∫−√3/2 √1−𝑥 2 dx + ∫−√3/2 √1−𝑥 2 dx
Odd function even function
√3 𝑥5
+𝑥 2 √3
𝑥2
√3
(1−𝑥 2)−1
2 5
⇒∫ 2
dx = 0 + 2∫02 dx = − 2 ∫02 dx
√3
− √1−𝑥 √1−𝑥 2 √1−𝑥 2
2
√3
1
= −2 ∫02 √1 − 𝑥 2 − √1−𝑥 2 𝑑𝑥

= −2[2 {𝑥√1 − 𝑥 2 + 𝑠𝑖𝑛−1 𝑥} − 𝑠𝑖𝑛−1 𝑥]√3/2


1
0
1 √3 3 1 √3
= −2 [ { √1 − − 𝑠𝑖𝑛−1 }] −0
2 2 4 2 2

Mathematics Page 177 of 260 XII


𝜋 √3
= −
3 4
CASE STUDY
General anesthesia is used for major operations to cure the patients and
conductpain free surgeries. Propofol is a commonly used anesthetic injected
for major operations such as knee replacement or open heart surgery. It also
acts as a sedative and an analgesic.

A patient is rushed to operation theatre for a 2-hour cardiac surgery. A


person is anesthetized when its blood stream contains at least 3 mg of
propofol per kg of body weight. The rate of change of propofol(𝑥), in the
body is proportional to thequantity of propofol present at that time.
Based on the above information answer the following questions:
a. Show that propofol given intravenously is eliminated exponentially
from the patients’ blood stream
b. What dose of propofol should be injected to induce
unconsciousness in a 50 Kg adult for a two hours operation?
(Given (2)1/5 =1.1487 & assume half-life of propofol = 5 hours)
SOLUTION
𝑑𝑥 dx
(a) ∴ 𝑑𝑡 ∝ 𝑥 ⇒ dt = −kx
dx
⇒ = −k dt
x
dx
⇒ ∫ x = ∫ −k dt ⇒ logx = −kt + c
⇒ x = e−kt+c
⇒ x = λe−kt
Let initially at 𝑡 = 0, 𝑥 = 𝑥0
𝑥0 = λe0 ⇒ 𝑥0 = λ
Mathematics Page 178 of 260 XII
⇒ x = 𝑥0 e−kt ----------- (1) where 𝑥0 = original quantity
𝑥0
(b) At 𝑡 = 5, 𝑥 = 2 (∵ half life = 5 hours)
1
𝑥0 1
= 𝑥0 e−5k ⇒ e−5k = 2 ⇒ e5k = 2 ⇒ ek = (2)5
2
The quantity of propofol needed in a 50 Kg adult at the end of 2 hours = 50 ×
3 = 150 mg
⇒ 150 = 𝑥0 e−2k [Using (1)]
⇒ 𝑥0 = 150 e2k ⇒ 𝑥0 = 150(𝑒 𝑘 )2
1
⇒ 𝑥0 = 150 × ( (2)5 )2
⇒ 𝑥0 = 150 × ( 1.1487)2 ⇒ 𝑥0 = 197.93 mg.

CHAPTER – 10
VECTOR ALGEBRA

VECTOR: A directed line segment is called the vector and those quantities
having magnitude as well as direction are called the vector quantities or
vectors.

But if the quantity has no direction then they are called scalars.

⃗⃗⃗⃗⃗ , A is the initial point and B is the terminal point.


In a vector 𝐴𝐵

POSITION VECTOR AND MAGNITUDE OF VECTORS:

Let a point A is (2,3,4) and origin O is (0,0,0) then position vector of the
point A with respect to origin is given by 𝑂𝐴 ⃗⃗⃗⃗⃗ = 𝐴 - 𝑂⃗ = 2𝑖̂ + 3𝑗̂ + 4𝑘̂ - (0𝑖̂ +
o𝑗̂ + 0𝑘̂ ) = 2𝑖̂ + 3𝑗̂ + 4𝑘̂ .

⃗⃗⃗⃗⃗ | = √22 + 32 + 42 = √4 + 9 + 16 =
Now its magnitude is given by |𝑂𝐴
√29.

DIRECTION COSINES : If a line makes α, β, ϒ angles with positive X, Y and Z


axes respectively

Mathematics Page 179 of 260 XII


Then cos α, cos β and cosϒ are called the direction cosines and
denoted by l, m, n.

⇒ l = cos α, m = cos β and n = cosϒ

And l2 + m2 + n2 = 1

UNIT VECTOR : A vector of magnitude 1 unit, is called the unit vector.

⃗⃗⃗ | = √12 + (−1)2 + 22 =


⃗⃗⃗ = 𝑖 ̂ − 𝑗 ̂ + 2𝑘 ̂ ⇒ |𝑎
Example:- let 𝑎
√1 + 1 + 4 = √6.
⃗⃗⃗
𝑎 𝑖 ̂ − 𝑗 ̂ + 2𝑘 ̂ 1 1 2
Now unit vector of 𝑎
⃗⃗⃗ = 𝑎̂ = |𝑎 | = = 𝑖̂ − 𝑗̂ + 𝑘̂.
⃗⃗⃗ √6 √6 √6 √6

1 −1 2
And , and are the direction cosines of vector 𝑎
⃗⃗⃗ .
√6 √6 √6

COLLINEAR VECTOR : If the two vectors are parallel or collinear then one of
them can be written

As the scalar multiple of other vector.

For example if the two vectors 𝑎 ⃗⃗ are parallel then 𝑎


⃗⃗⃗ and 𝑏 ⃗⃗
⃗⃗⃗ = ±λ 𝑏
where λ is any scalar.

COMPONENT OF VECTOR : If a vector 𝑎


⃗⃗⃗ = a 𝑖̂ + 𝑏 𝑗 ̂ + 𝑐𝑘 ̂

Then a, b, c are the scalar component and a 𝑖̂, 𝑏 𝑗̂, 𝑐𝑘 ̂ are the
vector components.

Vector joining two points: A vector joining the two points A(1,2,3) and
B(3,5,6) is given by

⃗⃗⃗⃗⃗ = 𝐵
𝐴𝐵 ⃗⃗⃗ = (3𝑖̂ + 5𝑗̂ + 6𝑘̂ ) – (𝑖̂ + 2𝑗̂ + 3𝑘̂ ) = (2𝑖̂ + 3𝑗̂ + 3𝑘̂ ) .
⃗ -𝐴

Mathematics Page 180 of 260 XII


SECTION FORMULA: If a point P divides the line segment joining the two
⃗⃗⃗ and ⃗𝑏⃗ , in the ratio m : n
points A and B whose position vectors are 𝑎
(internally) .
⃗⃗⃗ +𝑛 𝑎
𝑚𝑏 ⃗⃗⃗
Then position vector of division point P = ⃗⃗⃗
𝑃 = .
𝑚+𝑛

⃗⃗⃗ −𝑛 𝑎
𝑚𝑏 ⃗⃗⃗
In case of external division, position vector of division point P = ⃗⃗⃗
𝑃 =
𝑚−𝑛
.

DOT / SCALAR PRODUCT OF TWO VECTORS :

If ϴ is the angle between the two vectors 𝑎 ⃗⃗⃗ and ⃗𝑏⃗ then dot product
⃗⃗⃗ · ⃗𝑏⃗ is given by 𝑎
or scalar product is denoted by 𝑎 ⃗⃗⃗ · ⃗𝑏⃗ = |𝑎 ⃗⃗ | cosϴ ,
⃗⃗⃗ |. |𝑏
where 0 ≤ ϴ ≤ π.

Either |𝑎 ⃗⃗ | = 0 then ϴ is not defined.


⃗⃗⃗ | = 0 or |𝑏

Angle between two vectors 𝑎 ⃗⃗⃗ (ϴ) = cos −1 [ 𝑎⃗⃗⃗ · ⃗⃗⃗𝑏 ]


⃗⃗⃗ and 𝑏 |𝑎 ⃗⃗⃗ |
⃗⃗⃗ |.|𝑏

If 𝑎 ⃗⃗⃗ = 0 then either |𝑎


⃗⃗⃗ · 𝑏 ⃗⃗ | = 0 or vectors 𝑎
⃗⃗⃗ | = 0 or |𝑏 ⃗⃗⃗ are
⃗⃗⃗ and 𝑏
perpendicular to each other.

VECTOR/ CROSS PRODUCT OF TWO VECTOTS :

If ϴ is the angle between the two vectors 𝑎⃗⃗⃗ and ⃗𝑏⃗ then vector
product or cross product is denoted by 𝑎 ⃗⃗ is given by 𝑎
⃗⃗⃗ × 𝑏 ⃗⃗ =
⃗⃗⃗ × 𝑏
|𝑎 ⃗⃗⃗ | sinϴ . 𝑛̂ , where 0 ≤ ϴ ≤ π and 𝑛̂ is a unit vector perpendicular to
⃗⃗⃗ |. |𝑏
the plane of vectors 𝑎 ⃗⃗⃗ .
⃗⃗⃗ and 𝑏

Either |⃗⃗⃗ ⃗⃗ | = 0 then ϴ is not defined.


𝑎 | = 0 or |𝑏

|𝑎
⃗⃗⃗ × 𝑏 ⃗⃗⃗ |
⃗⃗⃗ × ⃗⃗⃗
⇒ |𝑎 𝑏 | = |𝑎 ⃗⃗⃗ | sinϴ ⇒ ϴ = sin−1 [
⃗⃗⃗ |. |𝑏 |𝑎 ⃗⃗⃗ |
]
⃗⃗⃗ |.|𝑏

Mathematics Page 181 of 260 XII


⦿ Area of a triangle whose two sides are represented by the vectors 𝑎
⃗⃗⃗ and
⃗⃗⃗
𝑏
1
⃗⃗⃗ × ⃗𝑏⃗ |
= 2 |𝑎

⦿ Area of a parallelogram whose adjacent sides are represented by the


⃗⃗⃗ and ⃗⃗⃗
vectors 𝑎 𝑏

= |𝑎 ⃗⃗ |
⃗⃗⃗ × 𝑏

ONE MARK QUESTIONS (MCQ)

⃗⃗ | , if |𝑎
⃗⃗⃗⃗⃗⃗ − 𝑏
1- Find |3𝑎 ⃗⃗ | = 1 and 𝑎
⃗⃗⃗ | = 3, |𝑏 ⃗⃗ = 2.
⃗⃗⃗ · 𝑏
(a) 2 (b) 10 (c) 70 (d) 14

⃗⃗⃗⃗⃗⃗ − ⃗𝑏⃗ | 2 = (3𝑎


Sol: |3𝑎 ⃗⃗⃗⃗⃗⃗ − ⃗𝑏⃗ ). (3𝑎
⃗⃗⃗⃗⃗⃗ − ⃗⃗⃗
𝑏 ) = 9 |𝑎 ⃗⃗ |2 - 6 𝑎
⃗⃗⃗ |2 + |𝑏 ⃗⃗⃗ · ⃗𝑏⃗

= 9× 9 + 1 − 6 × 2 = 70 (OPTION- C).
2- Which of the following is true :
⃗⃗⃗ · ⃗𝑏⃗ > |𝑎
(a) 𝑎 ⃗⃗ |
⃗⃗⃗ |. |𝑏
(b) 𝑎 ⃗⃗ < |𝑎
⃗⃗⃗ · 𝑏 ⃗⃗⃗ |
⃗⃗⃗ |. |𝑏
(c) 𝑎 ⃗⃗ ≤ |𝑎
⃗⃗⃗ · 𝑏 ⃗⃗⃗ |
⃗⃗⃗ |. |𝑏
(d) 𝑎 ⃗⃗ ≥ |𝑎
⃗⃗⃗ · 𝑏 ⃗⃗⃗ |
⃗⃗⃗ |. |𝑏

⃗⃗⃗ · ⃗⃗⃗
Sol: we know that 𝑎 𝑏 = |𝑎 ⃗⃗⃗ | cosϴ and
⃗⃗⃗ |. |𝑏 -1 ≤ cosϴ ≤ 1.

So 𝑎 ⃗⃗⃗ · ⃗⃗⃗
𝑏 ≤ |⃗⃗⃗ ⃗⃗⃗ | is true statement (OPTION- C).
𝑎 |. |𝑏
3- Find the value of x+y+z if the vectors 𝑎 ⃗⃗⃗ = x𝑖̂ + y𝑗̂ + 3𝑘̂ and ⃗𝑏⃗ = 𝑖̂ + 𝑗̂
+ z𝑘̂ are equal:
(a) 5 (b) 6 (c) 3 (d) 8.

Sol: Given that: x𝑖̂ + y𝑗̂ + 3𝑘̂ = 𝑖̂ + 𝑗̂ + z𝑘̂


Mathematics Page 182 of 260 XII
⇒ x = 1, y = 1 and z = 3 so x+y+z = 5 (OPTION- A).

4- Find a unit vector in the direction of the vector 𝑎 ⃗⃗⃗ = 𝑖̂ + 𝑗̂ + 𝑘̂ :


1 1 1 1
(a) (𝑖̂ + 𝑗̂ + 𝑘̂ ) (b) (𝑖̂ + 𝑗̂ + 𝑘̂ ) (c) (𝑖̂ + 𝑗̂ + 𝑘̂ ) (d) (𝑖̂ + 𝑗̂ +
3 √3 √2 2
𝑘̂ ) .

⃗⃗⃗
𝑎
⃗⃗⃗ = 𝑎̂ = |𝑎 | , where |𝑎
Sol: we know that : unit vector of 𝑎 ⃗⃗⃗ | =
⃗⃗⃗

√12 + 12 + 12 = √3
⃗⃗⃗
𝑎 𝑖̂+𝑗 ̂ +𝑘 ̂ 1
⇒ 𝑎̂ = |𝑎⃗⃗⃗ | = = (𝑖̂ + 𝑗̂ + 𝑘̂ ). (OPTION- B)
√3 √3

⃗⃗⃗ = 𝑖̂ - 2𝑗̂ - 3𝑘̂ , find its direction cosines:


5- If 𝑎
1 −2 −3 −1 2 3 1 1 1 1 −2
(a) , , (b) , , (c) , , (d) ,
√3 √3 √3 √3 √3 √3 √14 √14 √14 √14 √14
−3
, .
√14

Sol: Scalar components of a unit vector represents direction cosines.

⃗⃗⃗
𝑎 𝑖̂−2𝑗̂ −3𝑘 ̂ 1
⇒ 𝑎̂ = |𝑎⃗⃗⃗ | = = (𝑖̂ -2 𝑗̂ -3 𝑘̂ ).
√14 √14

1 −2 −3
So direction cosines are , , . (OPTION- D)
√14 √14 √14

⃗⃗⃗ =3 𝑖̂ - 𝑗̂ - 𝑘̂ on the vector 𝑏


6- Find the projection of 𝑎 ⃗⃗ = 𝑖̂ +𝑗̂ +𝑘̂ .

1 1 1 1
(a) (b) (c) (d) .
√3 √2 3 2

⃗⃗⃗ · ⃗⃗⃗
𝑎 𝑏 ̂ ).(𝑖̂ +𝑗̂ +𝑘
(3 𝑖̂ − 𝑗̂ − 𝑘 ̂)
⃗⃗⃗ on the vector ⃗𝑏⃗ =
Sol: Projection of 𝑎 ⃗⃗⃗ |
= =
|𝑏 √1+1+1
3−1−1 1
= .
√3 √3

(OPTION-A)

Mathematics Page 183 of 260 XII


1
7- If 𝑎
⃗⃗⃗ is a non zero vector of magnitude and λ is a non zero scalar
3
then λ𝑎 ⃗⃗⃗ is a unit vector if :
(a) λ = 1 (b) λ = 3 (c) λ= -1 (d) λ=
- 3.

1
⃗⃗⃗ is a unit vector so |λ𝑎
Sol: λ𝑎 ⃗⃗⃗ | = 1 ⇒ λ |𝑎
⃗⃗⃗ | = 1 ⇒ λ. 3 = 1 ⇒ λ =
3 (OPTION-B).

8- If 𝜃 is the angle between the vectors 𝑎 ⃗⃗ then ⃗⃗⃗


⃗⃗⃗ and 𝑏 𝑎 · 𝑏 ⃗⃗⃗ > 0
only when :
𝜋 𝜋
(a) 0< 𝜃 < 𝜋 (b) 0≤ 𝜃 ≤ (c) 0< 𝜃 ≤ (d) 0≤ 𝜃 <
2 2
𝜋
.
2

⃗⃗⃗ · ⃗⃗⃗
Sol: 𝑎 𝑏 = |𝑎 ⃗⃗⃗ | cosϴ > 0 if cosϴ > 0 so 0≤ 𝜃 < 𝜋
⃗⃗⃗ |. |𝑏 2
(OPTION-D)

̂ + 𝑗̂ .(𝑘̂ × 𝑖̂ )+ 𝑘.
9- The value of 𝑖̂ .(𝑗̂ × 𝑘) ̂ (𝑗̂ × 𝑖̂ ) :
(a) 1 (b) - 1 (c) 3 (d) -3.

̂ + 𝑗̂ .(𝑘̂ × 𝑖̂ )+ 𝑘.
Sol: 𝑖̂ .(𝑗̂ × 𝑘) ̂ (𝑗̂ × 𝑖̂ ) = 𝑖̂. 𝑖̂ +𝑗̂.𝑗̂ +𝑘̂.(-𝑘̂) = 1 + 1 – 1 = 1
(OPTION- A)

10- Let the vectors 𝑎 ⃗⃗⃗ and 𝑏⃗⃗⃗ be such that |𝑎⃗⃗⃗ | = 2 and |𝑏 ⃗⃗ | = 1 then
⃗⃗ ) is a unit vector , if the angle between 𝑎
⃗⃗⃗ × 𝑏
(𝑎 ⃗⃗ is :
⃗⃗⃗ and 𝑏
𝜋 𝜋 𝜋 𝜋
(a) (b) (c) (d) .
6 4 3 2

⃗⃗⃗ ) is a unit vector so |𝑎


⃗⃗⃗ × 𝑏
Sol: (𝑎 ⃗⃗ | = 1 ⇒ |𝑎
⃗⃗⃗ × 𝑏 ⃗⃗⃗ | sinϴ = 1
⃗⃗⃗ |. |𝑏

𝜋
⇒ 2 × 1 ×sinϴ = 1 ⇒ sinϴ = ½ ⇒ ϴ = 6 (OPTION-A)

Mathematics Page 184 of 260 XII


11- If |𝑎 ⃗⃗ | = |𝑎
⃗⃗⃗ · 𝑏 ⃗⃗⃗ | then angle 𝜃 between the vectors 𝑎
⃗⃗⃗ × 𝑏 ⃗⃗⃗ and
⃗⃗ is :
𝑏
𝜋 𝜋 𝜋 𝜋
(a) (b) (c) (d)
6 3 4 2

Sol: Given that |𝑎 ⃗⃗ | = |𝑎


⃗⃗⃗ · 𝑏 ⃗⃗ | ⇒ |𝑎
⃗⃗⃗ × 𝑏 ⃗⃗⃗ | cosϴ =
⃗⃗⃗ |. |𝑏
|𝑎 ⃗⃗⃗ | sinϴ
⃗⃗⃗ |. |𝑏

⇒ cosϴ = sinϴ ⇒ tanϴ = 1


𝜋
⇒ 𝜃 = 4 (OPTION-C)

⃗⃗⃗ = 2 𝑖̂ - 𝑗̂ + 𝑘̂
12- Find the angle between the vectors 𝑎 and ⃗𝑏⃗ = 3 𝑖̂ + 𝑗̂ -
2𝑘̂ :
1 5 1
(a) cos −1 ( 6) (b) cos −1 ( 6) (c) cos −1 ( 3) (d) None of these.

⃗⃗⃗ · ⃗⃗⃗
Sol: using 𝑎 𝑏 = |𝑎 ⃗⃗⃗ | cosϴ
⃗⃗⃗ |. |𝑏

⇒ (2 𝑖̂ - 𝑗̂ + 𝑘̂ ). (3 𝑖̂ + 𝑗̂ -2𝑘̂) = √6. √14 cosϴ

3 3
⇒ 6 – 1 – 2 = 2√21 cosϴ ⇒ cosϴ = 2 ⇒ ϴ = cos −1 ( 2 )
√21 √21
[OPTION-D]

13- The value of 𝑖̂. 𝑖̂ - 𝑗̂.𝑗̂ - 𝑘̂. 𝑘̂ is :


(a) - 1 (b) 1 (c) - 2 (d) 2.

Sol: 𝑖̂. 𝑖̂ - 𝑗̂.𝑗̂ - 𝑘̂ . 𝑘̂ = 1 – 1 – 1 = - 1 (OPTION-A)

14- Find a vector in the direction of vector ⃗⃗⃗ 𝑎 = 𝑖̂ +3 𝑗̂ +4 𝑘̂ that has


magnitude 5 units.
1 5 −1
(a) 26(𝑖̂ +3 𝑗̂ +4 𝑘̂) (b) 26(𝑖̂ +3 𝑗̂ +4 𝑘̂) (c) 26(𝑖̂ +3 𝑗̂ +4 𝑘̂) (d)
√ √ √
3
(𝑖̂ +3 𝑗̂ +4 𝑘̂) .
√26

Mathematics Page 185 of 260 XII


⃗⃗⃗ = 𝑖̂ +3 𝑗̂ +4 𝑘̂ ⇒ |𝑎
Sol: Given 𝑎 ⃗⃗⃗ | = √12 + 32 + 42 = √26.

Vector in the direction of vector 𝑎


⃗⃗⃗ and has magnitude 5 units = 5.
⃗⃗⃗
𝑎
𝑎̂ = 5 |𝑎⃗⃗⃗ |

5
= (𝑖̂ +3 𝑗̂ +4 𝑘̂) [OPTION- B]
√26

15- Find a vector joining the points A(1,2,3) and B(3,2,5) directed from A
to B.
(a) 𝑖̂ +2 𝑗̂ +3 𝑘̂ (b) 2 𝑖̂ +2 𝑗̂ (c) 𝑖̂ +3 𝑗̂ (d) 2 𝑖̂ +2 𝑘̂

Sol: ⃗⃗⃗⃗⃗
𝐴𝐵 = 𝐵 𝐴 = (3𝑖̂ + 2𝑗̂ + 5𝑘̂ ) – (𝑖̂ + 2𝑗̂ + 3𝑘̂ ) = (2𝑖̂ + 2𝑘̂ )
⃗ - ⃗⃗⃗
[OPTION-D]

TWO MARKS QUESTIONS (VSA TYPE)

16- Show that the vectors 2 𝑖̂ - 𝑗̂ +3 𝑘̂ and - 4 𝑖̂ +2 𝑗̂ -6 𝑘̂ are collinear.

⃗⃗⃗ =2 𝑖̂ - 𝑗̂ +3 𝑘̂ and ⃗⃗⃗


Sol: Let 𝑎 𝑏 = - 4 𝑖̂ +2 𝑗̂ -6 𝑘̂

⃗⃗⃗ = - 4 𝑖̂ +2 𝑗̂ -6 𝑘̂ = - 2.( 2 𝑖̂ - 𝑗̂ +3 𝑘̂ ) = - 2 𝑎
Here 𝑏 ⃗⃗⃗

⃗⃗⃗
𝑏 is a scalar multiple of 𝑎
⃗⃗⃗ so given vectors are collinear.

17- Find the value of x for which x.( 2 𝑖̂ +3 𝑗̂ +4 𝑘̂ ) is a unit vector.

Sol: Given that x.( 2 𝑖̂ +3 𝑗̂ +4 𝑘̂ ) is a unit vector

⇒ |x. ( 2 𝑖̂ + 3 𝑗̂ + 4 𝑘̂ ) | = 1 ⇒ x √22 + 32 + 42 = 1
1
⇒x= .
√29

18- Find the area of triangle having the points P(2,3,4), Q(3, -2,1) and R(-
4,3,2) as its vertices.

Mathematics Page 186 of 260 XII


⃗⃗⃗⃗⃗ = 𝑖̂ -5 𝑗̂ -3 𝑘̂ and 𝑃𝑅
Sol: 𝑃𝑄 ⃗⃗⃗⃗⃗ = -6 𝑖̂ -2 𝑘̂

1
𝑖̂ 𝑗̂ 𝑘̂
Now area of triangle PQR = 2 | 𝑃𝑄 ⃗⃗⃗⃗⃗ | = 1 | 1
⃗⃗⃗⃗⃗ × 𝑃𝑅 −5 −3|
2
−6 0 −2
1
= | 2 { 10 𝑖̂ + 20 𝑗̂ − 30 𝑘̂} | = | 5 𝑖̂ + 10 𝑗̂ − 15 𝑘̂ | = 5√14 sq
units.

𝑎 , ⃗⃗⃗
19- If ⃗⃗⃗ ⃗⃗⃗ + ⃗𝑏⃗ + 𝑐⃗⃗ = 0, find the value of
𝑏 , 𝑐⃗⃗ are unit vectors such that 𝑎
⃗⃗⃗ +𝑏
⃗⃗⃗ · 𝑏
𝑎 ⃗⃗⃗ · 𝑐⃗⃗ +𝑐⃗⃗ · ⃗⃗⃗⃗
𝑎.

2
⃗⃗⃗ + ⃗𝑏⃗ + 𝑐⃗⃗ = 0 ⇒ |𝑎
Sol: Given that 𝑎 ⃗⃗⃗ + ⃗⃗⃗
𝑏 + 𝑐⃗⃗ | = (𝑎 ⃗⃗ + 𝑐⃗⃗ ).( 𝑎
⃗⃗⃗ + 𝑏 ⃗⃗⃗ + ⃗𝑏⃗ + 𝑐⃗⃗ )
=0

2
⃗⃗ | + |𝑐⃗⃗ |2 + 2.( 𝑎
⃗⃗⃗ |2 + |𝑏
⇒ |𝑎 ⃗⃗⃗ · ⃗𝑏⃗ +𝑏
⃗⃗⃗ · 𝑐⃗⃗ +𝑐⃗⃗ · 𝑎
⃗⃗⃗ ) = 0

⃗⃗⃗ · ⃗𝑏⃗ +𝑏
⇒ 1 + 1 + 1 + 2.( 𝑎 ⃗⃗⃗ · 𝑐⃗⃗ +𝑐⃗⃗ · 𝑎 ⃗⃗⃗ , ⃗𝑏⃗ , 𝑐⃗⃗ are
⃗⃗⃗ ) = 0 { ∙.∙ 𝑎
unit vectors }

⃗⃗⃗ · ⃗⃗⃗
⇒ 2.( 𝑎 𝑏 +𝑏⃗⃗⃗ · 𝑐⃗⃗ +𝑐⃗⃗ · 𝑎
⃗⃗⃗ ) = - 3

⃗⃗⃗ · ⃗⃗⃗
⇒𝑎 𝑏 +𝑏⃗⃗⃗ · 𝑐⃗⃗ +𝑐⃗⃗ · 𝑎
⃗⃗⃗ = - 3/2.

20- Find λ and 𝜇 if (2𝑖̂ +6 𝑗̂ +27 𝑘̂ )×( 𝑖̂ +λ 𝑗̂ +𝜇 𝑘̂ ) = 0


⃗⃗ .

Sol: Given that (2𝑖̂ +6 𝑗̂ +27 𝑘̂ )×( 𝑖̂ +λ 𝑗̂ +𝜇 𝑘̂ ) = 0


⃗⃗

𝑖̂ 𝑗̂ 𝑘̂
⇒ |2 6 ⃗⃗⃗ ⇒ 𝑖̂ (6𝜇 − 27𝜆)+ 𝑗̂ (27 - 2𝜇)+ 𝑘̂ (2λ – 6) = 0
27| = 0 ⃗⃗
1 𝜆 𝜇

⇒ On compairing 6𝜇 − 27𝜆 = 0, 27 - 2𝜇 = 0 and 2λ – 6 = 0

Mathematics Page 187 of 260 XII


⇒ λ = 3 and 𝜇 = 27/2.

21- Find the position vector of a point P which divides the line joining the
points A and B whose position vectors are 2𝑖̂ - 𝑗̂ +3 𝑘̂ and 3𝑖̂ +2 𝑗̂ - 𝑘̂
respectively, in the ratio 3 : 2 internally.

⃗⃗⃗ +𝑛 𝑎
𝑚𝑏 ⃗⃗⃗
⃗⃗⃗ =
Sol: Position vector of division point P = 𝑃 =
𝑚+𝑛
̂ )+2 (2𝑖̂ − 𝑗̂ +3 𝑘
3 (3𝑖̂ +2 𝑗̂ − 𝑘 ̂)
3+2

̂ )+ (4𝑖̂ −2 𝑗̂ +6 𝑘
(9𝑖̂ +6 𝑗̂ −3 𝑘 ̂)
= 5

⃗⃗⃗ = 1(13𝑖̂ +4 𝑗̂ +3 𝑘̂ ).
𝑃 5

22- Find the area of parallelogram whose adjacent sides are determined
by the vectors ⃗⃗⃗ = 𝑖̂ - 𝑗̂ +3 𝑘̂ and ⃗𝑏⃗ = 2 𝑖̂ - 7 𝑗̂ + 𝑘̂ .
𝑎

⃗⃗⃗ × ⃗𝑏⃗ | {where the vectors 𝑎


Sol: Area of parallelgram = |𝑎 ⃗⃗⃗ and ⃗𝑏⃗
represents the

Adjacent sides of
parallelogram}

𝑖̂ 𝑗̂ 𝑘̂
= ||1 −1 3| | = |20 𝑖̂ + 5 𝑗̂ − 5𝑘̂|
2 −7 1

= √202 + 52 + (−5)2

= √400 + 25 + 25

= √450

= 15√2 sq units.
Mathematics Page 188 of 260 XII
⃗⃗⃗ =2 𝑖̂ +3 𝑗̂ +4 𝑘̂ and 𝑏
23- If 𝑎 ⃗⃗⃗ = 3 𝑖̂ +2 𝑗̂ - 𝑘̂ then find the projection of
vector ⃗𝑏⃗ on 𝑎 ⃗⃗⃗ .

⃗⃗⃗ · 𝑎
𝑏 ⃗⃗⃗ ̂ ).(2𝑖̂ +3𝑗̂ +4𝑘
(3 𝑖̂+2 𝑗̂ − 𝑘 ̂)
Sol: Projection of ⃗𝑏⃗ on the vector 𝑎
⃗⃗⃗ = |𝑎
=
⃗⃗⃗ | √4+9+16
6+6−4 8
= = units.
√29 √29
24- Let 𝑎 ⃗⃗⃗ = 𝑖̂ +4 𝑗̂ +2 𝑘̂ and ⃗𝑏⃗ = 3 𝑖̂ -2 𝑗̂ +7 𝑘̂ and 𝑐⃗⃗ =2 𝑖̂ - 𝑗̂ +4 𝑘̂ .
Find a vector ⃗⃗⃗ 𝑑 which is perpendicular to both 𝑎 ⃗⃗⃗ and ⃗𝑏⃗ and 𝑐⃗⃗ ·
𝑑⃗⃗⃗ = 15.

⃗⃗⃗ is perpendicular to both 𝑎


Sol: Given that vector 𝑑 ⃗⃗ and 𝑐⃗⃗ ·
⃗⃗⃗ and 𝑏
⃗⃗⃗ = 15 .
𝑑

⃗⃗⃗ · 𝑎
⇒ 𝑑 ⃗⃗⃗ · ⃗𝑏⃗ = 0 and 𝑐⃗⃗ · 𝑑
⃗⃗⃗ = 0, 𝑑 ⃗⃗⃗ = 15 { let 𝑑
⃗⃗⃗ = x 𝑖̂ +y 𝑗̂ +z 𝑘̂ }

⇒ (x 𝑖̂ +y 𝑗̂ +z 𝑘̂ ). (𝑖̂ +4 𝑗̂ +2 𝑘̂ ) = 0, (x 𝑖̂ +y 𝑗̂ +z 𝑘̂ ).( 3 𝑖̂ -2 𝑗̂ +7 𝑘̂ ) = 0
and (2 𝑖̂ - 𝑗̂ +4 𝑘̂ ). (x 𝑖̂ +y 𝑗̂ +z 𝑘̂ ) = 15.

⇒ x + 4y + 2z = 0, 3x – 2y + 7z = 0 and 2x – y + 4z = 15.

160 −5 −70 1
⇒ On solving, we get x = , y= and z = . So ⃗⃗⃗
𝑑 =3 ( 160 𝑖̂ -5 𝑗̂ -70
3 3 3
𝑘̂ ).

𝑎 , ⃗𝑏⃗ and 𝑐⃗⃗ be three vectors such that |𝑎


25- Let ⃗⃗⃗⃗ ⃗⃗ | = 4, |𝑐⃗⃗ | = 5
⃗⃗⃗ | = 3, |𝑏
and each one of them being perpendicular to the sum of other two,
find |𝑎
⃗⃗⃗ + 𝑏⃗⃗ + 𝑐⃗⃗ | .

⃗⃗⃗ + ⃗⃗⃗
Sol: |𝑎 ⃗⃗⃗ + ⃗⃗⃗
𝑏 + 𝑐⃗⃗ | 2 = (𝑎 ⃗⃗⃗ + ⃗𝑏⃗ + 𝑐⃗⃗ )
𝑏 + 𝑐⃗⃗ ).( 𝑎

2
⃗⃗ | + |𝑐⃗⃗ |2 + 𝑎
⃗⃗⃗ |2 + |𝑏
= |𝑎 ⃗⃗⃗ .( ⃗⃗⃗
𝑏 + 𝑐⃗⃗ ) + ⃗𝑏⃗ .( 𝑎
⃗⃗⃗ + 𝑐⃗⃗ ) +
⃗⃗⃗ + ⃗⃗⃗
𝑐⃗⃗ .( 𝑎 𝑏)

Mathematics Page 189 of 260 XII


= 32 + 42 + 52 + 0 + 0 + 0 = 9 + 16 + 25 = 50.

⃗⃗⃗ + ⃗⃗⃗
⇒ |𝑎 𝑏 + 𝑐⃗⃗ | = 5√2 .

Mathematics Page 190 of 260 XII


CHAPTER - 11

THREE DIMENSIONAL GEOMETRY

THEORTICAL CONCEPTS WITH EXAMPLES

1. Distance between Two Points Let P(x1, y1, z1) and Q(x2, y2, z2) be two
given points. The distance between these points is given by PQ √(x2 –
x1) 2 + (y2 – y1) 2 + (z2 – z1) 2
The distance of a point P(x, y, z) from origin O is OP = √x 2 + y2 + z2 .

2.Direction Cosines If a directed line segment makes angle α, β and γ with


OX , OY and OZ respectively, then cos α, cos β and cos γ are called direction
cosines of line and it is represented by l, m, n. i.e., l = cos α, m = cos β and n
= cos γ l2+m2+n2=1

Ex: Direction cosines of x axis are <1,0,0 >.

3. If θ is the angle between two lines having direction cosines l1, m1, n1 and
l2, m2, n2, then cos θ = l1l2 + m1m2 + n1n2

(a) Lines are parallel, if l1 / l2 = m1 / m2 = n1 / n2

(b) Lines are perpendicular, if l1l2 + m1m2 + n1n2 =0

4. If P(x1, y1, z1) and Q(x2, y2, z2) are two points, such that the direction ratios
of PQ are a,b,c. Then

a=x2– x1 , b= y2 – y1 , c= z2 – z1 .

5. If θ is the angle between two lines whose direction ratios are proportional
to a1, b1, c1 and a2, b2, c2 respectively, then the angle θ between them is
given by

cos θ = a1a2 + b1b2 + c1c2 / √a12 + b12 + c12 √ a22 + b22 + c22

Case 1: Lines are parallel, if a1 / a2 = b1 / b2 = c1 / c2


Mathematics Page 191 of 260 XII
𝑥−8 𝑦−4 𝑧−10 𝑥−15 𝑦−5 𝑧−5
Ex: 3
= 5
= −7
= and 6
= 10
= −14
are parallel lines

Case 2: Lines are perpendicular, if a1a2 + b1b2 + c1c2= 0.


𝑥−5 𝑦−3 𝑧−10 𝑥−15 𝑦−5 𝑧+5
Ex: 3
= 5
= −17
= and 6
= 10
= 4
are perpendicular lines.

6. Straight Line in Space

(A) Equation of a straight line passing through a fixed point A(x1, y1, z1) and
having direction ratios a, b, c is given by x – x1 / a = y – y1 / b = z – z1 / c, it is
also called the symmetrically form of a line.

(B)Equation of a straight line joining two fixed points A(x1, y1, z1) and B(x2,
y2, z2) is given by

x – x1 / (x2 – x1) = y – y1 /( y2 – y1) = z – z1 / (z2 – z1)

(C ) Vector equation of a line passing through a point with position vector


𝑎and parallel to vector 𝑏⃗ is 𝑟 = 𝑎+ λ𝑏⃗, where A, is a parameter.

(D) Vector equation of a line passing through two given points having
⃗⃗ is 𝑟= = 𝑎 + λ ( 𝑏⃗– 𝑎) , where λ is a parameter.
position vectors 𝑎and𝑏

7. Skew Lines Two straight lines in space are said to be skew lines, if they are
neither parallel nor intersecting.

8. The shortest distance between two skew lines

(A)Vector Form

We shall consider two skew lines, say l1 and l2 and we are to calculate the
distance between them. The equations of the lines are:

𝑟1 =𝑎1 +⋋𝑏⃗1

Mathematics Page 192 of 260 XII


𝑟2 =𝑎2 +µ𝑏⃗2

P = 𝑎1 is a point on line l1 and Q = 𝑎2 is a point on line l1. The vectro


from P to Q will be 𝑎2 –⃗⃗⃗𝑎1 . The unit vector normal to both the lines is given by,
(𝑏⃗1 ×𝑏⃗2 )/| 𝑏⃗1 ×𝑏⃗2 |

Then, the shortest distance between the two skew lines will be the projection
of PQ on the normal, which is given by

d=|(𝑎2 –𝑎1 ).( 𝑏⃗1 ×𝑏⃗2 )|/| 𝑏⃗1 ×𝑏⃗2 |

where d measures the distance or the length of the perpendicular.

(B)CARTESIAN FORM:
Let us consider two lines whose equations are given by:
(x–x1)/a1=(y–y1)/b1=(z–z1)/c1
(x–x2)/a2=(y–y)/b2=(z–z2)/c2
Then the shortest distance between these lines, when calculated using the
Cartesian equations, is given by

𝑥2 − 𝑥1 𝑦2 − 𝑦1 𝑧2 − 𝑧1
d =| 𝑎1 𝑏1 𝑐1 | / [(b1 c2–b2 c1)2+(c1 a2–a2 c2)2+(a1 b2–b2
𝑎2 𝑏2 𝑐2
a1)2]1/2

SOLVED QUESTIONS FOR CBSE EXAMINATION

Q 1: What is the equation of a line in three-dimensional geometry,


passing through the points (1, 3, -2), and (-1, 4, 3)?
Solution:
The given point are (1, 3, -2), and (-1, 4, 3).
The equation of a line passing through the two points is r = a + λ(b - a).
r = i+3j-2k +⋋(-2i+j+5k)
Let 𝑟 = xi+yj+zk be the position vector than
xi+yj+zk = i+3j-2k +⋋(-2i+j+5k)
Mathematics Page 193 of 260 XII
comparing the coefficients of I,j,k on both sides we have
x = 1-2⋋ ,y = 3+⋋,z= -2+5⋋
(x−1)/−2 = ⋋ , (y−3)/1 =⋋, (z+2)/5 = ⋋
Therefore, the equation of the line passing through the two point
is (x−1)/−2=(y−3)/1=(z+2)/5

Q 2: Find the direction ratios and direction cosines of a point (4, 5, -2) in 3D
geometry.
Solution:
The given point is (4, 5, -2) is represented as a vector 𝑎= 4𝑖̂ + 5𝑗̂ − 2𝑘̂
Here we have |a| = √42+52+(−2)2=√16+25+4=√45=3√5
Direction Ratios's: (a, b, c) = 4, 5, -2
Direction Cosines: 4/3√5, 5/3√5, −2/3√5

Q3:Show that the points A (2, 3, – 4), B (1, – 2, 3) and C (3, 8, – 11) are
collinear.
Solution: Direction ratios of line joining A and B are 1 – 2, – 2 – 3, 3 + 4 i.e.,
– 1, – 5, 7.
The direction ratios of line joining B and C are 3 –1, 8 + 2, – 11 – 3, i.e., 2, 10,
– 14.
It is clear that direction ratios of AB and BC are proportional, hence, AB is
parallel to BC.
But point B is common to both AB and BC. Therefore, A, B, C are collinear.

Q 4 :Find the equation of line in vector and Cartesian form which passes
through the point (1,2,3) and is parallel to the vector 3i+2j-2k .
Solution:Let 𝑎 be the position vector of the point (1,2,3)
Then 𝑎 =𝑖̂+2𝑗̂ +3𝑘̂
Now the equation of line passing through the point having position vector 𝑎
and parallel to vector 𝑏⃗ is 𝑟 = 𝑎 + ⋋b =( 𝑖̂+2𝑗̂+3𝑘̂ ) + ⋋(3𝑖̂+2𝑗̂-2𝑘̂)

Cartesian form: Let 𝑟 = x𝑖̂+y𝑗̂+z𝑘̂ be the position vector than


xi+yj+zk = (𝑖̂+2𝑗̂+3𝑘̂) + ⋋(3𝑖̂+2𝑗̂-2𝑘̂)
comparing the coefficients of I,j,k on both sides we have
x= 1+3⋋, y= 2+2⋋, z= 3-2⋋
ie( x-1)/3 = (y-2)/2 = (z-3)/ -2
Mathematics Page 194 of 260 XII
Q 5 : Find the value of p so that the lines
1−𝑥 7𝑦−14 5𝑧−10 7−7𝑥 𝑦−5 6−𝑧
= = and = = are perpendicular to each other.
3 2𝑝 11 3𝑝 1 5
1−𝑥 7𝑦−14 5𝑧−10
Solution : = = can be written as
3 2𝑝 11
𝑥−1 𝑦−2 𝑧−2
=2𝑝/7 = 11/5
−3
dr’s of this line are : -3,2p/7,11/5
7−7𝑥 𝑦−5 6−𝑧 𝑥−1 𝑦−5 𝑧−6
= = can be written as −3𝑝/7 = =
3𝑝 1 5 1 −5
dr’s of this line are: -3p/7,1,-5
since Lines are perpendicular therefore a1a2 + b1b2 + c1c2= 0
 -3(-3p/7)+(2p/7) x 1 + (11/5) x (-5) = 0
 (9p/7) +(2p/7) = 11
 11p/7 = 11
 p=7

Q 6: Find the shortest distance between the lines : 𝑟 = (𝑖̂+2𝑗̂ +𝑘̂ )+ ⋋(𝑖̂-𝑗̂+𝑘̂)
and

𝑟 = (2𝑖̂-𝑗̂ -𝑘̂ )+ µ(2𝑖̂+𝑗̂ +2𝑘̂)

Solution: Given lines are :


𝑟 = (𝑖̂+2𝑗̂ +𝑘̂ )+ ⋋(𝑖̂-𝑗̂+𝑘̂ ) ……………………..(1)
𝑟 = (2𝑖̂-𝑗̂ -𝑘̂ )+ µ(2𝑖̂+𝑗̂ +2𝑘̂) ……………………(2)
Comparing the equation (1) and (2) with 𝑟 = a1+⋋b1 and 𝑟 = a2+µb2
a 1 = (𝑖̂+2𝑗̂ +𝑘̂ ) a 2= (2𝑖̂-𝑗̂ -𝑘̂ )
b1=(i-j+k) b2 = (2i+j+2k)
now a2 – a1 = (2𝑖̂-𝑗̂ -𝑘̂ ) - (𝑖̂+2𝑗̂ +𝑘̂ ) = i -3j -2k
𝑖 𝑗 𝑘
b1 x b2 = 1 −1 1 = (-2-1)i – (2-2)j +(1+2)k = -3i +3k
2 1 2
I b1 x b2I = √(-3)2+ (3)2 = 3√2
Shortest distance d=|(𝑎2 –𝑎1 ).( 𝑏⃗1 ×𝑏⃗2 )|/| 𝑏⃗1 ×𝑏⃗2 |
Putting the values of a2 – a1 , b1 x b2, I b1 x b2I in this formula we get
d = ( i -3j -2k ) .( -3i +3k) / 3√2 = I(-3 -0-6)I/3√2 = 9/3√2 = 3√2 /2

Mathematics Page 195 of 260 XII


Q 7: Find the cartesian and vector equation of the line passing through
(1,2,-4) and perpendicular to the lines :
𝑥−8 𝑦+19 𝑧−10 𝑥−15 𝑦−29 𝑧−5
= = and = = .
3 −16 7 3 8 −5
Solution : Equation of any line through the point ( 1, 2,-4 ) is
𝒙−𝟏 𝑦−2 𝑧+4
= 𝑏 = 𝑐
𝒂
where a, b, and c are direction ratios of line (i) .
Now the line ( i) is perpendicular to the lines
𝑥−8 𝑦+19 𝑧−10 𝑥−15 𝑦−29 𝑧−5
= −16 = 7 = and 3 = 8 = −5
3
Having direction ratios 3, -16,7and 3,8,-5 respectively.Therefore
3a-16b + 7c = 0 ……(ii)(by perpendicular condition
a1a2+ b1b2+ c1c2=0)
3a + 8b -5c = 0 …...(iii)
Solving (ii) and (iii) by cross- multiplication method , We have
𝑎 𝑏 𝑐 𝑎 𝑏 𝑐
= 21+15 = 24+48 => = 36 = 72
80−56 24
𝑎 𝑏 𝑐
 =3=6
2
𝑎 𝑏 𝑐
Let 2 = 3 = 6 = ⋋ => a= 2⋋ , b = 3 ⋋ and c = 6⋋
Direction ratios of required line are :2,3,6
𝒙−𝟏 𝑦−2 𝑧+4
Cartesian equation of required line is 𝟐 = 3 = 6

The equation of required line which passes through point ( 1,2,-4)and


parallel to vector 2i +3j + 6k is 𝑟 = ( i+ 2j – 4k ) + ⋋ ( 2i + 3j +6k).

MULTIPLE CHOICE QUESTIONS

1. The co-ordinates of the foot of the perpendicular drawn from the


point ( 2,-3, 4) on the y- axis is

a. ( 2, 3, 4)
b. ( -2, -3, -4 )
c. (0, -3, 0 )
d. ( 2, 0, 4 )

2. The line of shortest distance between two skew lines is

Mathematics Page 196 of 260 XII


a. Parallel to both the lines
b. Perpendicular to both the lines
c. Coincide with both the lines
d. None of the above

3. The direction cosines of the vector ( 2 i + 2 j – k ) are :

a. 2/3, 2/3, -1/3


b. 1 /2,1 /2 , -1
c. 2/3, 1/ 2 , -1
d. None of these

4. If the direction cosines of a line are k, k, k then k is :

a. k>0
b. 0<k<1
c. k=1
d. k = 1/√3 or -1/√3

5. Distance of the point ( 3, 4, 5 ) from z- axis is :

a. 3
b. 4
c. 5
d. None of the above

6. The equation of x – axis in space is :

a. x = 0, y = 0
b. x = 0, z = 0
c. x=0
d. y=0, z=0

7. If a line makes angle π/3 and π/4 with x– axis and y-axis respectively ,
then the angle made by the line with z –axis is :

a. π/2
Mathematics Page 197 of 260 XII
b. π/3
c. π/4
d. 15π/12

𝑥−3 𝑦−2 𝑧−1


8. The straight line = = is :
3 1 0

a. Parallel to x- axis
b. Parallel to y- axis
c. Parallel to z- axis
d. Perpendicular to z- axis

9. The distance of a point P ( a, b, c ) from x-axis is :

a. √(a2+b2)
b. √(b2+c2)
c. (a2+b2)
d. (b2+c2)

𝑥−2 𝑦−3 4−𝑧 𝑥−1 𝑦−4 𝑧−5


10. The lines 1 = 1 = 𝑘 and 𝑘 = = are mutually
2 −2
perpendicular , if the value of k is :

a. -2/3
b. 2/3
c. -2
d. 2

ANSWERS

1. c
2. b
3. a
4. d
5. c
6. d
7. b

Mathematics Page 198 of 260 XII


8. d
9. b
10. a
Detailed solution

1.on y-axis x=0, z=0 so the right answer is (0,-3,0)

2.The line of shortest distance between two skew lines is Perpendicular to


both the lines

3. The direction ratios of the vector ( 2 i + 2 j – k ) are : 2,2,-1


The direction cosines of the vector ( 2 i + 2 j – k ) are : 2/√{22+22+(-1)2},
2//√{22+22+(-1)2},-1//√{22+22+(-1)2} ie. 2/3, 2/3, -1/3

4. if l,m,n are direction cosines then l2+m2+n2 =1


K2+k2+k2 =1 or 3k2 =1 ie k = ±1/√3

5.co-ordinate of the foot of perpendicular from P on z-axis is Q(0,0,5)


So PQ = √(3-0)2+((4-0)2+(5-5)2 = √25 = 5 unit

6. The equation of x – axis in space is y=0, z=0

7. l= cos π/3 = ½ , m= cos π/4 = 1/√2


We know that if l,m,n are direction cosines then l2+m2+n2 =1
Putting the values of l and m we get 1 /4 +1/2 + n 2 = 1
Or n2 = ¼ or n= ±1/2
n =1/2 implies cosƳ= cos π/3 ie Ƴ= π/3

𝑥−3 𝑦−2 𝑧−1


8. 3 = 1 = 0 direction ratios of this line are 3,1,0 and direction ratios
of z-axis are 0,0,1
Sum of the product of direction ratios is 3 x0 +1x0 +0x1 = 0 therefore this
line is perpendicular to z-axis.

9. co-ordinate of the foot of perpendicular from P on x-axis is Q(a,0,0)


So PQ = √(a-a)2+((b-0)2+(c-0)2 = √b2+c2
Mathematics Page 199 of 260 XII
𝑥−2 𝑦−3 4−𝑧 𝑥−1 𝑦−4 𝑧−5
10.since the lines = = and = = are mutually
1 1 𝑘 𝑘 2 −2
perpendicular therefore sum the products of direction ratios will be zero ie
a1a2 + b1b2 + c1c2= 0.
1xk + 1x2 +(-k)x(-2) = 0 or 3k = -2 or k = -2/3

CASE STUDY BASED QUESTIONS

Two motorcycles X and Y are running at the speed more than allowed speed
on the road along the lines 𝑟= ⋋(𝑖̂ +2𝑗̂-𝑘̂) and 𝑟= (3𝑖 ̂ +𝑗̂+𝑘̂)
̂ +3𝑗̂)+ µ(2𝑖
respectively.
Based on the above information,solve the following questions.

Q1.The Cartesian equation of the line along which motorcycle X is running ,is
:
𝑥+1 𝑦+1 𝑧−1
a. 1 = 2 = −1
𝑥 𝑦 𝑧
b. = 2 = −1
1

Mathematics Page 200 of 260 XII


𝑥 𝑦 𝑧
c. = =
1 2 1
d. None of these

Q2.The direction cosines of line along which motorcycle X is running,are :


a.<1,-2,1>
b.<1,2,-1>
c.< 1/√6 ,-2/√6,1/√6 >
d. < 1/√6 ,2/√6,-1/√6 >
Q.3 The direction ratios of the line along which motor cycle Y is running ,are
:
a.< 1,0,2>
b. <2,1,0>
c.<1,1,2>
d.<2,1,1>

Q.4 The shortest distance between the given lines is :


a.4 units
b.2√3 units
c.3√2 units
d.zero

Q.5 The motorcycles will meet with an accident at the point:


a.(-1,1,2)
b.(2,1,-1)
c.(1,2,-1)
d.none of these
Detailed Solution:
Answer 1. The line along which motorcycle X is running,is 𝑟= ⋋(𝑖̂ +2𝑗̂-𝑘̂)
which can rewrite as
(xi +yj+zk) = ⋋i +2⋋j -⋋k
Or x= ⋋ , y= 2⋋, z= -⋋
𝑥 𝑦 𝑧
Or 1 =2 =−1 = ⋋
So option (b) is correct.

Mathematics Page 201 of 260 XII


𝑥 𝑦 𝑧
Answer 2. The line along which motorcycle X is = = .
1 2 −1
Clearly direction ratios of this line are: 1,2,-1
√ 12 + 22+(-1)2 = √6
So direction cosines of this line are < 1/√6 ,2/√6,-1/√6 >
So option (d) is correct.

̂ +3𝑗̂)+ µ(2𝑖
Answer 3: The line along which motorcycle B is running is 𝑟= (3𝑖 ̂
+𝑗̂+𝑘̂)
̂ +𝑗̂+𝑘̂
This line is parallel to 2𝑖
Therefore direction ratios are 2,1,1
So option (d) is correct.

Answer 4. Shortest distance d=|(a 2–a 1).(b 1×b 2)|/|b 1×b 2|


The line along which motorcycle X is running,is 𝑟= ⋋(𝑖̂ +2𝑗̂-𝑘̂)
The line along which motorcycle B is running is 𝑟= (3𝑖 ̂ +𝑗̂+𝑘̂)
̂ +3𝑗̂)+ µ(2𝑖
comparing with 𝑟 = a1+⋋b1 and 𝑟 = a2+µb2
̂ +3𝑗̂)
a2 – a1 = (3𝑖
𝑖 𝑗 𝑘
b1 x b2 = 1 2 −1 = 3i-3j-3k
2 1 1
̂ +3𝑗̂). (3i-3j-3k) = 9-9 =0
(a2 – a1) .( b1 x b2) = (3𝑖
Hence shortest distance between the given line is zero.
So option (d) is correct.

Answer 5.Since the point (1,2,-1) satisfy both the equations of lines
therefore point of intersection of given lines is (1,2,-1).
So the motorcycles will meet with an accident at the point (1,2,-1)
So option (c ) is correct.

ASSERTION AND REASON TYPE QUESTIONS:


Directions :( Q NOS 1-2) This is Assertion-Reason type questions. This
questions contains two statements.
Statement-1 (Assertion) and Statement-2 (Reason) This question has four
alternative choices, only one of which is the correct answer. You have to
select the correct choice as given below.

Mathematics Page 202 of 260 XII


(a) Statement- 1 is true, Statement- 2 is true; Statement-2 is a correct
explanation for Statement- 1
(b) Statement-1 is true, Statement-2 is true; Statement-2 is not a correct
explanation for Statement- 1
(c) Statement 1 is true, Statement- 2 is false
(d) Statement- 1 is false, Statement- 2 is true

𝑥−5 𝑦+4 𝑧−6


Q1: Statement-1: If the Cartesian equation of a line is 3 = 7 = 2 , then
its vector form is 𝑟 = 5i – 4j +6k + ⋋ ( 3i + 7j + 2k).
Statement -2 : The Cartesian equation of a line which passes through the
𝑥+3 𝑦−4 𝑧+8 𝑥+3 𝑦−4
point ( -2,4,-5) and parallel to the line given by 3 = 5 = 6 is −2 = 4 =
𝑧+8
.
−5
Answer: (c) Statement 1 is true, Statement- 2 is false.

Q2: Statement-1: The pair of lines given by 𝑟 = i – j + ⋋ ( 2i +k) and 𝑟 = 2i -k


+ µ ( i + j -k) intersect .
Statement -2 : Two lines intersect each other ,if they are not parallel and
shortest distance =0
Answer: (a) Statement- 1 is true, Statement- 2 is true; Statement-2 is a
correct explanation for Statement- 1
Detailed solution: Shortest distance d=|(𝑎2 –𝑎1).( 𝑏⃗1 ×𝑏⃗2 )|/| 𝑏⃗1 ×𝑏⃗2 |
(𝑎2 –𝑎1 )= (2i –k) – (i – j) = i+j-k
𝑖 𝑗 𝑘
(𝑏⃗1 ×𝑏⃗2 ) = 2 0 1 = - i+3j+2k
1 1 −1
⃗⃗⃗⃗ 2 –𝑎1 ).( 𝑏⃗1 ×𝑏⃗2 )=( i+j-k).( - i+3j+2k) = -1+3-2 = 0
(𝑎

Therefore shortest distance is zero.

QUESTIONS FOR JEE MAINS/ADVANCED

Q1.The projection of any line on co-ordinate axes be ,respectively 3,4,5


then its length is ………………….
Solution: Let the line segment be AB,then as given ABcosa = 3, ABcosb = 4,
ABcosc = 5 where a,b,c are the angles made by the line with the axes.
Mathematics Page 203 of 260 XII
AB2(cos2a +cos2b+ cos2c) = 32+42+52
Or AB2 =50
Or AB= 5√2

Q2.The equation of the straight line passing through the point (a,b,c) and
parallel to z-axis, is …………………. .
Solution: The direction cosines of line parallel to z-axis are <0,0,1 >
𝑥−𝑎 𝑦−𝑏 𝑧−𝑐
Since, the line passes through ( a, b, c ), Hence the line will be = =
0 0 1

Q3.The acute Angle between the line joining the points (2,1,-3),(-3,1,7) and a
𝑥−1 𝑦 𝑧+3
line parallel to 3 = 4 = 5 through the point(1,0,4) is …………………….. .
Solution: Direction ratio of the line joining the point (2,1,-3)and (-3,1,7) are -
3-2,1-1,7-(-3) ie -5,0,10 .
𝑥−1 𝑦 𝑧+3
Direction ratio of the line parallel to line 3 = 4 = 5 are 3,4,5 .
If θ is the angle between two lines whose direction ratios are proportional to
a1, b1, c1 and a2, b2, c2 respectively, then the angle θ between them is given
by
cos θ = a1a2 + b1b2 + c1c2 / √a12 + b12 + c12 √ a22 + b22 + c22
a1 = -5, b1= 0, c1= 10
a2= 3, b2 = 4, c2 = 5
a1a2 + b1b2 + c1c2 = -5x3 +0x4+10x5 = -15+50 = 35
√a12 + b12 + c12 = √-52 + 02 + 102 = √125 = 5√5
√ a22 + b22 + c22 = √ 32 + 42 + 52 =√50 = 5√2
Putting these values in formula we get cos θ = 35/25√10 or θ= cos -1 (7/5√10)

Q.4A line makes angles a,b and c with the coordinate axes. Ifa+b=90∘,then
find c.
Solution:
Let a line makes the angles a,b,c with three axes then cos2a +cos2b+ cos2c=1
cos2a +cos2(90o-a)+ cos2c=1
or cos2a +sin2a+ cos2c=1
or 1+cos2c=1
or cos2c=0
or cosc=0
Mathematics Page 204 of 260 XII
c= 90

Chapter - 12
Linear Programming
Problems which seek to maximise (or, minimise) profit (or, cost) form a
general class of problems called optimisation problems. Thus, an
optimisation problem may involve finding maximum profit, minimum cost,
or minimum use of resources etc. A special but a very important class of
optimisation problems is linear programming problem. Linear programming
problems are of much interest because of their wide applicability in
industry, commerce, management science etc.
In this chapter, we shall study some linear programming problems and their
solutions by graphical method only.
Linear Programming Problem and its Mathematical Formulation
Optimal value (maximum or minimum value) of a linear function.
Objective function: is a linear function which has to be maximised or
minimized and given by Z = ax + by, where a, b are constants and x, y are
variables. The variables are non-negative and satisfy a set of linear
inequalities (called linear constraints).
The term linear implies that all the mathematical relations used in the
problem are linear relations while the term programming refers to the
method of determining a particular programme or plan of action.
Constraints: The linear inequalities or equations or restrictions on the
variables of a linear programming problem are called constraints. The
conditions x ≥ 0, y ≥ 0 are called non-negative restrictions.

Mathematics Page 205 of 260 XII


Optimisation problem A problem which seeks to maximise or minimise a
linear function (say of two variables x and y) subject to certain constraints as
determined by a set of linear inequalities is called an optimisation problem.
Feasible region: The common region determined by all the constraints
including non-negative constraints x, y ≥ 0 of a linear programming problem
is called the feasible region (or solution region) for the problem.
Infeasible region: The region other than feasible region is called an
infeasible region.
Feasible solutions Points within and on the boundary of the feasible region
represent feasible solutions of the constraints. Any point outside the
feasible region is called an infeasible solution.
Optimal (feasible) solution: Any point in the feasible region that gives the
optimal value (maximum or minimum) of the objective function is called an
optimal solution.

The following Theorems are fundamental in solving linear programming


problems:
Theorem 1 Let R be the feasible region (convex polygon) for a linear
programming problem and let Z = ax + by be the objective function. When Z
has an optimal value (maximum or minimum), where the variables x and y
are subject to constraints described by linear inequalities, this optimal value
must occur at a corner point* (vertex) of the feasible region.
Theorem 2 Let R be the feasible region for a linear programming problem,
and let Z = ax + by be the objective function. If R is bounded**, then the

Mathematics Page 206 of 260 XII


objective function Z has both a maximum and a minimum value on R and
each of these occurs at a corner point (vertex) of R.
Corner Points: A corner point of a feasible region is a point in the region
which is the intersection of two boundary lines.
Bounded Region: A feasible region of a system of linear inequalities is said
to be bounded if it can be enclosed within a circle. Otherwise, it is called
unbounded. Unbounded means that the feasible region does extend
indefinitely in any direction.
Corner Point Method. The method comprises of the following steps:
Step 1: Formulate the given LPP in mathematical form if it is not so.
Step 2: Convert all inequations into equations and draw their graphs. To
draw the graph of a linear equation, put y=0 in it and obtain a point on x-
axis. Similarly, by putting x=0 obtain a point on y-axis. Join these two points
to obtain the graph representing the equation.
Step 3: Determine the region represented by each inequation. To determine
the region represented by an inequation replace x and y both by zero, if the
inequation reduces to a valid statement, then the region containing the
origin is the region represented by the given inequation. Otherwise, the
region not containing the origin is the region represented by the given
inequation.
Step 4: Find the feasible region of the linear programming problem and
determine its corner points (vertices) either by inspection or by solving the
two equations of the lines intersecting at that point.
Step 5: Evaluate the objective function Z = ax + by at each corner point of
convex polygon. Let M and m, respectively denote the largest and smallest

Mathematics Page 207 of 260 XII


values of these points.
Step 6:
(i)When the feasible region is bounded, M and m are the
maximum and minimum values of Z, are optimal solution of given
LPP.
(ii) In case, the feasible region is unbounded, we have:
(a) M is the maximum value of Z, if the open half plane
determined by
ax + by > M has no point in common with the feasible region.
Otherwise, Z has no maximum value.
(b) Similarly, m is the minimum value of Z, if the open half plane
determined by ax + by < m has no point in common with the
feasible region. Otherwise, Z has no minimum value.

LINEAR PROGRAMMING

MCQS(1 MARKS)

1. The optimum value of the objective function is attained at the


points
A. given by the intersections of inequalities with the 𝑥- axis only.
B. given by the intersections of inequalities with 𝑥- axis and 𝑦-
axis only.
C. given by the corner points of the feasible region.
D. none of these.

2. Objective function of an LPP is …..


A. a constraints
Mathematics Page 208 of 260 XII
B. a function which is to be optimized.
C. A relation between variables.
D. none of these.

3. Which of the following is correct?


A. LPP always has a unique solution.
B. every LPP has a unique solution.
C. LPP admits two optimal solution.
D. if an LPP admits two optimal solution, then it has infinitely
many optimal solution.

4. The solution set of the inequation 𝑥 + 2𝑦 > 3 is …..


A. half plane containing the origin.
B. open half plane not containing the origin.
C. first quadrant
D. none of these.

5. If the feasible region for an LPP is ___________, then the optimal


value of the objective function 𝑍 = 𝑎𝑥 + 𝑏𝑦 may or may not exist.
A. bounded.
B. unbounded.
C. in circle form.
D. in pentagon form.

6. An LPP is one that is concerned with finding _________________of


a linear function called_________________function of several
variables (say 𝑥 and 𝑦), subject to the conditions that the variables
are ____________ and satisfy set of linear inequalities called linear
constraints.
A. objective, optimal value, negative.
B. optimal value, objective, negative.
C. optimal value, objective, non-negative.
Mathematics Page 209 of 260 XII
D. objective, optimal value, non – negative.
7. The corner points of the feasible region determined by the system
of linear constraints are (0, 0), (0, 40), (20, 40), (60, 20) and
(60, 0). The objective function is 𝑍 = 4𝑥 + 3𝑦.
Compare the quantity in column A and column B.
Column A Column B
Maximum of 𝑍 325
A. The quantity in column A is greater.
B. The quantity in column B is greater.
C. Two quantities are equal.
D. The relationship cannot be determined on the basis of the
information supplied.

8. The feasible solution for an LPP is shown in given figure. Let, 𝑍 =


3𝑥 − 4𝑦 be the objective function. Minimum of 𝑍 occurs at

A. (0, 0)
B. (0, 8)
C. (5, 0)
Mathematics Page 210 of 260 XII
D. (4, 10)

9. Refer to Q2, maximum of 𝑍 occurs at


A. (5, 0)
B. (6, 5)
C. (6, 8)
D. (4, 10)

10. Refer to Q2, maximum of 𝑍 + minimum of 𝑍 is equal to


A. 13
B. 1
C. −1
D. −17
11. The feasible region for an LPP is shown in the given figure. Let, 𝐹 =
3𝑥 − 4𝑦 be the objective function. Maximum value of 𝐹 is

A. 0
B. 8
C. 12
D. −18
12. Refer to Q5, minimum value of 𝐹 is
A. 0
B. −16
Mathematics Page 211 of 260 XII
C. 12
D. does not exist.
13. The corner points of the feasible region determined by the system
of linear constraints are (0, 2), (3, 0), (6, 0), (6, 8) and (0, 5). The
objective function is 𝐹 = 4𝑥 + 6𝑦.
The minimum value of 𝐹 occurs at
A. (0, 2) only
B. (3, 0) only
C. the mid-point of the line segment joining the points (0, 2) and
(3, 0)
D. any point on the line segment joining the points (0, 2) and
(3, 0)
14. Refer to Q7, maximum of 𝐹 − minimum of 𝐹 is equal to
A. 60
B. 48
C. 42
D. 18
15. Corner points of the feasible region determined by the system of
linear constraints are (0, 3), (1, 1) and (3, 0). The objective function
is 𝑍 = 𝑝𝑥 + 𝑞𝑦, where 𝑝, 𝑞 > 0. Condition on 𝑝 and 𝑞 so that the
minimum of 𝑍 occurs at (3, 0) and (1, 1) is
A. 𝑝 = 2𝑞
𝑞
B. 𝑝 = 2
C. 𝑝 = 3𝑞
D. 𝑝 = 𝑞
16. Feasible region is the set of points which satisfy
A. The objective functions
B. Some the given constraints
C. All of the given constraints
D. None of these

17. The optimal value of the objective function is attained at the


A. points inside the feasible region
Mathematics Page 212 of 260 XII
B. points at the boundary line of the feasible region
C. vertex point of the boundary of the feasible region
D. None of these

18. Objective function of a linear programming problem is


A. a constraint
B. a function that has to be optimized
C. A relation between the variables
D. None of these
19. Corner points of the feasible region for an LPP are
(0,2),(3,0),(6,0),(6,8) and (0,5). Let F = 4x + 6y be objective function.
The minimum value of F occurs at :
A. (0,2) only
B. (3,0) only
C. The mid point of the line segment joining the points (0,2) and
(3,0) only
D. Any point on the line segment joining the points (0,2) and
(3,0).
20. The solution set of the inequality X + 3Y ≥ 6 is :
A. half plane not containing the origin .
B. whole xy – plane except the points lying on the line x +3y = 6.
C. open half plane containing the origin .
D. none of these.

SECTION B

CASE BASED QUESTIONS

CASE STUDY -1

I. Read the following text and answer the following questions on the
basis of the same:
An aeroplane can carry a maximum of 200 passengers. A profit of Rs.
1000 is made on each executive class ticket and a profit of Rs. 600 is
Mathematics Page 213 of 260 XII
made on each economy class ticket. The airline reserves at least 20
seats for executive class. However, at least 4 times as many
passengers prefer to travel by economy class than by executive class.
It is given that the number of executive class ticket is 𝑥 and that of
economy class tickets is 𝑦.

1. The maximum value of 𝑥 + 𝑦 is ……


A. 100
B. 200
C. 20
D. 80

2. The relation between 𝑥 and 𝑦 is……..


A. 𝑥 < 𝑦
B. 𝑦 > 80
C. 𝑥 ≥ 4𝑦
D. 𝑦 ≥ 4𝑥

3. Which among these is not a constraint for this LPP?


A. 𝑥 ≥ 0
B. 𝑥 + 𝑦 ≤ 200
C. 𝑥 ≥ 80
D. 4𝑥 − 𝑦 ≤ 0
Mathematics Page 214 of 260 XII
4. The profit when 𝑥 = 20 and 𝑦 = 80 is ……….
A. Rs. 60,000
B. Rs. 68,000
C. Rs. 64,000
D. Rs. 1,36,000

5. The maximum profit is ………..


A. Rs. 1,36,000
B. Rs. 1,28,000
C. Rs. 68,000
D. Rs. 1,20,000

CASE STUDY 2:

A dealer in rural area wishes to purchase a number of sewing


machines. He has only Rs. 5,760 to invest and has space for at most 20
items for storage. An electronic sewing machine cost him Rs. 360 and a
manually operated sewing machine Rs. 240. He can sell an electronic sewing
machine at a profit of Rs.22 and a manually operated machine at a profit of
Rs. 18. Assume that the electronic sewing machines he can sell is 𝑥 and that
of manually operated machines is 𝑦.

Mathematics Page 215 of 260 XII


1. The objective function is ……………
A. Maximize 𝑍 = 360𝑥 + 240𝑦
B. Maximize 𝑍 = 22𝑥 + 18𝑦
C. Minimize 𝑍 = 360𝑥 + 240𝑦
D. Minimize 𝑍 = 22𝑥 + 18𝑦

2. The maximum value of 𝑥 + 𝑦 is ………


A. 5760
B. 18
C. 22
D. 20

3. Which of the following is not a constraint?


A. 𝑥 + 𝑦 ≥ 20
Mathematics Page 216 of 260 XII
B. 360𝑥 + 240𝑦 ≤ 5,760
C. 𝑥 ≥ 0
D. 𝑦 ≥ 0

4. The profit is maximum when (𝑥, 𝑦) = ……….

A. (5, 15)
B. (8, 12)
C. (12, 8)
D. (15, 5)

5. The maximum profit is ………..


A. Rs. 5,760
B. Rs. 392
C. Rs. 362
D. Rs. 290

CASE STUDY 3

Mathematics Page 217 of 260 XII


Observe the above information and answer the following questions

1. One of these three inequalities is x + 2y ≤ 28 ,the other two


inequalities are
A. X + 3y ≤ 24 , x ≥ 2
B. 3X + y ≤ 24 , x ≥ 2
C. X - 3y ≤ 24 , x ≥ 2
D. 3X + 3y ≤ 24 , x ≥ 2

2. Given that point (x ,y) is in the feasible region, the maximum value of
Z= 20x + 10y is at point

A. (2,0)
B. (8,0)
C. (2,13)
D. (4,12)

Mathematics Page 218 of 260 XII


3. And the maximum value of Z= 20x + 10y is
A. 170
B. 200
C. 240
D. 430

4. Area of the feasible region is


A. 49 sq. units
B. 54 sq. units
C. 48 sq. units
D. 34 sq. units

5. Which of the following does not lie on/in the feasible region
A. (2,2)
B. (8,1)
C. (5,5)
D. (3,12)

SECTION C (3 MARKS)

1 Solve the linear programming problem graphically:


Maximise Z= 𝟒𝒙 + 𝒚
Subject to the constraints:
𝑥 + 𝑦 ≤ 50
3𝑥 + 𝑦 ≤ 90
𝑥 ≥ 0, 𝑦 ≥ 0

2 Solve the linear programming problem graphically:


Minimise Z= 200𝒙 + 500𝒚
Subject to the constraints:
𝑥 + 2𝑦 ≥ 10
3𝑥 + 4𝑦 ≤ 24
𝑥 ≥ 0, 𝑦 ≥ 0

Mathematics Page 219 of 260 XII


3 Solve the linear programming problem graphically:
Minimise and Maximise Z= 3𝒙 + 9𝒚
Subject to the constraints:
𝑥 + 3𝑦 ≤ 60
𝑥 + 𝑦 ≥ 10
𝑥 ≤ 𝑦 , 𝑥 ≥ 0, 𝑦≥0

4 Minimise Z= 3𝒙 + 2𝒚
Subject to the constraints:
𝑥+𝑦≥8
3𝑥 + 5𝑦 ≤ 15
𝑥 ≥ 0, 𝑦 ≥ 0

5 Maximise Z = 3x + 4y
subject to the constraints :
𝑥+𝑦≤4 , 𝑥 ≥ 0, 𝑦 ≥ 0

6 Minimise Z = – 3x + 4 y
subject to x + 2y ≤ 8, 3x + 2y ≤ 12, x ≥ 0, y ≥ 0.

7 Maximise Z = 5x + 3y

subject to 3x + 5y ≤ 15, 5x + 2y ≤ 10, x ≥ 0, y≥0

8 Minimise Z = x + 2y

subject to 2x + y ≥ 3, x + 2y ≥ 6, x ≥ 0, y ≥ 0.

9 Show that the minimum of Z occurs at more than two points.


Minimise and Maximise Z = 5x + 10 y
subject to x + 2y ≤ 120, x + y ≥ 60, x – 2y ≥ 0, x ≥ 0, y ≥ 0.
10 Minimise and Maximise Z = x + 2y
subject to x + 2y ≥ 100, 2x – y ≤ 0, 2x + y ≤ 200; x, y ≥ 0.

Mathematics Page 220 of 260 XII


SECTION D CBSE QUESTIONS

1 Solve the linear programming problem graphically:


Maximise Z= 60𝒙 + 15𝒚
Subject to the constraints:
𝑥 + 𝑦 ≤ 50
3𝑥 + 𝑦 ≤ 90 ,
𝑥 ≥0, 𝑦 ≥0
2 Solve the linear programming problem graphically:
Maximise Z= 2𝒙 + 5𝒚
Subject to the constraints:
2𝑥 + 4𝑦 ≤ 8
3𝑥 + 𝑦 ≤ 6
𝑥 + 𝑦 ≤ 4 , 𝑥 ≥ 0, 𝑦≥0

3 Solve the linear programming problem graphically:


Maximise Z= 20𝒙 + 10𝒚
Subject to the constraints:
𝑥 + 2𝑦 ≤ 28
3𝑥 + 𝑦 ≤ 24 ,
𝑥 ≥ 2,
𝑥 ≥ 0, 𝑦≥0
4 Solve the linear programming problem graphically:
Minimise Z= 𝑥 − 5𝑦 + 20
Subject to the constraints:
𝑥−𝑦≥0
−𝑥 + 2𝑦 ≥ 2
𝑥≥ 3
𝑦≤4 ,
𝑥 ≥ 0, 𝑦≥0
5 Minimize 𝑍 = 13𝑥 − 15𝑦 subject to the constraints:
𝑥+𝑦 ≤7,
2𝑥 − 3𝑦 + 6 ≥ 0
x ≥ 0, y ≥ 0

Mathematics Page 221 of 260 XII


Answer Key (MCQ)
LINEAR PROGRAMMING

MCQS

1 (C ) 2 (B ) 3 (D) 4 (B ) 5 (B )
6 (C ) 7 (B ) 8 (B ) 9(A ) 10 (D )
11 (C ) 12 (B ) 13 (D ) 14 (A ) 15 (B )
16 (C ) 17 (C) 18 (B) 19 (D) 20 (A)

CASE BASED QUESTIONS

CASE 1 (B) 2 (D ) 3 (C) 4 (B ) 5 (A )


STUDY 1
CASE 1 (B ) 2 (D ) 3 (A ) 4(B ) 5 (B )
STUDY 2
CASE 1 (B ) 2 ( D) 3 (B) 4( A) 5 (B)
STUDY 3

SECTION C

1 The maximum value of Z is 120 at the point (30, 0)

2 The minimum value of Z is 2300 at the point (4,3)

3 The maximum value of Z on the feasible region occurs at the two corner
points
C (15, 15) and D (0, 20) and it is 180 in each case

4 The problem is having no feasible region and hence no feasible


solution.

5 The maximum value of Z is 16 at the point (0,4)

Mathematics Page 222 of 260 XII


6 The minimum value of Z is -12 at the point (4,0)

7 235 20 45
The maximum value of Z is ( ) at the point ( , )
19 19 19

8 The value of Z is minimum at every point on the line, x + 2y = 6

9 The minimum value of Z is 300 at (60,0) and the maximum value of Z is


600 at all the points on the line segment joining (120,0) and (60,30)

10 The maximum value of Z is 400 at point (0, 200) and the minimum value
of Z is 100 at all the points on the line segment joining the points (0, 50)
and (20, 40)

ANSWERS SECTION D

1 The maximum value of Z is 1800 at the point (30, 0)

2 The maximum value of Z is 10 at the point (0,2)

3 The minimum value of z is at point D (4, 12) So, minimum value of z =


200

4 The minimum value of z is at point D (4,4) So, minimum value of z = 4

5 The maximum value of Z is – 30 at the point (0,2)

Mathematics Page 223 of 260 XII


1

The maximum value of Z is 120 at the point (30, 0)

Mathematics Page 224 of 260 XII


3

The maximum value of Z on the feasible region occurs


at the two corner points C (15, 15) and D (0, 20) and it i
180 in each case
4 The problem is having no feasible region and hence no
feasible solution.

Mathematics Page 225 of 260 XII


5

The maximum value of Z is 16 at the point (0,4)

Mathematics Page 226 of 260 XII


6

Mathematics Page 227 of 260 XII


7

Mathematics Page 228 of 260 XII


8

Mathematics Page 229 of 260 XII


9

Mathematics Page 230 of 260 XII


10

Mathematics Page 231 of 260 XII


SOLUTION AND HINT

SECTION C

SECTION D CBSE SOLUTIONS

Mathematics Page 232 of 260 XII


2

Mathematics Page 233 of 260 XII


3

Mathematics Page 234 of 260 XII


4

The minimum value of z is at point D(4,4) So, minimum value of z = 4

Mathematics Page 235 of 260 XII


5

Mathematics Page 236 of 260 XII


CHAPTER – 13
PROBABILITY

CONDITIONAL PROBABILITY:

Probability of an event E given that event F has already occurred.

𝐏(𝐄∩𝐅)
P(E/F)=) , Where P(F)≠0.
𝑷(𝑭)

Property 1: Let E and F be events of a samplE space S of an experiment, then


we have P(S|F) = P(F|F) = 1.
Property 2: f A and B are any two events of a sample space S and F is an
event of S such that P(F) ≠ 0, then P((A ∪ B)|F) = P(A|F) + P(B|F) – P((A ∩
B)|F).
Property 3: P(A′|B) = 1 − P(A|B)
EXAMPLE:In a school, there are 1000 students, out of which 430 are girls. It is
known that out of 430, 10% of the girls study in class XII. What is the probability
that a student chosen randomly studies in class XII given that chosen student is a
girl ?

Mathematics Page 237 of 260 XII


SOLUTION: Let A be the event that a student chosen randomly studies in class XII
and B be the event that the randomly chosen student is a girl.
P(A∩B)=P(Chosen students is a girl in class XII)= 43/1000 ,here n(A∩B)=10% of
430=43
P(B)= 430/1000
Required probability = P(A/B)= P(A∩B)/P(B) ==0.043/0.43 = 0.1

MULTIPLICATION THEOREM ON PROBABILITY:

P(E∩F)= P(E) .P(F/E)

= P(F) P(E/F) PROVIDED P(E) ≠ 0 ,P(F) ≠ 0

MULTIPLICATION RULE OF PROBABILITY FOR MORE THAN TWO EVENTS:

P(E∩F∩G)= P(E)P(F/E)P(G/EF)

EXAMPLE: An urn contains 20 red and 10 blue balls. Two balls are drawn
from a bag one after the other without replacement. What is the probability
that both the balls are drawn are red?

Solution: Let A and B denote the events that the first and the second balls
are drawn are red balls. We have to find P(A∩B) or P(AB).
P(A) = P(red balls in first draw) = 20/30
Now, only 19 red balls and 10 blue balls are left in the bag. The probability of
drawing a red ball in the second draw too is an example of conditional
probability where the drawing of the second ball depends on the drawing of
the first ball.
Hence Conditional probability of B on A will be,
P(B|A) = 19/29
By multiplication rule of probability,
P(A∩B) = P(A) × P(B|A)=(20/30 ) x (19/29) = 38/87

Mathematics Page 238 of 260 XII


INDEPENENT EVENTS:
Let E and F be two events associated with a sample space S. If the
probability of occurrence of one of them is not affected by the occurrence
of the other, then we say that the two events are independent.
Thus, two events E and F will be independent, if
(a) P (F | E) = P (F), provided P (E) ≠ 0
(b) P (E | F) = P (E), provided P (F) ≠ 0
(c) P (E ∩ F) = P (E) P (F)
For Three independent events A, B and C
(d) P (A ∩ B ∩ C) = P (A) P (B) P (C)

NOTE:1. If two events A nad B are mutually exclusive IF A and B are disjoint
sets i.e. no common element between them.

2.If A and B are two Independent events ,then

(i) A and B’ are independent

(ii) A’ and B are independent

(iii) A’ and B’ are independent

(iv)P(AUB) = 1-P(A’)P(B’)

Note: If E and F are dependent events, then P(E ∩ F) ≠ P(F) . P(F)

Mathematics Page 239 of 260 XII


EXAMPLE: Two dice are thrown together. Let A be the event ‘getting 6 on
the first die’ and B be the event ‘getting 2 on the second die’. Are the events
A and B independent?

Solution: A = {(6, 1), (6, 2), (6, 3), (6, 4), (6, 5), (6, 6)} B = {(1, 2), (2, 2), (3, 2),
(4, 2), (5, 2), (6, 2)}, A ∩ B = {(6, 2)}

P(A) 6/36 = =1/6, P(B) = 1/6 P(A∩ B) =1/ 36

Events A and B will be independent if P (A ∩ B) = P (A) P (B)

i.e 1/36 = (1/6)X (1/6), Hence, A and B are independent.

PARTITION OF A SAMPLE SPACE: A set of events E1 , E2 ,...., En is said to


represent a partition of a sample space S if

THEOREM OF TOTAL PROBABILITY:

P (A) = ∑𝑛𝑗=1 P(Ej )P(A | Ej ) ,j=1,2,3......n

= P(E1)P(A/E1)+P(E2)P(A/E2)+.........P(En)P(A/En)

BAYE’S THEOREM:

If E1, E2,…,En are n non-empty events which constitute a partition of sample


space S, and A is any event of non zero probability, then
Mathematics Page 240 of 260 XII
𝑨
𝑷(𝑬𝒊)𝑷( )
𝑬𝒊
P(Ei/A)= 𝑨 𝑨 𝑨 .
𝑷(𝑬𝟏)𝑷( )+𝑷(𝑬𝟐)𝑷( )+⋯……+𝑷(𝑬𝒏)𝑷( )
𝑬𝟏 𝑬𝟐 𝑬𝒏

EXAMPLE:

A car manufacturing factory has two plants, X and Y. Plant X manufactures


70% of cars and plant Y manufactures 30%. 80% of the cars at plant X and
90% of the cars at plant Y are rated of standard quality. A car is chosen at
random and is found to be of standard quality. What is the probability that it
has come from plant X?

Solution: Let E be the event that the car is of standard quality. Let B1 and B2
be the events that the car is manufactured in plants X and Y, respectively.

Now P (B1) = 70/ 100=7/ 10 = ,

P (B2) = 30/100 =3/10

P (E | B1) = Probability that a standard quality car is manufactured in = 80/


100=8/ 10

P (E | B2) = 90 /100=9/ 10 ,

P (B1 | E) = Probability that a standard quality car has come from plant X=
𝐸 7 8
𝑃(𝐵1)𝑃( ) 𝑋
𝐵1 10 10
= 7 8 3 9 = 56/83
P (E |B1 ) P (B1 ) + P (B2 ) .P (E |B2 ) 𝑋 + 𝑋
10 10 10 10

Hence the required probability is 56 /83 .

RANDOM VARIABLE: A random variable is a real-valued function, whose


domain is the sample space of a random experiment. Generally, denoted by
capital letter X.

Note: More than one random variables can be defined in the same sample
space. PROBABILITY DISTRIBUTION: The system in which the values of a
random variable are given along with their corresponding probabilities is

Mathematics Page 241 of 260 XII


called probability distribution. Let X be a random variable which can take n
values x1, x2,…, xn.

Let p1, p2,…, pn be the respective probabilities. Then, a probability


distribution table is given as follows:

X x1 x2 x3 ....... xn
P(X) p1 p2 p3 ....... pn

Here p1+p2+p3+....pn=1

Note: If xi is one of the possible values of a random variable X, then


statement X = xi is true only at some point(s) of the sample space. Hence
,the probability that X takes value x, is always non-zero, i.e. P(X = xi) ≠ 0.

MEAN OF RANDOM VARIABLE:

Let X be a random variable whose possible values are x1, x2 x3 x4…… ….xn
occur with probabilities are p1, p2 p3 p4…… ….pn respectively . Then mean
of X, denoted by µ , is the number ∑𝑛𝑖=1(𝑥𝑖 𝑃 (𝑥𝑖)) ie. The mean of a random
variable X is also called the expectation of X,

Denoted by E(X)= µ = ∑𝒏𝒊=𝟏(𝒙𝒊 𝑷(𝒙𝒊))) = 𝒙𝟏𝒑𝟏 + 𝒙𝟐 𝒑𝟐 + ⋯ . xn𝒑n

Q1. If P (A) = 0.8, P (B) = 0.5 and P (B|A) = 0.4, what is the value of P (A ∩
B)?

A. 0.32 B. 0.25 C. 0.1 D. 0.5


Q2. The probability of solving the specific problems independently by A
and B are 1/2 and 1/3 respectively. If both try to solve the problem

Mathematics Page 242 of 260 XII


independently, The probability that exactly one of them solves the
problem is :
A. 1 B. ½ C. ⅓ D. ¼
Q3. If A and B are two events such that P(A)≠0 and P(B/A)=1 ,Then

A A⊂ B B. B⊂A C.A=ᵠ D.B=ᵠ


Q4.The probability of simultaneous occurrence of at least one of two events
A and B is p. If the probability that exactly one of A, B occurs is q, then P (A′)
+ P (B′) = ?
A. p-2q B. q-2p+1 C. = 2 – 2p + q. D. 2 – 2q + p.
Q5. If A and B are two events such that P(A) = 1/ 2 , P(B) = 1/ 3 , P(A/B)= 1/4,
then P(A’∩ B ‘) equals to
A 1/ 12 B. 3/ 4 C. 1 /4 D. 3/ 16
Answers:
Q1.As we know that P(B/A)=P(A∩B)/P(A)  P(A∩B)=P(B/A) P(A)
So replacing the values,we get P(A∩B)= 0.32
So option A is correct.
Q2.P(Exactly one of them solve ths problem)= P(AB’)+P(A’B)= ½ X 2/3 +
1/3X1/2=1/2
So option B is correct.
Q3. P(A))≠ 0 and P(B∣A)=1
P(B∣A)=P(B∩A)/P(A)
1= P(B∩A)/P(A)
P(A)=P(B∩A)
⇒A⊂B
Thus, the correct option is A.

Q4. Since P (exactly one of A, B occurs) = q (given),


we get P (A∪B) – P ( A∩B) = q

Mathematics Page 243 of 260 XII


⇒ p – P (A∩B) = q ⇒ P (A∩B) = p – q ⇒ 1 – P (A′∪ B′) = p – q
⇒ P (A′∪ B′) = 1 – p + q
⇒ P (A′) + P (B′) – P (A′∩ B′) = 1 – p + q
⇒ P (A′) + P (B′) = (1 – p + q) + P (A′ ∩ B′)
= (1 – p + q) + (1 – P (A∪ B))
= (1 – p + q) + (1 – p)
= 2 – 2p + q.
So option C is correct.
Q5. P(A∣B)= P(A∩B)/P(B)
P(B∩A) =P(A/B)P(B)
=1/4X1/3=1/12
Now, P(A’ ∩ B’) = 1 – P(A ∪ B) = 1 – [P(A) + P(B) – P(A ∩ B)] [∵P(A ∪ B) = P(A)
+ P(B) – P(A ∩ B)]
Putting the values we get , P(A’ ∩ B’) = 3/12.=1/4
So option C is correct.
SOME MORE QUESTIONS FOR SELF ASSESSMENT:
1. If A and B are two events and A ≠ φ, B ≠ φ, then
A. P(A | B) = P(A).P(B) B. P(A | B) = P(A ∩B) /P(B)
C. P(A | B).P(B | A)=1 D. P(A | B) = P(A) | P(B)

2.If A and B are such events that P(A) > 0 and P(B) ≠ 1, then P( A′ | B′ )
equals.

A. 1 – P(A |B) B. 1– P( A′ | B)

C. 1–P(AU B)/P(B') D. P( A′ ) | P( B′ )

Mathematics Page 244 of 260 XII


3.If A and B are two independent events with P(A) = 3 /5 and P(B) = 4/9,
then P( A′ ∩ B′ ) equals

A. 4 /15 B. 8/ 45

C. 1/ 3 D. 2/ 9

4.For the following probability distribution:

X -4 -3 -2 -1 0
P(X) 0.1 0.2 0.3 0.4 0.5
E(X) is equal to :

A. 0 B –1 C –2 D
–1.8

5. The probability distribution of a discrete random variable X is given below:

X 2 3 4 5
P(X) 5K 7K 9K 11K
The value of k is

A8 B 16 C 32
D 48

State True and False:

1. If A and B are independent events, then A′ and B′ are also


independent.
2. Another name for the mean of a probability distribution is expected
value.

ANSWERS:

1.True 2. True

ASSERTION-REASON BASED QUESTIONS:

Mathematics Page 245 of 260 XII


In the following questions, a statement of assertion (A) is followed by a
statement of Reason (R). Choose the correct answer out of the following
choices.

(a) Both A and R are true and R is the correct explanation of A.

(b) Both A and R are true but R is not the correct explanation of A.

(c) A is true but R is false.

(d) A is false but R is true.

Q6. Assertion (A): When two coins are tossed simultaneously,then the
probability of getting no tail is ¼

Reason(R): The probability of getting a head in one toss of a coin is ½.

Q7.Assertion(A): The probability distribution of a random variable X can be


given by :

X 0 1 2 3 4
P(X) 0.1 0.5 0.2 -0.1 0.3

Reason(R) : ∑5𝑖=1 𝑃 (𝑥𝑖)=1

ANSWERS:

Q6 OPTION (a)

Explaination: P(H)=1/2 ,P(T)= ½

P(notail) = ½ X ½ = ¼

Q7.Option (d)

Explaination:

Mathematics Page 246 of 260 XII


Assertion is false as probability can never be <0

Q8. A and B are two candidates seeking admission in a college. The


probability that A is selected is 0.7 and the probability that exactly one of
them is selected is 0.6. Find the probability that B is selected.

Q9.10% of the bulbs produced in a factory are of red colour and 2% are red
and defective. If one bulb is picked up at random, determine the probability
of its being defective if it is red.

Q10.A box of oranges is inspected by examining three randomly selected


oranges drawn without replacement.If all the three oranges are good the
box is approved for sale,otherwise it is rejected.Find the probability that the
box containing 15 oranges out of which 12 are good ,is approved for sale?

Q11.Two cards are drawn successively with replacement from a well


shuffled pack of 52 cards.Find the probability distribution of number of aces.

Q12. Bag I contains 3 red and 4 black balls while another Bag II contains 5
red and 6 black balls. One ball is drawn at random from one of the bags and
it is found to be red. Find the probability that it was drawn from Bag II?

Q13.Suppose that 5% of men and 0.25% of women have grey hair. A grey
haired person is selected at random. What is the probability of this person
being male? Assume that there are equal number of males and females.

Solution(Q8):
Mathematics Page 247 of 260 XII
Let p be the probability that B gets selected. P (Exactly one of A, B is
selected) = 0.6 (given) ⇒ P (A is selected, B is not selected; B is selected, A is
not selected) = 0.6 ⇒ P (A∩B′) + P (A′∩ B) = 0.6

⇒ P (A) P (B′) + P (A′) P (B) = 0.6 ⇒ (0.7) (1 – p) + (0.3) p = 0.6 ⇒ p = 0.25

Thus the probability that B gets selected is 0.25.

Solution(Q9)

Let A and B be the events that the bulb is red and defective, respectively. P
(A) = 10/ 100 =1/10, P (A∩ B) =2/100 =1/50

P (B | A) = P (A∩ B) / P (A) =1/5

Thus the probability of the picked up bulb of its being defective, if it is red,
is 1/5.

Solution(Q10.)

It is given that in a box containing 15 oranges, 12 are good and 3 are bad.
Now let, the events that the first, second, and third drawn orange is good be
represented by A,BA,B and CC respectively.
Now, as given, there are 12 good oranges in the box.
So, the probability that the first drawn orange is good is,
P(A)=12/15
Also given that the oranges are not replaced.
So, the probability of getting the second orange good is,
P(B)=11/15
And, the probability of getting the third orange good is,
P(C)=10/15
Mathematics Page 248 of 260 XII
According to the question, the box is approved for sale if all three oranges
are good.
The probability of getting all three oranges good =P(A∩B∩C)=P(A)⋅P(B)⋅P(C)
⇒ The probability of getting all three oranges good =12/15⋅11/15⋅10/15

⇒ The probability of getting all three oranges good =44/91


⇒ The probability of getting all three oranges good =0.48
Hence, the probability of a box being approved for sale is 0.48

Solution(Q11):

The number of aces is a random variable. Let it be denoted by X. Clearly, X


can take the values 0, 1, or 2. Now, since the draws are done with
replacement, therefore, the two draws form independent experiments.
Therefore,

P(X = 0) = P(non-ace and non-ace) = P(non-ace) × P(non-ace) = 48/52 X


48/52= 144/169

P(X=1)=P(ace and non-ace or non-ace and ace)

= P(ace and non-ace) + P(non-ace and ace)

= P(ace). P(non-ace) + P (non-ace) . P(ace)

= 4/52 X 48/52+ 48/52 X 4/52 =24/169 and

P(X = 2) = P (ace and ace) = 4/52 X 4/ 52=1/ 169

Thus, the required probability distribution is :

X 0 1 2
P(X) 144/169 24/169 1/169
Mathematics Page 249 of 260 XII
Solution(Q12)

Let E1 be the event of choosing the bag I, E2 the event of choosing the bag
II and A be the event of drawing a red ball. Then P(E1 ) = P(E2 ) = 1/ 2

Also P(A|E1 ) = P(drawing a red ball from Bag I) = 3 /7 and P(A|E2 ) =


P(drawing a red ball from Bag II) = 5 /11

Now, the probability of drawing a ball from Bag II, being given that it is red,
is P(E2 |A)
P(E2 )P(A|E )
By using Bayes' theorem, we have P(E2 |A) = P(E1 )P(A|E1 )+P(E2 )P(A|E2 )
1 5
𝑋
2 11
= 1 3 1 5 = 35/68
X + 𝑋
2 7 2 11

Solution 13.

Let the events M, F and G be defined as follows:

Mathematics Page 250 of 260 XII


M: A male is selected
F: A female is selected
G: A person has grey hair
It is given that the number of males = the number of females
P(M)=P(F)=21
Now, P(G/M)= Probability of selecting a grey haired person given that they are
male =5%=5/100
Similarly, P(G/F)=0.25%=0.25/100
A grey haired person is selected at random, the probability that this person is a
male

𝐏(𝐌)×𝐏(𝐆∣𝐌)
=P(M∣G)=𝐏(𝐅)×𝐏( 𝐆∣𝐅 )+𝐏(𝐌)×𝐏(𝐆∣𝐌) using Baye's theorem

𝟏
/𝟏𝟎𝟎
𝟐𝐱𝟓
=𝟏 𝟓 𝟏 𝟎.𝟐𝟓
𝐱 + 𝐱
𝟐 𝟏𝟎𝟎 𝟐 𝟏𝟎𝟎

=20/21

Q14. A doctor is to visit a patient. From the past experience, it is known


that the probabilities that he will come by cab, metro, bike or by other
means of transport are respectively 0.3, 0.2, 0.1 and 0.4. The probabilities
that he will be late are 0.25, 0.3, 0.35 and 0.1 if he comes by cab, metro,
bike and other means of transport respectively

Mathematics Page 251 of 260 XII


.

Based on the above information, answer the following questions.


1.When the doctor arrives late, what is the probability that he comes by
metro ?

A. 5/14 B. 2/7 C. 5/21 D. 1/6

2.When the doctor arrives late, what is the probability that he comes by CAB
?

A. 4/21 B. 1/7 C. 5/14 D. 2/21

3.When the doctor arrives late, what is the probability that he comes by
BIKE?

A. 5/21 B. 4/7 C. 5/6 D. 1/6

4.What is the probability that doctor is late by any means?

A0 B 2 /105 C ½ D¼

Q15.A card is lost from a apck of 52 cards,from the reramining cards two
cards are drawn

Mathematics Page 252 of 260 XII


Based on the above information answer the following:

1.The probability of drawing two diamond cards ,given that a diamond card
has lost ,is

A.21/425 B.22/425 C.23/425 D.1/425

2. The probability of drawing two diamond cards ,given that a card of heart
has lost ,is

A.26/425 B.22/425 C.19/425 D.23/425

3.Let A be the event of drawing two diamonds from remaining 51 cards,and


E1,E2,E3,E4 are the events that lost card is of diamond,club,spade,heart
respectively.Then approximate value of ∑4𝑖=1 𝑃(𝐴/𝐸𝑖 ) is

A.0.17 B.0.24 C.0.25 D.0.18

4.All of a sudden ,the missing card is found and then two cards are drawn
simultaneously without replacement.Probability that both drawn cards are
king is

A.1/52 B.1/221 C.1/121 D.2/221

Q16.Shweta was doing a project on school survey,on the average number of


hours spent on study by the students selected at random.At the end of the
survey,Shweta prepared the following report related to data.

Let X denotes the average hours spent on study by students.The probability


of X ,that can take values x,has the following form,where k is some unknown
constant.

Mathematics Page 253 of 260 XII


0.2 ,if x=0

Kx ,if x=1 or 2

P(X=x)= k(6-x) if x=3 or 4

0, otherwise

Based on the above information answer the following questions:

1.The value of k is

A.0.1 B.0.2 C.0.3 D.0.05

2.What is the probability taht the average study hours of students is not
more than one hour

A.0.4 B.0.3 C.0.5 D. 0.1

3. What is the probability that the average study hours of students is at least
three hour?

A.0.5 B.0.9 C.0.8 D.0.1

4. What is the probability that the average study hours of students is at least
one hour?

Mathematics Page 254 of 260 XII


A.0.2 B.0.4 C.0.8 D.0.6

Q17. A factory has three machines A, B and C, which produce 30%, 20% and
50% of items of a particular type daily. The machines produce 5%, 2% and
4% defective items respectively. One day when the production was over, an
item was picked up randomly and it was found to be defective

Based on the above information ,answer the following questions:


1.Probability that defective item drawn was produced by Machine A is:
A.4/13 B. 5/13 C.6/13 D.9/13
2. Probability that defective item drawn was produced by Machine B is
A.0.3 B. 0.1 C.0.2 D.0.4
3. Probability that defective item drawn was produced by Machine C is
A.16/39 B.17/39 C. 20/39 D.15/39
4. Probability that defective item drawn was NOT produced by Machine B is
A.35/39 B.61/39 C.41/39 D. None of these
Solution 14:

Suppose that,

T1 : Event that doctor come by cab

T2: Event that doctor come by metro

T3: Event that doctor come by bike

T4: Event that doctor come by other means

E: Event that doctor comes late


Mathematics Page 255 of 260 XII
Now,
Probability that doctor come by cab P(T1)=3/10
Probability that doctor comes late if he comes by cab
P(E/T1)=1/4
Now,
Probability that doctor comes by Metro P(T2)=1/5
Probability that doctor comes late if he comes by Metro P(E/T2) = 3/10
Probability that doctor comes by Bike =P(T3)=1/10
Probability that doctor comes late if he comes by Bike P(E/T3)=7/20

Now, Probability that doctor comes by other means of transport ⇒P(T4)=2/5


Probability that doctor comes late if he comes by other means of transport
P(E/T4))=1/10
then , put the values in formula.

𝐏(𝐓𝟐)×𝐏(𝐄∣𝐓𝟐)
P(T2/E)= 𝐄 𝐄
𝐏(𝐓𝟏)×𝐏( 𝐄∣∣𝐓𝟏 )+𝐏(𝐓𝟐)×𝐏( 𝐄∣∣𝐓𝟐 )+ 𝐏(𝐓𝟑)𝐏( )+𝐏(𝐓𝟒)𝐏( )
𝐓𝟑 𝐓𝟒

Putting the values we get


1.option B is correct
Similarly for
2.option C is correct
3.option D is correct
4.option B is correct

Solution15:
1.Required prob= 12/51 x 11/50= 22/425
Option B is correct.
2.Required prob= 13/51 x 12/50 =26/425
Option A is correct
Mathematics Page 256 of 260 XII
1. 0.24
Option B is correct
44/52 x 3/51 = 1/221
Option B is correct.
Solution16:
1. As we know that,
∑ 𝑃𝑖=1,So 0.2+k+2k+3k+2k+0=1
We get k=0.1
Option A is correct
2.P(X=0)+P(X=1)=0.2+0.1=0.3
Option B is correct
3.P(X=3)+P(X=4)=0.3+0.2=0.5
So option A is correct
4.1-P(X=0)=1-0.2=0.8
So option C is correct.
Solution 17.

Let E1,E2,E3 be the events of producing the item by machines A,B,C


respectively

P(E1)=30/100 , P(E2)=20/100 ,P(E3)=50/100

LetE be the event of drawing the defective item

Then,P(E/E1)=5/100=1/20 ,P(E/E2)= 2/100=1/50 P(E/E3)= 4/100=1/25

Now using Baye’s theorem,we get

1.Option B is correct

2.Option B is correct

Mathematics Page 257 of 260 XII


3.Option C is correct

4.P(The item is not produced by Machine B )=P(item is producec by machine


A or B )

= 15/39 + 20/39= 35/39

So option A is correct.

Q1. The probabilities that A and B will die within a year are p and q,
respectively, then the probability that only one of them will be alive at the
end of the year is __________.

Solution: Required probability is P [(A will die and B alive) or (B will die and A
alive)]
= P [(A ∩ B′) ∪ (B ∩ A′)]
Since events are independent, so
Required probability= P (A) * P (B′) + P (B) * P (A′)
= p * (1 − q) + q * (1 − p)
= p + q − 2pq
Q2. If A and B are two independent events such that P (A ∩ B′) = 3 / 25 and
P (A′ ∩ B) = 8 / 25, then P (A) = _______.
Solution:
Since events are independent.
So, P (A ∩ B′) = P (A) × P (B′) = 3 / 25
⇒ P (A) × {1 − 2 P (B)} = 3 / 25 ….(i)
Similarly, P (B) × {1 − P (A)} = 8 / 25 ….(ii)
Mathematics Page 258 of 260 XII
On solving (i) and (ii), we get
P (A) = 1/5 and 3/5
Q3. If the probability of X to fail in the examination is 0.3 and that for Y is
0.2, then the probability that either X or Y failing in the examination is
______.
Solution:
Here P (X) = 0.3; P (Y) = 0.2
Now P (X ∪ Y) = P (X) + P (Y) − P (X ∩ Y)
Since these are independent events, so P (X ∩ Y) = P (X) * P (Y)
Thus required probability = 0.3 + 0.2 − 0.06 = 0.44
Q4. If a is an integer lying in [−5, 30], then the probability that the graph y
= x2 + 2 (a + 4) x − 5a + 64 is strictly above the x-axis is __________.
Solution:
x2 + 2 (a + 4) x − 5a + 64 ≥ 0
If D ≤ 0, then (a + 4)2 − (−5a + 64) < 0 Or
a2 + 13a − 48 < 0 Or
(a + 16) (a − 3) < 0
⇒ −16 < a < 3 ⇔ −5 ≤ a ≤ 2
Then, the favourable cases are equal to the number of integers in the
interval [−5, 2], i.e., 8.
The total number of cases is equal to the number of integers in the interval
[−5, 30], i.e., 36.
Hence, the required probability is 8 / 36 = 2 / 9.

Q5. There are three bags B1, B2, and B3. The bag B1 contains 5 red and 5
green balls, B2 contains 3 red and 5 green balls, and B3 contains 5 red and 3
green balls, Bags B1, B2 and B3 have probabilities 3/10, 3/10, and 4/10

Mathematics Page 259 of 260 XII


respectively of being chosen. A bag is selected at random and a ball is
chosen at random from the bag. Then which of the following options is/are
correct?
(a) Probability that the selected bag is B3 and the chosen ball is green equals
3/10
(b) Probability that the chosen ball is green equals 39/80
(c) Probability that the chosen ball is green, given that the selected bag is B 3,
equals 3/8
(d) Probability that the selected bag is B3, given that the chosen balls is
green, equals 5/13
Solution:
(i) P(B3⋂G) = P(G/B3).P(B3)
= (⅜) × (4/10)
= 3/20
(ii) P(G) = (5/10)×(3/10) + (⅝)×(3/10) + (⅜)×(4/10)
= (60 + 75 + 60)/400
= 195/400
= 39/80
(iii) P(G/B3) = 3/8
(iv) P(B3/G) = P(G⋂B3)/P(G)
= (3/20)/(39/80)
= 4/13
Hence option b and c are correct.

Mathematics Page 260 of 260 XII

You might also like